You are on page 1of 78

Topic Name: Modern History

Q1. Which of the following Wars ended with “The Treaty of Aix-La पर सट थोम म महफूज खान के नेतृ म मजबूत भारतीय सेना को
Chapelle”?
हराया। यह भारत म यूरोपीय लोगों के िलए एक आं ख खोलने वाला था,
(a) Second Anglo-Mysore War
(b) First Anglo-Mysore War इससे पता चला िक एक छोटी सी अनुशािसत सेना भी एक बड़ी भारतीय

(c) Third Carna c War


सेना को आसानी से हरा सकती है । इसके अलावा, इस यु ने द नम
(d) First Carna c War and Austrian War of Succession
एं ो- ांसीसी संघष म नौसैिनक बल के मह को पया प से सामने
(e) None of the above/More than one of the above
Answer: (d) लाया।
Op on (d) is correct: The First Carna c War ended in 1748 when the Q2. Which of the following was the most important outcome of the
Treaty of Aix-La Chapelle was signed bringing the Austrian War of Third Ba le of Panipat?
Succession to a conclusion. Under the terms of this treaty, Madras (a) It laid the founda on of the Mughal Empire.
was handed back to the English, and the French, in turn, got their (b) Ahmad Shah Abdali became the ruler of India.
territories in North America. (c) It ended the Maratha's ambi on of ruling over India.
The First Carna c War is remembered for the Ba le of St. Thome (in (d) It prepared the ground for the East India Company to rule the
Madras) fought between the French forces and the forces of whole of India.
Anwar-ud-din, the Nawab of Carna c, to whom the English appealed (e) None of the above/More than one of the above
for help. A small French army under Captain Paradise defeated the Answer: (c)
strong Indian army under Mahfuz Khan at St. Thome on the banks of Op on (c) is correct): On Yamuna's banks and between the expansive
the River Adyar. This was an eye-opener for the Europeans in India, it plains of the Ganga and Indus rivers in modern-day Haryana, Panipat
revealed that even a small disciplined army could easily defeat a much and its surrounding area have witnessed innumerable ba les. These
larger Indian army. Further, this war adequately brought out the wars changed the course of Indian history at different mes. The First
importance of naval force in the Anglo-French conflict in the Deccan. Ba le of Panipat was fought in 1526 between Babur and Ibrahim Lodi.
Q1. िन िल खत म से कौन सा यु "ए -ला शापेल की संिध" के साथ The conflict laid the founda on for the Mughal Empire by removing

समा आ? the Delhi Sultanate from power. Akbar and Hemu engaged in the

(a) दू सरा एं ो-मैसूर यु Second Ba le of Panipat in 1556, which resulted in a victory for
Mughal supremacy. The Marathas' ambi on to govern India was
(b) थम आं -मैसूर यु
crushed in the Third Ba le of Panipat, which took place in 1761
(c) तीसरा कनाटक यु between them and Ahmad Shah Abdali.

(d) थम कनाटक यु और ऑ याई उ रािधकार का यु Q2. िन िल खत म से कौन सा पानीपत की तीसरी लड़ाई का सबसे

(e) उपरो म से कोई नही/ं उपरो म से एक से अिधक मह पूण प रणाम था?

(a) इसने मुगल सा ा की नी ंव रखी।

उ र: (d)
(b) अहमद शाह अ ाली भारत का शासक बना।

िवक (d) सही है :


(c) इसने भारत पर शासन करने की मराठों की मह ाकां ा को समा

पहला कनाटक यु 1748 म समा आ जब ऐ -ला शापेल की संिध


कर िदया।

पर ह ा र िकए गए, िजससे ऑ याई उ रािधकार यु समा हो


(d) इसने पूरे भारत पर ई इं िडया कंपनी के शासन के िलए जमीन तैयार

गया। इस संिध की शत के तहत, म ास को वापस अं ेजों को सौ ंप िदया


की।

गया, और बदले म, ांसीसी को उ री अमे रका म अपना े िमल गया।


(e) उपरो म से कोई नही/ं उपरो म से एक से अिधक

थम कनाटक यु को सट थोम (म ास म) की लड़ाई के िलए याद िकया


उ र: (c)

जाता है , जो ांसीसी सेना और कनाटक के नवाब अनवर-उद-दीन की


िवक (c) सही है ):

सेनाओं के बीच लड़ी गई थी, िजनसे अं ेजों ने मदद की अपील की थी।

कै न पैराडाइज के तहत एक छोटी ांसीसी सेना ने अडयार नदी के तट


Topic Name: Modern History
यमुना के तट पर और आधुिनक ह रयाणा म गंगा और िसंधु निदयों के Q4. Who among the following was given the tle of ‘Rangeela’ due
to his luxurious lifestyle?
िव ृत मैदानों के बीच, पानीपत और इसके आसपास के े म
(a) Ahmad Shah
अनिगनत यु ए ह। इन यु ों ने अलग-अलग समय पर भारतीय (b) Rafi-ud-Daula
(c) Tipu Sultan
इितहास की धारा को बदल िदया। पानीपत का थम यु 1526 म बाबर
(d) Jahandar Shah
और इ ािहम लोदी के बीच लड़ा गया था। संघष ने िद ीस नत को
(e) None of the above/More than one of the above
स ा से हटाकर मुगल सा ा की नी ंव रखी। अकबर और हे मू 1556 म Answer: (e)
Op on (e) is correct: A er Rafi-ud- Daula passing, the Sayyid Brothers
पानीपत की दू सरी लड़ाई म शािमल ए, िजसके प रणाम प मुगल
selected Raushan Akhtar to succeed them. Muhammad Shah
वच की जीत ई। भारत पर शासन करने की मराठों की मह ाकां ा (1719–48) was given the nickname "Rangeela" because of his opulent

पानीपत की तीसरी लड़ाई म कुचल दी गई थी, जो 1761 म उनके और lifestyle. Nizam-ul-Mulk assisted Muhammad Shah in killing the Sayyid
Brothers. In 1724, Nizam-ul-Mulk was elected as the wazir, and
अहमद शाह अ ाली के बीच ई थी।
Hyderabad became a sovereign state. Invading Delhi in 1737 was Baji
Q3. The Congress Ministries resigned from office in November 1939 Rao I, the Maratha Peshwa, who had a small force of 500 horsemen.
due to: During the Ba le of Karnal in 1739, Nadir Shah routed the Mughals,
(a) Failure of Government to manage Infla on. captured Muhammad Shah, and annexed lands west of the Indus into
(b) Viceroy, on its own, had made India a par cipant in the the Persian empire.
imperialist war without consul ng Congress.
Q4. िन िल खत म से िकसे उनकी शानदार जीवन शैली के कारण
(c) Communal riots broke out in Madras Province.
(d) Non-Coopera on of Muslim League in administra on. 'रं गीला' की उपािध दी गई थी?

(e) None of the above/More than one of the above (a) अहमद शाह
Answer: (b)
(b) रफी-उद-दौला
Op on (b) is correct: In November 1939, the Congress Ministries
resigned, claiming that the Viceroy had unilaterally rendered India a (c) टीपू सु ान

par cipant in the imperialist war without consul ng the Congress. (d) जहांदार शाह
This was called "deliverance day" by the Muslim League under Jinnah.
(e) उपरो म से कोई नही/ं उपरो म से एक से अिधक
Q3. नवंबर 1939 म कां ेस मंि म लों ने कायालय से इ ीफा दे िदया,
उ र: (e)
इसके कारण:
िवक (e) सही है : रफ़ी-उद-दौला की मृ ु के बाद, सै द बंधुओं ने रौशन
(a) मु ा ीित का बंधन करने म सरकार की िवफलता।
अ र को अपना उ रािधकारी चुना। मुह द शाह (1719-48) को
(b) वायसराय ने कां ेस से परामश िकए िबना अपने दम पर भारत को उसकी भ जीवन शैली के कारण "रं गीला" उपनाम िदया गया था।
िनज़ाम-उल-मु ने सै द बंधुओ ं को मारने म मुह द शाह की सहायता
सा ा वादी यु म भागीदार बनाया था।
की। 1724 म, िनज़ाम-उल-मु को वज़ीर के प म चुना गया और
(c) म ास ांत म सा दाियक दं गे भड़क उठे । है दराबाद एक सं भु रा बन गया। 1737 म िद ी पर आ मण करने

(d) शासन म मु म लीग का असहयोग। वाला मराठा पेशवा बाजीराव थम था, िजसके पास 500 घुड़सवारों की
एक छोटी सी सेना थी। 1739 म करनाल की लड़ाई के दौरान, नािदर शाह
(e) उपरो म से कोई नही/ं उपरो म से एक से अिधक
ने मुगलों को भगाया, मुह द शाह को बंदी बना िलया और िसंधु के पि म
उ र: (b) की भूिम को फारस सा ा म िमला िलया।
Q5. Under whose leadership did the Kingdom of Bharatpur emerge
िवक (b) सही है :
as a strong State?
नवंबर 1939 म, कां ेस मं ालयों ने यह दावा करते ए इ ीफा दे िदया
(a) Rajaram I
िक वायसराय ने कां ेस से परामश िकए िबना भारत को एकतरफा प (b) Suraj Mal
(c) Raja Nahar Singh
से सा ा वादी यु म भागीदार बना िदया था। इसे िज ा के अधीन
(d) Todar Mal
मु म लीग ारा "मु िदवस" कहा जाता था।
Topic Name: Modern History
(e) None of the above/More than one of the above Op on (e) is correct: The four major European powers—Portuguese,
Answer: (b) Dutch, Bri sh, and French—arrived in India to establish trade rela ons
Op on (b) is correct: The Jats were prosperous farmers, and in the and later acquired colonial possessions. An outline of how European
region they controlled, towns like Panipat and Ballabgarh developed companies were founded:
into significant trading hubs. Under the direc on of Suraj Mal, the 1. Portuguese (1498)
kingdom of Bharatpur developed into a significant na on. Following 2. English East India Company (1600)
Nadir Shah's invasion of Delhi in 1739, many prominent ci zens of the 3. Dutch East India Company (1602)
city sought refuge. Son of Suraj Mal, Jawahir Shah, gathered 20,000 4. Danish East India Company (1616)
Maratha and 15,000 Sikh soldiers in addi on to his own 30,000 troops 5. French East India Company (1664)
to fight the Mughals. Q6. . िन िल खत यूरोपीय श यों को उनके भारत आगमन के
Q5. िकसके नेतृ म भरतपुर रा एक श शाली रा के पम
काल म म व थत कर:
उभरा?
1. डच
(a) राजाराम थम
2. च
(b) सूरज मल
3. अं ेजी
(c) राजा नाहर िसंह
4. पुतगाली
(d) टोडर मल
नीचे िदए गए कूट का योग कर सही उ र चुिनए:
(e) उपरो म से कोई नही/ं उपरो म से एक से अिधक (a) 4-2-3-1

उ र: (b) (b) 1-2-4-3


(c) 1-2-3-4
िवक (b) सही है :
(d) 4-3-2-1
जाट समृ िकसान थे, और उनके िनयं ण वाले े म, पानीपत और (e) उपरो म से कोई नही/ं उपरो म से एक से अिधक

ब भगढ़ जैसे शहर मह पूण ापा रक क ों के प म िवकिसत ए। उ र-(e)

सूरज मल के िनदशन म, भरतपुर का रा एक मह पूण रा के प म िवक (e) सही है :

िवकिसत आ। 1739 म नािदर शाह के िद ी पर आ मण के बाद, शहर चार मुख यूरोपीय श यां-पुतगाली, डच, ि िटश और

के कई मुख नाग रकों ने शरण ली। सूरज मल के बेटे, जवािहर शाह ने ांसीसी- ापार संबंध थािपत करने के िलए भारत आईं और बाद म

मुगलों से लड़ने के िलए अपने 30,000 सैिनकों के अलावा 20,000 मराठा औपिनवेिशक संपि हािसल कर ली। यूरोपीय कंपिनयों की थापना की

और 15,000 िसख सैिनकों को इक ा िकया। एक परे खा:


Q6. Arrange the following European powers in the chronology of
1. पुतगाली (1498)
their arrival in India:
1. Dutch 2. अं ेजी ई इं िडया कंपनी (1600)

2. French 3. डच ई इं िडया कंपनी (1602)


3. English
4. डे िनश ई इं िडया कंपनी (1616)
4. Portuguese
Select the correct answer using the code given below: 5. चई इं िडया कंपनी (1664)

(a) 4-2-3-1
(b) 1-2-4-3 Q7. The term “Factory” in the 18th and 19th centuries was used to
(c) 1-2-3-4 denote:
(d) 4-3-2-1 (a) A company for the produc on of Indigo and ar ficial dyes.
(e) None of the above/More than one of the above (b) A for fied area for the protec on of kings under the subsidiary
Answer-(e) alliance.
Topic Name: Modern History
(c) A place where goods were stored and from where traders used Portuguese manufacturing centres. Masulipatnam (1605), Pulicat
to work. (1610), Surat (1616), Bimlipatam (1641), Karaikal (1645), Chinsurah
(d) A company for the produc on of all the goods sold by the Bri sh (1653), Cossimbazar (Kasimbazar), Baranagore, Patna, Balasore,
traders in India. Nagapatam (1658), and Cochin were all Dutch manufacturing centres
(e) None of the above/More than one of the above (1663).
Answer-(c) Q8. कालीकट (कोिझकोड), कोचीन और क ानोर (क ूर) जैसी जगहों
Op on (c) is correct: On the banks of the river Hugli, the first English
पर कारखाने शु म िकसके ारा थािपत िकए गए थे?
factory was built in 1651. The Company's dealers, referred to at the
me as "factors," operated out of this loca on. The factory contained (a) ि िटश
offices where Company representa ves sat in addi on to a storage
(b) च
where goods for export were kept. The Company convinced traders
(c) पुतगाली
and merchants to move close to the factory as trade increased. It
started construc ng a fort around the village in 1696. (d) डच

Q7. . 18वी ं और 19वी ं शता ी म "कारखाना" श का योग (e) उपरो म से कोई नही/ं उपरो म से एक से अिधक

िन िल खत को िन िपत करने के िलए िकया गया था: उ र: (c)


(a) नील और कृि म रं गों के उ ादन के िलए एक कंपनी।
िवक (c) सही है :
(b) सहायक गठबंधन के तहत राजाओं की सुर ा के िलए एक मजबूत
सूरत (1613), आगरा, अहमदाबाद और भ च, बंबई, म ास और
े ।
कलक ा भी अं ेजी कारखानों के थल थे।
(c) वह थान जहाँ माल रखा जाता था और जहाँ से ापारी काम करते
थे। सूरत, मसूलीप नम और पांिडचेरी ां स के थल थे । कालीकट

(d) भारत म ि िटश ापा रयों ारा बेचे जाने वाले सभी सामानों के (कोिझकोड), कोचीन, क ानोर (क ूर), गोवा और दमन पुतगाली
उ ादन के िलए एक कंपनी।
िविनमाण क थे। मसूलीप नम (1605), पुिलकट (1610), सूरत (1616),
(e) उपरो म से कोई नही/ं उपरो म से एक से अिधक
िबमलीप म (1641), कराईकल (1645), िचनसुराह (1653), कोिसमबाजार
उ र-(c)
(कािसम बाजार), बारानागोर, पटना, बालासोर, नागाप म (1658), और
िवक (c) सही है :
कोचीन (1663) सभी डच िनमाण क थे ।
गली नदी के तट पर, पहला अं ेजी कारखाना 1651 म बनाया गया था।
कंपनी के डीलर, िजसे उस समय "फै र" कहा जाता था, इस थान से
संचािलत होता था । कारखाने म ही कायालय थे जहां कंपनी के ितिनिध Q9. Consider the following pairs:
बैठते थे । यहाँ भंडारण के अलावा िनयात के िलए सामान भी रखा जाता Colonial Powers Europeans Centre of influence in India

था । ापार बढ़ने पर कंपनी ने ापा रयों को कारखाने के नजदीक बसने 1. French : Chanderanagor

के िलए मना िलया । इसने 1696 म गांव के चारों ओर एक िकले का 2. Portuguese : Malabar coast

िनमाण शु िकया । 3. Dutch : Goa

Q8. Factories at places like Calicut (Kozhikode), Cochin, and Which of the pairs given above is/are correctly matched?
Cannanore (Kannur) were established ini ally by? (a) 2 and 3 only

(a) The Bri sh (b) 3 only

(b) The French (c) 1 and 2 only

(c) The Portuguese (d) 1 and 3

(d) The Dutch (e) None of the above/More than two of the above

(e) None of the above/More than one of the above Answer: (e)

Answer: (c) Op on (e) is correct: Europeans were the dominant colonial force in

Op on (c) is correct: Surat (1613), Agra, Ahmedabad and Broach, India. The French-run enterprises included Chandernagor in Bengal,
Bombay, Madras, and Calcu a were also sites of English factories. Mahe on the Malabar Coast, and Pondicherry, Karaikal, and Yanam on

Surat, Masulipatnam, and Pondicherry are in French. Calicut the Coromandel Coast. (Pair 1 has the appropriate match)
(Kozhikode), Cochin, Cannanore (Kannur), Goa, and Daman were
Topic Name: Modern History
Portuguese had control over the coastal regions, and their superior डच मालाबार, िजसे आमतौर पर कोचीन के नाम से जाना जाता है , 1661
naval might was very helpful. By the end of the 16th century, the
और 1795 के बीच मालाबार तट पर डच ई इं िडया कंपनी की कमान
Portuguese had taken control of a significant por on of the Indian
coastline in addi on to Goa, Daman, Diu, and Salse e. (Pair 2 has the का नाम था। यह संपूण डच भारत के प म संदिभत एक िह ा था।

appropriate match)
(जोड़ी 3 का उपयु िमलान है ।)
The se lements of Tranquebar (Tamil Nadu), Serampore (West
Bengal), and the Nicobar Islands were among the Danish possessions
Q10. Fierce ba les among the trading companies of Europe is an
in India.
important feature of the 17th and 18th centuries. What led to these
Dutch Malabar, commonly known as Cochin, was the name of the
ba les?
Dutch East India Company's commandment on the Malabar Coast
(a) All the companies were interested in buying the same things
between 1661 and 1795. It was a por on of what was referred to as
(b) Urge to secure markets
Dutch India as a whole. (Pair 3 has the appropriate match.)
(c) Urge to increase profit
Q9. . िन िल खत यु ों पर िवचार कीिजएः
(d) statements (b) and (c) are correct in this context
औपिनवेिशक श याँ भारत म यूरोपीय भाव के क (e) None of the above/More than one of the above
Answer: (e)
1. च: चं नगर
Op on (e) is correct: A number of violent wars between European
2. पुतगाली : मालाबार तट commerce enterprises occurred in the 17th and 18th centuries. The

3. डच : गोवा problem was that each company wanted to purchase iden cal
products. India produced high-quality co on and silk that found a
ऊपर िदए गए यु ों म से कौन-सा/से सही सुमेिलत है /ह?
sizable market in Europe. Addi onally in high demand were spices like
(a) केवल 2 और 3 pepper, cloves, cardamom, and cinnamon.
The prices at which these commodi es could be purchased
(b) केवल 3
subsequently climbed as a result of the compe on among European
(c) केवल 1 और 2
businesses, lowering the profits that could be made. The elimina on
(d) 1 और 3 of their compe tors was the only way for the trading corpora ons to
prosper.
(e) उपरो म से कोई नही/ं उपरो म से दो से अिधक
Trade corpora ons got into violent fights because they wanted to
उ र: (e) defend their markets. In the 17th and 18th centuries, they frequently

िवक (e) सही है : sank one other's ships, blockedade trade routes, and stopped vying
for ships carrying supplies of goods. Trading posts were built to defend
यूरोपीय भारत म मुख औपिनवेिशक श थे। ांसीसी ारा संचािलत
them because trade involved the use of weapons. This effort to
ापार के ों म बंगाल म चं नगर, मालाबार तट पर माहे , और protect ci es and carry on prosperous trade also caused a lot of
conflict with regional lords. The East India Company, therefore, strove
कोरोमंडल तट पर पांिडचेरी, कराईकल और यनम शािमल थे। (जोड़ी 1
to separate commerce from poli cs.
का उपयु िमलान है )
Q10. यूरोप की ापा रक कंपिनयों के बीच भयंकर यु 17वीं और 18वीं
पुतगािलयों का तटीय े ों पर िनयं ण था, और उनकी े नौसैिनक
शता ी की एक मह पूण िवशेषता है । इन लड़ाइयों के कारण ा आ?
श ब त मददगार थी। 16वी ं शता ी के अंत तक, पुतगािलयों ने गोवा,
(a) सभी कंपिनयां समान चीज खरीदने म िच रखती थीं
दमन, दीव और सालसेट के अलावा भारतीय समु तट के एक मह पूण
(b) बाजारों को सुरि त करने का आ ह
िह े पर िनयं ण कर िलया था। (जोड़ी 2 का उपयु िमलान है )
(c) लाभ बढ़ाने का आ ह
टां ूबार (तिमलनाडु ), सेरामपुर (पि म बंगाल) और िनकोबार ीप
(d) िवक (b) और (c) इस संदभ म सही ह
समूह की ब याँ भारत म डे िनश संपि म से एक थी ं।
(e) उपरो म से कोई नहीं/उपरो म से एक से अिधक

उ र: (e)
Topic Name: Modern History
िवक (e) सही है : उ र: (a)

17वीं और 18वीं शता ी म यूरोपीय वािण उ मों के बीच कई िहं सक यु िवक (a) सही है :

ए। सम ा यह थी िक ेक कंपनी समान उ ाद खरीदना चाहती थी। भारत वां डीवाश की लड़ाई तीसरे कनाटक यु की िनणायक लड़ाई थी। यह अं ेजों

उ गुणव ा वाले कपास और रे शम का उ ादन करता था िजसे यूरोप म एक ारा 22 जनवरी 1760 को तिमलनाडु के वां डीवाश म जीता गया था। अं ेजों के

बड़ा बाजार िमला आ था। इसके अित र उ मां ग म काली िमच, लौंग, जनरल आइरे कूट ने काउं ट थॉमस आथर डी लैली के तहत ां सीसी सेना को

इलायची और दालचीनी जैसे मसाले थे। भगा िदया और बु ी को एक कैदी के प म रख िलया।

िजन कीमतों पर इन व ुओं को बाद म खरीदा जा सकता था, वे यूरोपीय Q12. ‘Black Hole Tragedy’ related to?
(a) Famines of 1896–97
वसायों के बीच ित धा के प रणाम प चढ़ गए, िजससे मुनाफा कम हो
(b) Ba le of Plassey
सकता था। ापा रक िनगमों के समृ होने का एकमा तरीका उनके (c) Ba le of Buxar

(d) Ba le of panipat
ित िधयों का सफाया ही था।इसिलए ापार िनगम िहं सक झगड़ों म पड़ गए
(e) None of the above/More than one of the above
ोंिक वे अपने बाजारों की र ा करना चाहते थे। 17वीं और 18वीं शता ी म,
Answer: (b)
वे बार-बार एक-दू सरे के जहाजों को डु बा दे ते थे, ापार माग को अव कर Op on (b) is correct: Black Hole Tragedy: Fort William was besieged
by Nawab Siraj-ud-Daulah on 15 June 1756. Following the usual
दे ते थे, और माल की आपूित करने वाले जहाजों के िलए होड़ करना बंद कर दे ते
customs of war, some of the English women and children in Calcu a
थे। उनकी र ा के िलए ापा रक चौिकयां बनाई गईं ोंिक ापार म were kept in the prison cell of the fort. The number of prisoners were

हिथयारों का इ ेमाल होता था। शहरों की र ा करने और समृ ापार को 146 and the dimensions of the prison are stated to be 18 feet long by
14 feet 10 inches wide. On June 20, 146 white prisoners were locked
जारी रखने के इस यास के कारण भी े ीय ािमयों के साथ ब त संघष
up, when the prison room was opened the next morning, only 23
आ। इसिलए, ई इं िडया कंपनी ने वािण को राजनीित से अलग करने का people survived; the rest trampled each other for places near the
window. The East India officials used the episode as a propaganda tool
यास िकया।
to discredit the Nawab and supported Bri sh public opinion for the
War of aggression. A er news of Calcu a's capture was received by
Q11. . The famous Ba le of Wandiwash was won by:
the Bri sh in Madras in August 1756, Lieutenant Colonel Robert Clive
(a) The Bri sh
was sent to retaliate against the Nawab. With his troops and local
(b) The French
Indian allies, Clive recaptured Calcu a in January 1757, and defeated
(c) The Portuguese
Siraj ud-Daulah at the Ba le of Plassey, which resulted in Siraj being
(d) The Dutch
overthrown as Nawab of Bengal and executed.
(e) None of the above/More than one of the above
12. ' ैक होल ासदी' िकससे संबंिधत है ?
Answer: (a)
Op on (a) is correct: The ba le of Wandiwash was the decisive ba le (a) 1896-97 के अकाल
of the Third Carna c War. It was won by the English on January 22
(b) ासी की लड़ाई
1760, at Wandiwash (or Vandavasi) in Tamil Nadu. General Eyre Coote
of the English routed the French army under Count Thomas Arthur de (c) ब र की लड़ाई

Lally and took Bussy as a prisoner. (d) पानीपत की लड़ाई


11. वां डीवाश का िस यु जीता था:
(e) उपरो म से कोई नहीं/उपरो म से एक से अिधक
(a) ि िटश
उ र: (b)
(b) च
िवक (b) सही है : ैक होल ासदी: फोट िविलयम को 15 जून 1756 को
(c) पुतगाली
नवाब िसराज-उद-दौला ारा घेर िलया गया था। यु के सामा रीित- रवाजों
(d) डच
के बाद, कलक ा म कुछ अं ेजी मिहलाओं और ब ों को िकले के जेल की
(e) उपरो म से कोई नहीं/उपरो म से एक से अिधक
Topic Name: Modern History
कोठरी म रखा गया था। कैिदयों की सं ा 146 थी और जेल का आकार 18 ब र की लड़ाई के बाद ई इं िडया कंपनी बंगाल की स ी ामी बन गई।

फीट लंबा और 14 फीट 10 इं च चौड़ा बताया गया है । 20 जून को, 146 ेत बंगाल म, रॉबट ाइव ने सरकार की दोहरी णाली की थापना की, यानी दो

कैिदयों को बंद कर िदया गया था, और अगली सुबह जब जेल का कमरा खोला का शासन- कंपनी और नवाब- िजसम कंपनी ने दीवानी, यानी कर सं ह, और

गया, तो केवल 23 लोग ही बचे थे। ई इं िडया के अिधका रयों ने नवाब को िनज़ामत, यानी पुिलस और ाियक काय दोनों को िनयंि त िकया। दीवान के

बदनाम करने के िलए एक चार उपकरण के प म इस करण का इ ेमाल प म, कंपनी ने िड ी सूबेदार को नािमत करने के िलए अपने अिधकार के

िकया और आ ामकता के यु के िलए ि िटश जनता की राय का समथन मा म से दीवानी अिधकारों और िनज़ामत अिधकारों का योग िकया।

िकया। अग 1756 म अं ेजों ारा कलक ा पर क ा करने की खबर म ास बादशाह ने कंपनी को दीवानी काय सौंप,े जबिक बंगाल के सूबेदार ने कंपनी

म िमलने के बाद, ले नट कनल रॉबट ाइव को नवाब के खलाफ जवाबी को िनजामत के काय सौंपे।

कारवाई के िलए भेजा गया था। अपने सैिनकों और थानीय भारतीय सहयोिगयों Q14. The Ba le of Wandiwash (1760), in the history of India, was a
confronta on between the:
के साथ, ाइव ने जनवरी 1757 म कलक ा पर क ा कर िलया और ासी
(a) French and Bri sh
की लड़ाई म िसराज उद-दौला को हरा िदया, िजसके प रणाम प िसराज (b) French and Portuguese

(c) Bri sh and Portuguese


को बंगाल के नवाब के प म उखाड़ फका गया और मार डाला गया।
(d) Bri sh and Local Indian King
Q13. Who among the following introduced the Dual System of
(e) None of the above/More than one of the above
Government in Bengal a er the Ba le of Buxar?
Answer: (a)
(a) Mir Qasim
Op on (a) is correct: Ba le of Wandiwash
(b) Lord Dalhousie
● The Ba le of Wandiwash (22 January 1760), in the history of
(c) Warren Has ngs
India, was a confronta on between the French, under the
(d) Alivardi Khan
Comte de Lally, and the Bri sh, under Sir Eyre Coote.
(e) None of the above/More than one of the above
● It was the decisive ba le in the Anglo-French struggle in
Answer: (e)
southern India during the Seven Years’ War (1756–63).
Op on (e) is correct: The East India Company became the true
● Lally was cut off from receiving any sea support by the
masters of Bengal a er the Ba le of Buxar. In Bengal, Robert Clive
withdrawal of Admiral d’Aché’s fleet. He was also hampered
established the Dual System of Government, i.e., the rule of two—the
by a lack of funds and due to dissensions among his troops.
Company and the Nawab—in which the Company controlled both the
He tried to recover the fort of Wandiwash near Pondicherry
Diwani, i.e., tax collec on, and the nizamat, i.e., police and judicial
(Puducherry) but he was a acked and routed by Coote, with
func ons. As the diwan, the Company exercised Diwani rights and
about 1,700 Bri sh troops against about 2,000 French.
Nizamat rights through its authority to nominate the deputy
● Lally’s best general, the Marquis de Bussy, was captured.
Subahdar. The Emperor gave the Company the Diwani func ons, while
● The French were therea er confined to Pondicherry, which
the Subahdar of Bengal gave the Company the Nizamat func ons.
surrendered on 16 January 1761, a er much priva on.
13. िन िल खत म से िकसने ब र की लड़ाई के बाद बंगाल म सरकार की
● Lally was later imprisoned and executed a er a trial in Paris
दोहरी णाली की शु आत की? for alleged treason. This war ended French supremacy in
India.
(a) मीर कािसम
14. वां डीवाश की लड़ाई (1760), भारत के इितहास म, िकसके बीच टकराव
(b) लॉड डलहौजी
थी:
(c) वारे न हे ं
(a) च और ि िटश
(d) अलीवद खान
(b) च और पुतगाली
(e) उपरो म से कोई नहीं/उपरो म से एक से अिधक
(c) ि िटश और पुतगाली
उ र: (e)
(d) ि िटश और थानीय भारतीय राजा
िवक (e) सही है :
Topic Name: Modern History
(e) उपरो म से कोई नहीं/उपरो म से एक से अिधक Jafar was restored. The Nawab was required to pay Rs 500,000 per
month, but the Company needed more money to fund its ba les,
उ र: (a)
meet trade needs, and cover other costs. It wanted bigger territories
िवक (a) सही है : as well as increased revenue. The Company's a tude had shi ed by
the me Mir Jafar died in 1765. "We must indeed become nawabs
वां डीवाश की लड़ाई
ourselves," Clive said a er failing to cooperate with puppet nawabs.
● वां डीवाश की लड़ाई (22 जनवरी 1760), भारत के इितहास म, कॉ टे डी लैली
15. िन िल खत म से िकस े के संदभ म रॉबट ाइव ने घोषणा की, "हम
के अधीन ां सीिसयों और सर आइरे कूट के अधीन अं ेजों के बीच टकराव
वा वम यं नवाब बनना चािहए।"
था।
(a) अवध
● यह सात साल के यु (1756-63) के दौरान दि ण भारत म एं ो- ां सीसी
(b) है दराबाद
संघष म िनणायक लड़ाई थी।
(c) बंगाल
● एडिमरल डी'चे के बेड़े को हटाकर लैली को कोई भी समु ी समथन ा
(d) म ास
करने से रोक िदया गया था। वह धन की कमी और अपने सैिनकों के बीच
(e) उपरो म से कोई नहीं/उपरो म से एक से अिधक
असंतोष के कारण भी बािधत था। उसने पां िडचेरी (पुदुचेरी) के पास वां डीवाश
उ र: (c)
के िकले को िफर से हािसल करने की कोिशश की, लेिकन लगभग 2,000
िवक (c) सही है : बंगाल के नवाब िसराजु ौला की ासी म हार के बाद ह ा
ां सीसी के खलाफ लगभग 1,700 ि िटश सैिनकों के साथ कोटे ारा उस पर
कर दी गई थी, और मीर जाफर को नवाब िनयु िकया गया था। कंपनी अभी
हमला िकया गया और उसे भगा िदया गया।
भी शासन को अपने हाथ म नहीं लेने पर अड़ी ई थी। ापार का िव ार
● लैली के सबसे अ े जनरल, मा स डे बूसी को पकड़ िलया गया।
इसका मुख ल था। यिद यह जीत के िबना पूरा िकया जा सकता है , तो
● इसके बाद ां सीिसयों को पां िडचेरी तक सीिमत कर िदया गया, िजसने 16
िवशेषािधकार दे ने के इ ु क थानीय शासकों की सहायता से सीधे दे शों को
जनवरी 1761 को ब त किठनाई के बाद आ समपण कर िदया।
लेने की कोई आव कता नहीं होगी। जब मीर जाफर ने आपि की तो कंपनी
● किथत राज ोह के िलए पे रस म मुकदमे के बाद लैली को बाद म जेल म
ने उसे अपद थ कर िदया और उसकी जगह मीर कािसम को िनयु कर
डाल िदया गया और उसे मार िदया गया। इस यु ने भारत म ां सीसी वच
िदया। जब मीर कािसम ने िवरोध िकया, तो वह ब र (1764) की लड़ाई म हार
को समा कर िदया।
गया, बंगाल से िन ािसत कर िदया गया और मीर जाफर को बहाल कर िदया
Q15. In the context of which of the following regions, Robert Clive
गया। नवाब को ित माह 500,000 पये का भुगतान करना था, लेिकन कंपनी
declared, “We must indeed become nawabs ourselves.”?
(a) Awadh को अपनी लड़ाई को िनिध दे ने, ापार की ज रतों को पूरा करने और अ
(b) Hyderabad
लागतों को पूरा करने के िलए अिधक धन की आव कता थी। यह बड़े दे शों
(c) Bengal
(d) Madras के साथ-साथ राज म वृ चाहता था। 1765 म मीर जाफर की मृ ु के समय

(e) None of the above/More than one of the above कंपनी का रवैया बदल गया था। कठपुतली नवाबों के साथ सहयोग करने म
Answer: (c)
िवफल रहने के बाद ाइव ने कहा, "हम वा व म खुद नवाब बनना चािहए।"
Op on (c) is correct: Sirajuddaulah, the nawab of Bengal, was
assassinated a er the defeat at Plassey, and Mir Jafar was appointed
as nawab. The Company was s ll adamant about not taking over the Q16. A er which of the following ba les did the English East India
administra on. The expansion of trade was its primary goal. If this Company get the Diwani rights in Bengal?
could be accomplished without conquering, there would be no need (a) Third Carna c war
to take territories directly, with the aid of local rulers willing to grant (b) Ba le of Plassey
privileges. When Mir Jafar objected, the Company deposed him and (c) Ba le of Buxar
replaced him with Mir Qasim. When Mir Qasim protested, he was (d) Third Anglo Sikh war
defeated in a fight at Buxar (1764), expelled from Bengal, and Mir (e) None of the above/More than one of the above
Topic Name: Modern History
Answer: (c) 16. िसंध के अमीरों के साथ अं ेजों ारा ह ा रत 'शा त िम ता' की संिध
Op on (c) is correct: In the ba le of Buxar, the East India Company
का उ े था:
defeated the combined armies of Mughal emperor Shah Alam II, Mir
Qasim the Nawab of Bengal and Shuja-ud-Daula, the Nawab of (a) िसंध के मा म से मु माग ा करने के िलए

Awadh. This ba le ended with the Treaty of Allahabad, and the East
(b) पंजाब के साथ कुछ सीमा िववादों को हल करना
India Company got the Diwani rights of Bengal from the Mughal
(c) चीन और ि िटश भारत के बीच बाधा उ करने के िलए
Emperor.

16. िन िल खत म से िकस लड़ाई के बाद अं ेजी ई इं िडया कंपनी को (d) अफगानों पर एक ितकूल भाव ा करने के िलए

बंगाल म दीवानी का अिधकार ा आ? (e) उपरो म से कोई नहीं/उपरो म से एक से अिधक

(a) तीसरा कनाटक यु उ र: (e)

(b) ासी की लड़ाई िवक (e) सही है :

(c) ब र की लड़ाई 'शा त िम ता' की संिध- नेपोिलयन बोनापाट जून 1807 म स के अले डर

(d) तीसरा आं िसख यु थम के साथ ितलिसत के गठबंधन म शािमल हो गए। समझौते की

(e) उपरो म से कोई नहीं/उपरो म से एक से अिधक आव कताओं म से एक भारत पर संयु भूिम आ मण था। अं ेज अब स

उ र: (c) और ि िटश भारत के बीच एक अवरोध पैदा करना चाहते थे। ऐसा करने के

िवक (c) सही है : िलए, लॉड िमंटो ने गठबंधन बनाने के िलए तीन ितिनिधमंडल भेजे, िजनम से

ब र की लड़ाई म, ई इं िडया कंपनी ने मुगल स ाट शाह आलम ि तीय, ेक एक िस के नेतृ म था। प रणाम प मेटकाफ को

बंगाल के नवाब मीर कािसम और अवध के नवाब शुजा-उद-दौला की संयु लाहौर, एलिफं न को काबुल और मै म को तेहरान भेजा गया।

सेनाओं को हराया। यह लड़ाई इलाहाबाद की संिध के साथ समा ई और िनकोलस थ ने िसंध का दौरा िकया और एक र ा क समझौते को अंितम

ई इं िडया कंपनी को मुगल स ाट से बंगाल के दीवानी अिधकार िमल गए। प दे ने के िलए अमीरों से मुलाकात की। ब त िवचार-िवमश के बाद, अमीर

एक संिध के िलए सहमत ए, जो अं ेजों के साथ उनकी पहली संिध थी। दोनों
Q16. The aim of the Treaty of ‘Eternal Friendship’ signed by the
प शा त िम ता की शपथ लेने के बाद एक दू सरे के दरबार म ां सीिसयों को
Bri sh with Amirs of Sindh was:
(a) To get Free passage through Sindh िसंध से बाहर रखने और एजटों की अदला-बदली करने पर सहमत ए।

(b) Resolving some border disputes with Punjab


(c) To create a barrier between China and Bri sh India Q17. Who among the following signed the Treaty of Madras on April
(d) To obtain a counterac ng influence over the Afghans 4, 1769, with the Bri sh?
(e) None of the above/More than one of the above (a) Madhavrao
Answer: (e) (b) Tipu Sultan
Op on (e) is correct: Treaty of ‘Eternal Friendship’- Napoleon (c) Haidar Ali
Bonaparte joined Tilsit's alliance with Alexander I of Russia in June (d) Raghunathrao
1807. One of the requirements of the agreement was a united land (e) None of the above/More than one of the above
invasion of India. The Bri sh now wanted to create a barrier between Answer: (c)
Russia and Bri sh India. To do this, Lord Minto sent three delega ons, Op on (c) is correct: The English were confident in their military
each led by a famous figure, to forge alliances. Metcalfe was sent to power a er their easy victory in Bengal. They signed a pact with the
Lahore, Elphinstone to Kabul, and Malcolm to Tehran as a result. Nizam of Hyderabad in 1766, promising to give them the Northern
Nicholas Smith paid a visit to Sindh and met with the Amirs to finalize Circars (region) in exchange for protec ng the Nizam from Haidar Ali.
a defensive agreement. A er much delibera on, the Amirs agreed to Haidar was already embroiled in a territorial conflict with the Nawab
a treaty, which was their first with the English. Both par es agreed to of Arcot and a feud with the Marathas. Against Haidar Ali, the Nizam,
keep the French out of Sindh and swap agents at each other's courts the Marathas, and the English allied together. Haidar acted with a
a er swearing eternal friendship. great deal of delicacy and diplomacy. He bought the Marathas'
Topic Name: Modern History
neutrality by pledging to split acquired lands with the Nizam, and the इस घटना म िक है दर अली पर दू सरी श ारा हमला िकया गया था, अं ेज
Nizam became his ally. A er that, he joined the Nizam in an a ack on
उसकी सहायता करने के िलए तैयार हो गए।
the Nawab.
The first Anglo-Mysore war dragged on for a year and a half with no
end in sight. Haidar shi ed his strategy and appeared in front of Q18. Which of the following was the immediate cause for the
Madras' gates. At Madras, there was full confusion and fear, forcing breakout of the Second Anglo-Mysore War?
the English to sign a humilia ng deal with Haidar Ali on April 4, 1769, (a) Territorial disputes with the Nawab of Arcot
known as the Treaty of Madras. The pact s pulated the exchange of (b) English a empt to capture Mahe
prisoners as well as the reciprocal res tu on of conquests. In the (c) War against Travancore
event that Haidar Ali was a acked by another power, the English (d) Break out of the American War of Independence
agreed to assist him. (e) None of the above/More than one of the above
Answer: (b)
17. िन िल खत म से िकसने 4 अ ैल, 1769 को अं ेजों के साथ म ास की
Op on (b) is correct: Between 1780 and 1784, the Second
संिध पर ह ा र िकए थे? Anglo-Mysore War happened. When Haidar Ali was a acked by the

(a) माधवराव Marathas in 1771, and the English failed to help him, he accused them
of breach of faith and non-observance of the Treaty of Madras. In
(b) टीपू सु ान
addi on, he discovered that the French were far more helpful than
(c) है दर अली the English in supplying his troops with firearms, saltpetre, and lead.
As a result, some French war material was delivered to Mysore via
(d) रघुनाथराव
Mahe, a French territory on the Malabar coast.
(e) उपरो म से कोई नहीं/उपरो म से एक से अिधक
Meanwhile, the American War of Independence had begun, with the
उ र: (c) French siding with the rebels against the English. In these condi ons,
Haidar Ali's affinity with the French alarmed the English even more. As
िवक (c) सही है :
a result, they a empted to seize Mahe, whom Haidar considered to
बंगाल म अपनी आसान जीत के बाद अं ेजों को अपनी सै श पर भरोसा be under his protec on. The English effort to take Mahe was seen by

था। उ ोंने 1766 म है दराबाद के िनज़ाम के साथ एक समझौते पर ह ा र Haidar as a direct threat to his power.
On December 7, 1782, Haidar Ali died of cancer. His son, Tipu Sultan,
िकए, िजसम है दर अली से िनज़ाम की र ा के बदले उ उ री सरकार ( े )
con nued the fight for a year without achieving any success. The
दे ने का वादा िकया। है दर पहले से ही अक ट के नवाब के साथ एक े ीय Treaty of Mangalore (March 1784), which concluded the war,
s pulated that each party return the territory it had gained from the
संघष और मराठों के साथ एक झगड़े म उलझा आ था। है दर अली के खलाफ
other.
िनजाम, मराठों और अं ेजों ने एक साथ गठबंधन िकया। है दर ने बड़ी िवन ता
18. िन िल खत म से कौन सा दू सरा आं -मैसूर यु िछड़ने का
और कूटनीित से काम िलया। उ ोंने िनज़ाम के साथ अिध हीत भूिम को
ता ािलक कारण था?
िवभािजत करने का वचन दे कर मराठों की तट थता खरीदी और िनज़ाम उनके
(a) आक ट के नवाब के साथ े ीय िववाद
सहयोगी बन गए। उसके बाद, वह नवाब पर हमले म िनज़ाम म शािमल हो
(b) माहे पर क ा करने का अं ेजी यास
गया।
(c) ावणकोर के खलाफ यु
पहला आं -मैसूर यु डे ढ़ साल तक चला, िजसका कोई अंत नज़र नहीं
(d) अमे रकी तं ता सं ाम से बाहर हो जाना
आया। है दर ने अपनी रणनीित बदली और म ास के ार के सामने उप थत
(e) उपरो म से कोई नहीं/उपरो म से एक से अिधक
आ। म ास म, पूण म और भय था, िजसने अं ेजों को 4 अ ैल, 1769 को
उ र: (b)
है दर अली के साथ अपमानजनक समझौते पर ह ा र करने के िलए मजबूर
िवक (b) सही है :
िकया, िजसे म ास की संिध के प म जाना जाता है । संिध ने कैिदयों के
1780 और 1784 के बीच, दू सरा एं ो-मैसूर यु आ। 1771 म जब है दर
आदान- दान के साथ-साथ िवजय की पार रक बहाली को िनधा रत िकया।
अली पर मराठों ने हमला िकया और अं ेज़ उनकी मदद करने म नाकाम रहे ,
Topic Name: Modern History
तो उ ोंने उन पर आ था के उ ंघन और म ास की संिध का पालन न करने Lord Canning's government passed the General Service Enlistment Act
in 1856, requiring all future recruits to the Bengal Army to provide an
का आरोप लगाया। इसके अलावा, उ ोंने पाया िक ां सीसी अपने सैिनकों को
assurance to serve wherever the government wanted their services.
आ ेया ों, शोरा और सीसे की आपूित करने म अं ेजों की तुलना म कहीं 19. जब 1824 म भारत म ि िटश सेना के िसपािहयों को कंपनी के िलए लड़ने

अिधक सहायक थे। प रणाम प, मालाबार तट पर एक ां सीसी े माहे के िलए समु ी माग से बमा जाने के िलए कहा गया, तो उ ोंने आदे श का पालन

के मा म से कुछ ां सीसी यु साम ी मैसूर प ं चाई गई। करने से इनकार कर िदया, हालां िक वे भूिम माग से जाने के िलए तैयार हो गए।

इस बीच, अमे रकी तं ता सं ाम शु हो गया था, िजसम ां सीसी अं ेजों इसका कारण आमतौर पर िन िल खत म से िकस कथन से पता लगाया जाता

के खलाफ िव ोिहयों के प म थे। इन प र थितयों म, है दर अली की ां सीसी है ?

के साथ आ ीयता ने अं ेजों को और भी अिधक िचंितत कर िदया। (a) उनम से कई िकसान थे और गां वों म रहने वाले प रवार थे।

प रणाम प, उ ोंने माहे को ज करने का यास िकया, िजसे है दर अपने (b) वे अपने वेतन, भ े और सेवा की शत से नाखुश थे।

संर ण म मानता था। माहे को लेने के अं ेजी यास को है दर ने अपनी श (c) उ डर था िक समु पार करने का मतलब जाित का नुकसान था।

के िलए सीधे खतरे के प म दे खा। (d) (बी) और (सी) इस संदभ म सही ह

7 िदसंबर, 1782 को है दर अली की कसर से मृ ु हो गई। उनके बेटे टीपू (e) उपरो म से कोई नहीं/उपरो म से एक से अिधक

सु ान ने िबना िकसी सफलता के एक साल तक लड़ाई जारी रखी। मगलोर उ र: (e)

की संिध (माच 1784), िजसने यु को समा िकया, ने िनधा रत िकया िक िवक (e) सही है :

ेक प उस े को वापस कर दे गा जो उसने दू सरे से ा िकया था। कंपनी के भारतीय िसपािहयों के असंतोष के अपने कारण थे। वे अपने वेतन,

लाभ और काम करने की थित से असंतु थे। इसके अलावा, कई नए िनयम


Q19. When in 1824, the sepoys of the Bri sh Army in India were told
to go to Burma by the sea route to fight for the Company, they उनकी धािमक मा ताओं और संवेदनाओं के िव गए। समु पार करने का

refused to follow the order, though they agreed to go by the land अथ उस समय के धािमक िहं दुओं के िलए अपनी जाित को खोना था।
route. The reason for this is normally traced to which one of the
नतीजतन, जब िसपािहयों को 1824 म कंपनी के िलए लड़ने के िलए समु के
following asser ons?
(a) Many of them were peasants and had families living in the रा े बमा जाने का आदे श िदया गया, तो उ ोंने आदे श का पालन करने से

villages.
इनकार कर िदया, भले ही उ ोंने जमीन से जाने का वादा िकया था। उ
(b) They were unhappy about their pay, allowances and condi ons
कठोर दं ड िदया गया।
of service.
(c) They had a fear that crossing the seas meant the loss of caste. लॉड कैिनंग की सरकार ने 1856 म जनरल सिवस एनिल मट ए पा रत
(d) (b) and (c) are correct in this context
िकया, िजसम बंगाल सेना म भिव की सभी भितयों की आव कता थी, जहाँ
(e) None of the above/More than one of the above
Answer: (e) सरकार उनकी सेवाएँ चाहती थी, वहाँ सेवा करने का आ ासन दान करना।

Op on (e) is correct: The Company's Indian sepoys had their own


causes for dissa sfac on. They were dissa sfied with their wages, Q20.Consider the following kingdoms:
benefits, and working condi ons. Furthermore, several of the new 1. Satara
rules went against their religious beliefs and sensibili es. Crossing the 2. Sambalpur
seas meant losing one's caste to the religious Hindus of the me. As a 3. Jhansi
result, when the sepoys were ordered to sail to Burma by sea to fight 4. Awadh
for the Company in 1824, they refused to obey the command, even Which of the following is the correct chronological sequence of the
though they promised to go by land. They were subjected to harsh annexa on of the above-given kingdoms under the Doctrine of
punishment. Lapse?
(a) 1– 2 – 4 – 3
(b) 1 – 3 – 2 – 4
Topic Name: Modern History
(c) 2 – 1 – 4 – 3 अं ेजों के अनुसार, नवाब के "कुशासन" से लोगों को मु करने के िलए अवध
(d) 2 – 1 – 3 – 4
पर क ा करने के िलए वे "कत से बा " थे।
(e) None of the above/More than one of the above
Answer: (a) Q21. The final wave of annexa ons occurred under Lord Dalhousie,

Op on (a) is correct: Lord Dalhousie was the Governor-General from who was the Governor-General from 1848 to 1856. In 1856, the East
1848 to 1856. He formulated the Doctrine of Lapse, and the final wave India Company took over Awadh. What was the reason given by the
of annexa ons occurred under this policy of Dalhousie. According to Bri sh for the Annexa on of Awadh?
the doctrine, if an Indian ruler died without a male heir, his kingdom (a) Adopted son of Wajid Ali Shah could not be the heir as per the
would “lapse”, that is, become part of Company territory. Mul ple terms of the Doctrine of Lapse.

kingdoms were annexed by applica on of this doctrine: Satara (1848), (b) Frequent raids by Bhutanese into adjoining territories in Assam
Sambalpur (1850), Udaipur (1852), Nagpur (1853) and Jhansi (1854). could be managed.

Finally, in 1856, the Company also took over Awadh. Annexa on of (c) To free the people of Awadh from the misgovernment of the
Awadh was carried out on the pretext of the “misgovernment” of the Nawab.

Nawab. According to the Bri sh, they were “obliged by duty” to take (d) To increase the revenue collec on of the Bri sh Empire.
over Awadh in order to free the people from the “misgovernment” of (e) None of the above/More than one of the above
the Nawab. Answer: (c)
Op on (c) is correct: Lord Dalhousie was Governor-General from 1848
Q20.िन िल खत सा ा ों पर िवचार कर:
to 1856. Under him, the final annexa ons of the Bri sh occurred. He
1. सतारा formulated the Policy of Doctrine of Lapse. According to the doctrine,

2. संबलपुर if an Indian ruler died without a male heir, then the kingdom would
lapse, which means it would become part of the territory of the
3. झां सी
company. The kingdoms that were annexed were Satara (1848),
4. अवध Sambalpur (1850), Udaipur (1852), Nagpur (1853) and Jhansi (1854).
In 1856, Awadh was taken over. But Awadh was not annexed on the
पगत के िस ां त के तहत ऊपर िदए गए रा ों के िवलय का सही
basis of the Doctrine of Lapse. This me the Bri sh had an added
कालानु िमक म िन िल खत म से कौन सा है ?
argument – Awadh was annexed on the pretext of misgovernance of
(a) 1-2-4-3 the state. The people of Awadh were angry at the manner in which
(b) 1 - 3 - 2 - 4 the state was annexed; hence they joined the revolt of 1857.
(c) 2 - 1 - 4 - 3 21. िवलय की अंितम लहर लॉड डलहौजी के अधीन आई, जो 1848 से 1856
(d) 2 - 1 - 3 - 4
तक गवनर-जनरल थे। 1856 म, ई इं िडया कंपनी ने अवध पर अिधकार कर
(e) उपरो म से कोई नहीं/उपरो म से एक से अिधक
िलया। अवध के िवलय के िलए अं ेजों ने ा कारण बताया था?
उ र: (a)
(a) वािजद अली शाह का द क पु डॉ न ऑफ लै की शत के अनुसार
िवक (a) सही है :
उ रािधकारी नहीं हो सकता था।
लॉड डलहौजी 1848 से 1856 तक गवनर-जनरल थे। उ ोंने पगत का
(b) असम म आस-पास के े ों म भूटािनयों ारा लगातार छापे मारे जा सकते
िस ां त तैयार िकया, और डलहौजी की इस नीित के तहत िवलय की अंितम
ह।
लहर आई। िस ां त के अनुसार, यिद कोई भारतीय शासक िबना पु ष
(c) अवध के लोगों को नवाब के कुशासन से मु कराना।
उ रािधकारी के मर जाता है , तो उसका रा " पगत" हो जाएगा, अथात,
(d) ि िटश सा ा के राज सं ह म वृ करना।
कंपनी के े का िह ा बन जाएगा। इस िस ां त को लागू करके कई रा ों
(e) उपरो म से कोई नहीं/उपरो म से एक से अिधक
को हड़पा गया: सतारा (1848), संबलपुर (1850), उदयपुर (1852), नागपुर
उ र: (c)
(1853) और झां सी (1854)। अंततः 1856 म कंपनी ने अवध पर भी अिधकार
िवक (c) सही है :
कर िलया। नवाब के "कुशासन" के बहाने अवध का िवलय िकया गया था।
Topic Name: Modern History
लॉड डलहौजी 1848 से 1856 तक गवनर-जनरल थे। उनके अधीन, अं ेजों का (a) मैसूर के शासक

अंितम िवलय आ। उ ोंने डॉ न ऑफ लै की नीित तैयार की। िस ां त के (b) तंजौर के शासक

अनुसार, यिद कोई भारतीय शासक िबना पु ष उ रािधकारी के मर जाता है , (c) जोधपुर के राजपूत रा

तो रा समा हो जाएगा, िजसका अथ है िक यह कंपनी के े का िह ा (d) म ास के िनज़ाम

बन जाएगा। िजन रा ों पर क ा िकया गया था वे सतारा (1848), संबलपुर (e) उपरो म से कोई नहीं/उपरो म से एक से अिधक

(1850), उदयपुर (1852), नागपुर (1853) और झां सी (1854) थे। 1856 म अवध उ र: (e)

पर अिधकार कर िलया गया। लेिकन हड़प के िस ां त के आधार पर अवध का िवक (e) सही है :

िवलय नहीं िकया गया था। इस बार अं ेजों के पास एक अित र तक था - लॉड वेलेजली 1798 से 1805 तक गवनर-जनरल थे। सहायक गठबंधन णाली
की मदद से, उ ोंने भारत म सा ा का िव ार करने की कोिशश की। इस
रा के कुशासन के बहाने अवध पर क ा कर िलया गया था। अवध के लोग
णाली के अनुसार, सहयोगी भारतीय रा अपने े म ि िटश सेना की
रा म िवलय के तरीके से नाराज थे; इसिलए वे 1857 के िव ोह म शािमल हो थायी तैनाती के िलए बा थे, और उ अपने रखरखाव के िलए स डी का

गए। भुगतान भी करना पड़ता था। भारतीय शासकों को अपने दरबार म ि िटश

िनवािसयों की शत माननी पड़ी। भारतीय शासकों को गवनर-जनरल से परामश


Q22. With reference to Indian history, who among the following
िकए िबना िकसी अ भारतीय शासक को िनयु करने की भी अनुमित नहीं
rulers first accepted the subsidiary alliance?
(a) Ruler of Mysore थी। इसके बदले म अं ेजों को अपने श ुओं से शासक की र ा करनी थी और

(b) Ruler of Tanjore िम रा ों के आं त रक मामलों म अह ेप की नीित अपनानी थी।


(c) Rajput states of Jodhpur
सहायक णाली को ीकार करने वाले भारतीय शासक है दराबाद के िनज़ाम
(d) Nizam of Madras
(e) None of the above/More than one of the above (िसतंबर 1798 और 1800), मैसूर के शासक (1799), तंजौर के शासक

Answer: (e)
(अ ू बर 1799), अवध के नवाब (नवंबर 1801), पेशवा (िदसंबर) थे। 1801),
Op on (e) is correct: Lord Wellesley was governor-general from
बरार के भोंसले राजा (िदसंबर 1803), िसंिधया (फरवरी 1804), जोधपुर,
1798-to 1805. With the help of a subsidiary alliance system, he tried
to expand the Empire in India. According to this system, the ally Indian जयपुर, माचेरी, बूंदी के राजपूत रा और भरतपुर के शासक (1818)। हो र
state was bound to have permanent sta oning of the Bri sh force in
1818 म सहायक गठबंधन को ीकार करने वाले अंितम मराठा संघ थे।
their territory, and they were also required to pay a subsidy for their
maintenance. Indian rulers had to accept the condi on of Bri sh
Q23. “Treaty of Gandamak” was signed a er:
residents in their court. The Indian rulers were also not allowed to
(a) Second Anglo-Afghan War
employ any other Indian ruler without consul ng the
(b) First Anglo-Afghan War
governor-general. In exchange for this, the Bri sh were to defend the
(c) Third Burma War
ruler from their enemies and adopt a policy of non-interference in the
(d) Second Burma War
internal ma ers of the allied state.
(e) None of the above/More than one of the above
The Indian rulers who accepted the subsidiary system were the Nizam
Answer: (a)
of Hyderabad (September 1798 and 1800), the ruler of Mysore (1799),
Op on (a) is correct: Sher Ali granted permission to keep a Russian
the ruler of Tanjore (October 1799), the Nawab of Awadh (November
envoy in Kabul but refused to keep a Bri sh envoy in Kabul. Owing to
1801), the Peshwa (December 1801), the Bhonsle Raja of Berar
this displeasure, Lord Ly on invaded Afghanistan when Russia
(December 1803), the Sindhia (February 1804), the Rajput states of
withdrew their envoy from Kabul. Sher Ali fled, and the Bri sh signed
Jodhpur, Jaipur, Macheri, Bundi and the ruler of Bharatpur (1818). The
a Treaty of Gandamak (May 1879) with Yakub Khan, the eldest son of
Holkars were the last Maratha confedera on to accept the Subsidiary
Sher Ali.
Alliance in 1818.
Treaty of Gandamak (May 1879):
22. भारतीय इितहास के स भ म िन िल खत म से िकस शासक ने सव थम
● The treaty signed a er the Second-Anglo Afghan War provided
सहायक संिध को ीकार िकया? that:
Topic Name: Modern History
● The Amir conduct his foreign policy with the advice of the (a) To recover Peshawar from the Sikhs.
Government of India (b) To campaign against Maharaja Ranjit Singh’s claims over the
● A permanent Bri sh resident be sta oned in Kabul Afghan territories.
● The Amir got all support against foreign aggression and an annual (c) To protect the boundary of Bri sh India from a probable Russian
subsidy from the government of India. But very soon, Yakub a ack.
abdicated under popular pressure, and the Bri sh recaptured (d) To restore Dost Mohammed as ruler of Afghanistan.
Kabul and Kandahar. Abdur Rehman became the new Amir. (e) None of the above/More than one of the above
Ly on tried to dismember Afghanistan but could not execute it. Answer: (c)
Later, Ripon abandoned this plan and decided to have Op on (c) is correct: The East India Company became concerned
Afghanistan as a buffer state. about Russia in the late 1830s. It was envisioned that Russia might

23. "गंडमक की संिध" के बाद ह ा र िकए गए थे: spread over Asia and invade India from the northwest. Because of this
fear, the Bri sh wanted to maintain their control over the northwest.
(a) दू सरा एं ो-अफगान यु
Between 1838 and 1842, they waged a protracted war with
(b) थम आं -अफगान यु Afghanistan and established indirect Company administra on there.
Lord Auckland, the governor-general of India from 1836 to 1842,
(c) तीसरा बमा यु
advocated for a forward policy. This meant that the Company
(d) दू सरा बमा यु
government in India had to take steps to protect Bri sh India's border
(e) उपरो म से कोई नहीं/उपरो म से एक से अिधक from a possible Russian a ack. This objec ve was to be accomplished
either by trea es with neighbouring countries or by en rely annexing
उ र: (a)
them.
िवक (a) सही है :
24. 1838 और 1842 के बीच अं ेजों ारा अफगािन ान के साथ दीघकालीन
शेर अली ने काबुल म एक सी दू त को रखने की अनुमित दी लेिकन काबुल म
यु लड़ने का मु कारण ा था?
एक ि िटश दू त को रखने से इनकार कर िदया। इस नाराजगी के कारण, लॉड
(a) िसखों से पेशावर को पुन ा करने के िलए।
िलटन ने अफगािन ान पर आ मण िकया जब स ने काबुल से अपना दू त
(b) अफगान े ों पर महाराजा रणजीत िसंह के दावों के खलाफ अिभयान
वापस ले िलया। शेर अली भाग गया, और अं ेजों ने शेर अली के सबसे बड़े बेटे
चलाने के िलए।
याकूब खान के साथ गंडमक (मई 1879) की संिध पर ह ा र िकए।
(c) संभािवत सी हमले से ि िटश भारत की सीमा की र ा के िलए।
गंडामक की संिध (मई 1879):
(d) अफगािन ान के शासक के प म दो मोह द को बहाल करने के
● दू सरे आं -अफगान यु के बाद ह ा रत संिध म ावधान था िक:
िलए।
● अमीर अपनी िवदे श नीित का संचालन भारत सरकार की सलाह से करता है
(e) उपरो म से कोई नहीं/उपरो म से एक से अिधक
● एक थायी ि िटश िनवासी काबुल म तैनात हो
उ र: (c)
● अमीर को िवदे शी आ मण के खलाफ सभी समथन और भारत सरकार से
िवक (c) सही है :
वािषक स डी िमली। लेिकन ब त ज , लोकि य दबाव म याकूब ने ग ी
ई इं िडया कंपनी 1830 के अंत म स के बारे म िचंितत हो गई थी। यह
छोड़ दी और अं ेजों ने काबुल और कंधार पर िफर से क ा कर िलया। अ ु र
क ना की गई थी िक स एिशया म फैल सकता है और उ र पि म से भारत
रहमान नए अमीर बने। िलटन ने अफगािन ान के टु कड़े -टु कड़े करने की
पर आ मण कर सकता है । इसी भय से अं ेज उ र पि म पर अपना
कोिशश की, लेिकन उसे अंजाम नहीं दे सका। बाद म, रपन ने इस योजना को
अिधकार बनाए रखना चाहते थे। 1838 और 1842 के बीच, उ ोंने
ाग िदया और अफगािन ान को एक बफर े ट के प म रखने का फैसला
अफगािन ान के साथ एक लंबी लड़ाई लड़ी और वहां अ कंपनी
िकया।
शासन की थापना की।
Q24. What was the main reason for the Bri sh to fight a prolonged
war with Afghanistan between 1838 and 1842?
Topic Name: Modern History
1836 से 1842 तक भारत के गवनर जनरल लॉड ऑकलड ने आगे की नीित की के िलए यह िनणय िलया गया था। अ मह पूण कारवाइयों म नौकरशाही को

वकालत की। इसका मतलब था िक भारत म कंपनी सरकार को संभािवत सी अपनी पसंद के किमयों के साथ पुनगिठत करना और सेना के कौशल और

हमले से ि िटश भारत की सीमा की र ा के िलए कदम उठाने थे। यह उ े द ता म सुधार के िलए उसे िफर से आकार दे ना शािमल था। मीर कािसम को

या तो पड़ोसी दे शों के साथ संिधयों ारा या उ पूरी तरह से समा करके पूरा कंपनी ने उनके िलए आदश कठपुतली माना था। दू सरी ओर, मीर कािसम ने

िकया जाना था। कंपनी की उ ीदों की अवहे लना की।

Q25. Mir Kasim shi ed the capital from Murshidabad to Munger in


Bihar because: Q26. Who among the following Congress leaders said ‘Our true
(a) He wanted clear informa on on accounts of the revenues of policy is a peaceful policy’ on aggressive imperialism in 1897?
Bihar. (a) C. Sankaran Nair
(b) He wanted to stop the misuse of the East India Company’s (b) Dadabhai Naoroji
dastak. (c) W. C. Bannerji
(c) He wanted a safe distance from the East India Company at (d) Badruddin Tyabji
Calcu a. (e) None of the above/More than one of the above
(d) He wanted to train a be er army for administra ve purposes. Answer: (a)
(e) None of the above/More than one of the above Op on (a) is correct: A er 1878, the Bri sh ini ated a number of
Answer: (c) expansionist opera ons, which were met with s ff resistance by
Op on (c) is correct: Among Alivardi Khan's successors, Mir Kasim was na onalists. The na onalists argued for a policy of peace rather than
the most capable nawab. Mir Kasim moved the capital from aggressive imperialism. "Our true policy is a peaceful policy,"
Murshidabad to Munger in Bihar a er acquiring power. The decision remarked Congress President C. Sankaran Nair in 1897. So, the
was made in order to maintain a safe distance from the Company in emerging themes during 1880-1914 were-
Calcu a. Other significant ac ons included reorganising the ● Solidarity with other colonies figh ng for freedom, such as
bureaucracy with personnel of his choosing and reshaping the army to Russia, Ireland, Egypt, Turkey, Ethiopia, Sudan, Burma and
improve its skill and efficiency. Mir Kasim had been regarded by the Afghanistan.
Company to be the ideal puppet for them. Mir Kasim, on the other ● Condemna on of Burma's annexa on in 1885, inspira on from
hand, defied the Company's expecta ons. Japan as an example of industrial development, condemna on of

25. मीर कािसम ने राजधानी को मुिशदाबाद से िबहार म मुंगेर थानां त रत Japan's par cipa on in the interna onal suppression of the
I-Ho-Tuan uprising (1895), condemna on of imperialist efforts to
िकया ोंिक:
divide China, defeat of Czarist Russia by Japan, which exploded
(a) वह िबहार के राज खातों पर जानकारी चाहता था। the myth of European superiority, and congressional support for
Burma's freedom.
(b) वह ई इं िडया कंपनी के द क के दु पयोग को रोकना चाहता था।
26. िन िल खत कां ेस नेताओं म से िकसने 1897 म आ ामक सा ा वाद
(c) वह कलक ा म ई इं िडया कंपनी से सुरि त दू री चाहता था।
पर 'हमारी स ी नीित शां ितपूण नीित है ' कहा था?
(d) वह शासिनक उ े ों के िलए एक बेहतर सेना को िशि त करना
(a) सी शंकरन नायर
चाहता था।
(b) दादाभाई नौरोजी
(e) उपरो म से कोई नहीं/उपरो म से एक से अिधक
(c) ड ू सी बनज
उ र: (c)
(d) बद ीन तैयबजी
िवक (c) सही है :
(e) उपरो म से कोई नहीं/उपरो म से एक से अिधक
अलीवद खान के उ रािधका रयों म, मीर कािसम सबसे स म नवाब था। स ा
उ र: (a)
हािसल करने के बाद मीर कािसम ने राजधानी को मुिशदाबाद से िबहार के
िवक (a) सही है :
मुंगेर म थानां त रत कर िदया। कलक ा म कंपनी से सुरि त दू री बनाए रखने
Topic Name: Modern History
1878 के बाद, अं ेजों ने कई िव ारवादी अिभयान शु िकए, िजनका (c) भारतीय शासक कंपनी के साथ पूव परामश के िबना िकसी भी यूरोपीय को

रा वािदयों ने कड़ा िवरोध िकया। रा वािदयों ने आ ामक सा ा वाद के अपनी सेवा म िनयु कर सकता था।

बजाय शां ित की नीित का तक िदया। 1897 म कां ेस अ सी. शंकरन नायर (d) भारतीय शासकों को अपनी तं सश सेना रखने की अनुमित थी।

ने कहा, "हमारी स ी नीित एक शां ितपूण नीित है ।" इसिलए, 1880-1914 के (e) उपरो म से कोई नहीं/उपरो म से एक से अिधक

दौरान उभरते ए िवषय थे- उ र: (e)

● स, आयरलड, िम , तुक , इिथयोिपया, सूडान, बमा और अफगािन ान िवक (e) सही है :

जैसे तं ता के िलए लड़ रहे अ उपिनवेशों के साथ एकजुटता। लॉड वेले ी, िज ोंने 1798 से 1805 तक भारत के गवनर-जनरल के पम

● 1885 म बमा के िवलय की िनंदा, औ ोिगक िवकास के उदाहरण के प म काय िकया, ने भारत म एक सा ा थािपत करने के िलए सहायक गठबंधन

जापान से ेरणा, आई-हो-तुआन िव ोह (1895) के अंतरा ीय दमन म जापान णाली का उपयोग िकया। एक सहयोगी भारतीय रा के शासक को अपने

की भागीदारी की िनंदा, चीन को िवभािजत करने के सा ा वादी यासों की े के भीतर एक ि िटश सेना की थायी तैनाती को ीकार करने और

िनंदा, जारशाही स की हार जापान ारा, िजसने यूरोपीय े ता के िमथक व था के तहत इसके रखरखाव के िलए वजीफा दे ने के िलए मजबूर होना

और बमा की तं ता के िलए कां ेस के समथन का खंडन िकया। पड़ा। यिद भारतीय शासकों ने भुगतान नहीं िकया, तो उनके दे श का एक

Q27. Which one of the following statements is correct with reference िह ा सजा के प म ले िलया गया। उदाहरण के िलए, अवध के नवाब को
to the Subsidiary Alliance of Lord Wellesley?
1801 म अपना आधा े कंपनी को सौंपने के िलए बा िकया गया था, जब
(a) Indian rulers were to be protected by the East India Company
without any payment. रचड वेले ी गवनर-जनरल (1798-1805) थे ोंिक वह "सहायक सैिनकों"

(b) The territory of Indian rulers was not taken away in any case.
के िलए भुगतान करने म िवफल रहे थे। इसी तरह के आधार पर, है दराबाद े
(c) The Indian ruler could employ any European in his service
आ समपण करने के िलए बा था।
without prior consulta on with the Company.
(d) Indian rulers were allowed to have their independent armed
forces. Q28. In the 4th Anglo – Mysore war, Tipu Sultan was defeated by

(e) None of the above/More than one of the above Governor-General:

Answer: (e) (a) Sir Hughe Rose

Op on (e) is correct: Lord Wellesley, who served as Governor-General (b) John Shore
of India from 1798 to 1805, used the Subsidiary Alliance system to (c) Wellesley
establish an empire in India. The ruler of an allying Indian state was (d) Thomas Munroe
forced to accept the permanent sta oning of a Bri sh force within his (e) None of the above/More than one of the above
territory and pay a s pend for its maintenance under the system. If Answer: (c)
the Indian rulers did not pay, a por on of their country was taken Op on (c) is correct: In 1798, Lord Wellesley became the new
away as a punishment. For example, the Nawab of Awadh was obliged Governor-General. An imperialist to the core, Wellesley was
to hand over half of his territory to the Company in 1801, when concerned about Tipu’s growing friendship with the French and aimed
Richard Wellesley was Governor-General (1798-1805) since he failed at annihila ng Tipu’s independent existence or forcing him to
to pay for the "subsidiary soldiers." On similar grounds, Hyderabad submission through the system of Subsidiary Alliance.
was obliged to surrender territory. The war began on April 17, 1799, and ended on May 4, 1799, with the

27. लॉड वेले ी के सहायक गठबंधन के संदभ म िन िल खत म से कौन सा fall of Seringapatam. Tipu was defeated first by English General Stuart
and then by General Harris.
कथन सही है ?
28. चौथे आं -मैसूर यु म, टीपू सु ान को गवनर-जनरल ारा परािजत
(a) भारतीय शासकों को िबना िकसी भुगतान के ई इं िडया कंपनी ारा
िकया गया था:
संरि त िकया जाना था।
(a) सर ह्यूज रोज
(b) िकसी भी सूरत म भारतीय शासकों का े नहीं छीना गया।
(b) जॉन शोर
Topic Name: Modern History
(c) वेले ी थे। जून 1838 म यह ि प ीय संिध मूल प से एक दू सरे की ज रत के समय

(d) थॉमस मुनरो म मदद करने के िलए थी, लेिकन इसने थम आं -अफगान यु को ज

(e) उपरो म से कोई नहीं/उपरो म से एक से अिधक िदया।

उ र: (c)
Q30. Under the ryo system of Indigo cul va on in Bengal, the ryots
िवक (c) सही है :
who signed the contract:
1798 म, लॉड वेले ी नए गवनर-जनरल बने। मूल प से एक सा ा वादी, (a) Did not get cash advances from the planters

(b) Got cash advances from the planters at very high rates of interest
वेले ी टीपू की ां सीसी के साथ बढ़ती दो ी के बारे म िचंितत था और इसका
(c) Did not receive seed and the drill from the planters
उ े टीपू के तं अ को समा करना था या उसे सहायक गठबंधन
(d) Got cash advances from the planters at High rates of interest
की णाली के मा म से ुत करने के िलए मजबूर करना था। (e) None of the above/More than one of the above
Answer: (e)
यु 17 अ ैल, 1799 को शु आ और 4 मई, 1799 को ीरं गपटम के पतन
Op on (e) is correct: The planters required the ryots to sign a
के साथ समा आ। टीपू को पहले अं ेज जनरल ु अट और िफर जनरल contract, an agreement, under the ryo system of Indigo farming in

है रस ने हराया था। Bengal (sa a). They pressed the village chiefs to sign the contract on
behalf of the ryots at mes. Those who accepted the contract
received low-interest monetary advances from the planters in order to
Q29. The tripar te treaty of 1838 was entered into between:
produce indigo. However, the ryot was required to cul vate indigo on
(a) Bri sh, Sindh and Afghans
at least 25% of his land as part of the loan. The cul vators prepared
(b) Sikhs, Sindh and Bri sh
the soil, planted the seed, and cared for the crop, while the planter
(c) Sikhs, Bri sh and Afghans
provided the seed and the drill.
(d) Sikhs, Bri sh and Afghans
(e) None of the above/More than one of the above 30. बंगाल म नील की खेती की रै यती णाली के अंतगत िजन रै यतों ने

Answer: (e) अनुबंध पर ह ा र िकए:


Op on (e) is correct: A tripar te treaty was signed between Ranjit
(a) ) बागान मािलकों से नकद अि म नहीं िमला
Singh and Shah Shuja ((who had been deposed from the Afghan
throne in 1809 and had been living since then as a Bri sh pensioner at (b) बागान मािलकों से ब त अिधक ाज दरों पर नकद अि म ा िकया

Ludhiana).) and Lord Auckland (Bri sh) in June 1838. This tripar te (c) ) बागान मािलकों से बीज और िडल नहीं िमला
treaty was basically to help each other in their me of need, but it led
(d) बागान मािलकों से ाज की उ दरों पर नकद अि म ा िकया
to the First Anglo-Afghan War.

29. 1838 की ि प ीय संिध के बीच म वेश िकया गया था: (e) उपरो म से कोई नहीं/उपरो म से एक से अिधक

(a) ि िटश, िसंध और अफगान उ र: (e)

(b) िसख, िसंध और ि िटश िवक (e) सही है :

(c) िसख, ि िटश और अफगान बागवानों को बंगाल (स ा) म इं िडगो खेती की रै यत णाली के तहत एक

(d) िसख, ि िटश और अफगान अनुबंध, एक समझौते पर ह ा र करने के िलए रै यतों की आव कता थी।

(e) उपरो म से कोई नहीं/उपरो म से एक से अिधक उ ोंने कई बार ाम धानों पर रै यतों की ओर से अनुबंध पर ह ा र करने के

उ र: (e) िलए दबाव डाला। िजन लोगों ने अनुबंध ीकार िकया, उ नील का उ ादन

िवक (e) सही है : करने के िलए बागान मािलकों से कम ाज पर मौि क अि म ा आ।

रणजीत िसंह और शाह शुजा ((िज 1809 म अफगान िसंहासन से हटा िदया हालां िक, ऋण के िह े के प म रै यत को अपनी भूिम के कम से कम 25%

गया था और तब से लुिधयाना म एक ि िटश पशनभोगी के प म रह रहे थे) पर नील की खेती करने की आव कता थी। का कारों ने िम ी तैयार की,

और लॉड ऑकलड (ि िटश) के बीच एक ि प ीय संिध पर ह ा र िकए गए


Topic Name: Modern History
बीज बोए और फसल की दे खभाल की, जबिक बोने वाले ने बीज और िडल करने की आव कता थी। का कारों ने िम ी तैयार की, बीज बोए और फसल

दान िकया। की दे खभाल की, जबिक बोने वाले ने बीज और िडल दान िकया। कटाई के

बाद, फसल को बोने वाले को थानां त रत कर िदया गया, रै यत को एक नया


Q31. Under the ryo system of Indigo cul va on in Bengal, which
ऋण िदया गया, और च िफर से शु हो गया। ऋणों ने मूल प से िकसानों
one of the following is not a problem faced by ryots?
(a) Requirement of a large area for cul va on को लुभाया, लेिकन उ ज ी ही एहसास आ िक व था िकतनी कठोर थी।

(b) Limited scope for cul va on of rice


उ उनके ारा उ ािदत इं िडगो के िलए थोड़ा भुगतान िकया गया था, और
(c) Forced contracts
उधार लेने का च अिनि त काल तक चलता रहा।
(d) Never-ending cycle of the loan
(e) None of the above/More than one of the above
Answer: (a) Q32. In Bri sh revenue records, ‘mahal’ is a:

Op on (a) is correct: The planters forced the ryots to sign a contract, (a) Revenue estate from which the company collected revenue
an agreement, under the ryo system (sa a). They pressed the village directly from cul vators.

chiefs to sign the contract on behalf of the ryots at mes. (b) Revenue estate, which is under control of Zamindars.

Those who accepted the contract received low-interest monetary (c) Revenue estate, which must have 84 villages.
advances from the planters in order to produce indigo. However, the (d) Revenue estate, which may be a village or a group of villages.
ryot was required to cul vate indigo on at least 25% of his land as part (e) None of the above/More than one of the above
of the loan. The cul vators prepared the soil, planted the seed, and Answer: (d)
cared for the crop, while the planter provided the seed and the drill. Op on (d) is correct: An Englishman named Holt Mackenzie created
A er the harvest, the crop was transferred to the planter, a fresh loan the new mahalwari se lement system in the Bengal Presidency's
was granted to the ryot, and the cycle began all over again. The loans North-Western Provinces (much of this area is now in U ar Pradesh),
originally en ced peasants, but they quickly realised how harsh the which went into force in 1822.
system was. They were paid a li le for the indigo they produced, and Mahal is a revenue estate in Bri sh revenue records, which might be a
the cycle of borrowing con nued indefinitely. village or a group of villages. Revenue demand under Mahalwari

31. बंगाल म नील की खेती की रै यती णाली के अंतगत रै यतों को se lement was to be reviewed on a regular basis, it was not
permanent. The village headman, not the zamindar, was in charge of
िन िल खत म से कौन-सी सम ा का सामना नहीं करना पड़ता है ?
collec ng the revenue and paying it to the Company.
(a) खेती के िलए एक बड़े े की आव कता 32. ि िटश राज अिभलेखों म, 'महल' एक है :

(b) चावल की खेती के िलए सीिमत गुंजाइश (a) राज स दा िजससे कंपनी सीधे िकसानों से राज एक करती थी।

(c) मजबूर अनुबंध (b) राज संपि , जो जमींदारों के िनयं ण म है ।

(d) ऋण का कभी न ख होने वाला च (c) राज संपि , िजसम 84 गां व होना चािहए।

(e) उपरो म से कोई नहीं/उपरो म से एक से अिधक (d) राज संपि , जो एक गां व या गां वों का समूह हो सकता है ।

उ र: (a) (e) उपरो म से कोई नहीं/उपरो म से एक से अिधक

िवक (a) सही है : उ र: (d)

बागवानों ने रै यतों को रै यत णाली (स ा) के तहत एक अनुबंध, एक समझौते िवक (d) सही है :

पर ह ा र करने के िलए मजबूर िकया। उ ोंने कई बार ाम धानों पर हो मैकज़ी नाम के एक अं ेज ने बंगाल ेसीडसी के उ र-पि मी ां तों (इस

रै यतों की ओर से अनुबंध पर ह ा र करने के िलए दबाव डाला। े का अिधकां श िह ा अब उ र दे श म है ) म नई महालवारी बंदोब

िजन लोगों ने अनुबंध ीकार िकया, उ नील का उ ादन करने के िलए णाली बनाई, जो 1822 म लागू ई।

बागान मािलकों से कम ाज पर मौि क अि म ा आ। हालां िक, ऋण के ि िटश राज अिभलेखों म महल एक राज स दा है , जो एक गाँ व या गाँ वों

िह े के प म रै यत को अपनी भूिम के कम से कम 25% पर नील की खेती का समूह हो सकता है । महालवाड़ी ब ोब के अ गत राज माँ ग की
Topic Name: Modern History
िनयिमत आधार पर समी ा की जानी थी, यह थायी नहीं थी। ाम धान, संघष िकया। बागान मािलकों के कमचा रयों को सामािजक प से बिह ृ त

जमींदार नहीं, राज एक करने और कंपनी को भुगतान करने का भारी था। कर िदया गया और लगान वसूल करने आए गुमा ों - बागवानों के ितिनिधयों

Q33. Which one of the following statements appropriately describes - को बुरी तरह पीटा गया। रै यतों ने ित ा की िक वे अब नील की बुवाई की
the term "Gomasthas"?
पेशगी को कभी ीकार नहीं करगे या बागान मािलकों के लिठयालों - बागान
(a) The lathi-wielding strongmen maintained by the planters of
Indigo in Bengal मािलकों के लाठी चलाने वाले बलवानों ारा मजबूर नहीं होंगे।

(b) Agents of planters who came to collect rent from the ryots who Q34. A er the demarca on of “Damin-i-Koh”, Santhal se lements
grew Indigo in Bengal expanded rapidly. What is “Damin-i-Koh”?
(c) A unit of measurement of land under Indigo cul va on in Bengal (a) It is the land given to Santhals in the foothills of Rajmahal.
(d) Cul vators who had lled the land for genera ons in South India (b) It is the land given to Santhals for the cul va on of commercial
(e) None of the above/More than one of the above crops
Answer: (b) (c) It is the land controlled by zamindars in Bengal
Op on (b) is correct: Gomasthas were the planter's agents who (d) statements (b), and (c) given above is correct in this context
travelled to Bengal to collect rent from the ryots who grew Indigo. (e) None of the above/More than one of the above
Thousands of ryots in Bengal refused to plant indigo in March 1859. As Answer: (a)
the insurrec on grew, ryots refused to pay planter rents and assaulted Op on (a) is correct: The Santhals were given land and convinced to
indigo factories with swords, spears, and bows and arrows. Women relocate to Rajmahal's slopes. Damin-i-Koh was established in 1832
fought with pots, pans, and other culinary tools. Planters' employees and covered a wide region of territory. This was designated as the
were socially ostracised, and gomasthas – planters' representa ves – Santhals' homeland. They were supposed to live in it, cul vate the
who came to collect rent were beaten severely. Ryots pledged they land, and se le down as peasants.
would never longer accept indigo sowing advances or be coerced by The Santhals' land grant required them to clear and develop at least
the planters' lathiyals – the planters' lathi-wielding strongmen. one-tenth of the land during the first ten years. The area was plo ed
33. िन िल खत म से कौन सा कथन "गोमा " श का उिचत वणन करता and surveyed. It was isolated from both the world of the established
agriculturists of the plains and the Paharias of the hills by boundary
है ?
pillars.
(a) बंगाल म इं िडगो के ां टस ारा बनाए गए लाठी चलाने वाले मजबूत Santhal se lements grew swi ly a er the delinea on of Damin-i-Koh.

(b) बागान मािलकों के एजट जो बंगाल म इं िडगो उगाने वाले रै यतों से िकराया By 1851, the territory had grown from 40 Santhal se lements in 1838
to 1,473 villages. During this me, the Santhal popula on grew from
लेने आए थे
3,000 to nearly 82,000 people. Revenue went into the Company's
(c) बंगाल म नील की खेती के अंतगत भूिम की माप की एक इकाई coffers at a faster rate as cul va on grew.

(d) िकसान िज ोंने दि ण भारत म पीिढ़यों से भूिम की जुताई की थी 34. दािमन-ए-कोह के सीमां कन के बाद संथाल ब यों का तेजी से िव ार

(e) उपरो म से कोई नहीं/उपरो म से एक से अिधक आ। "दािमन-ए-कोह" ा है ?

उ र: (b) (a) यह राजमहल की तलहटी म संथालों को दी गई भूिम है ।

िवक (b) सही है : (b) यह संथालों को वािण क फसलों की खेती के िलए दी गई भूिम है

गुमा बागान मािलक के एजट थे जो इं िडगो उगाने वाले रै यतों से िकराया लेने (c) यह बंगाल म जमींदारों ारा िनयंि त भूिम है

के िलए बंगाल गए थे। (d) उपरो कथन (b), और (c) इस संदभ म सही ह

माच 1859 म बंगाल म हजारों रै यतों ने इं िडगो लगाने से इनकार कर िदया। (e) उपरो म से कोई नहीं/उपरो म से एक से अिधक

जैसे-जैसे िव ोह बढ़ता गया, रै यतों ने बागान के िकराए का भुगतान करने से उ र: (a)

इनकार कर िदया और तलवारों, भालों और धनुष-बाणों से इं िडगो कारखानों पर िवक (a) सही है :

हमला िकया। मिहलाओं ने बतन, धूपदान और अ पाक उपकरणों के साथ संथालों को जमीन दी गई थी और उ राजमहल की ढलानों पर थानां त रत

करने के िलए राजी िकया गया था। दािमन-ए-कोह की थापना 1832 म ई थी


Topic Name: Modern History
और इसम एक िव ृत े शािमल था। इसे संथालों की मातृभूिम के प म उ र: (e)

नािमत िकया गया था। उनसे अपे ा की गई थी िक वे उसम रह, भूिम पर खेती िवक (e) सही है :

कर और कृषकों के प म बस जाएँ । भारत म कपास कारखाने के उ ादन के िवकास म पहला बड़ा उछाल थम

संथालों के भूिम अनुदान के िलए उ पहले दस वष के दौरान भूिम के कम से िव यु के दौरान आ था। ि टे न से आयात म िगरावट आई और इसिलए

कम दसव िह े को साफ़ करने और िवकिसत करने की आव कता थी। े भारतीय कारखानों ने अपनी उ ादन मा ा म वृ की और सै आपूित के

का मानिच ण एवं सव ण िकया गया। यह थािपत की दोनों दु िनया से अलग िलए कपड़े का उ ादन िकया।

था ारं िभक वष म भारतीय कपड़ा उ ोग के सामने आने वाली सम ाएँ :

दािमन-ए-कोह के िच ण के बाद संथाल ब यों का तेजी से िवकास आ। अपनी नींव के पहले कुछ दशकों के दौरान, भारतीय कपड़ा उ ोग को गंभीर

1851 तक, े 1838 म 40 संथाल ब यों से बढ़कर 1,473 गाँ व हो गया था। सम ाओं का सामना करना पड़ा। भारतीय व ों के िलए ि टे न से आयाितत

इस समय के दौरान, संथाल की आबादी 3,000 से बढ़कर लगभग 82,000 हो स ेव ों से ित धा करना ब त किठन था। अिधकां श दे शों की सरकारों ने

गई। जैसे-जैसे खेती बढ़ी, राज कंपनी के खजाने म तेजी से गया। आयात पर भारी शु लगाया और थानीय उ ोगों को फलने-फूलने म मदद

की। औपिनवेिशक सरकार ने थानीय उ ोगों को िकसी भी कार के संर ण


Q35. The reason behind the growth of the tex le industry in India
से इं कार कर िदया था।
during the First World War was:
(a) Increase in the import of tex le from Britain
(b) Growth in the number of Indian owned factories Q36. A er the destruc on of the handloom industry, handloom

(c) Produc on by Indian factories for military supplies weavers found work in the new co on mills. By 1900, co on mills in

(d) Rapid decrease in the urban popula on India were set up at:

(e) None of the above/More than one of the above (a) Bombay, Kanpur and Ahmedabad

Answer: (e) (b) Madras, Pune and Bombay

Op on (e) is correct: The first major spurt in the development of (c) Banaras, Mathura and Lucknow
co on factory produc on in India was during the First World War. (d) Amritsar, Bombay and Pune
There was a decline in imports from Britain, and therefore Indian (e) None of the above/More than one of the above
factories enhanced their produc on quan ty and produced clothes Answer: (a)
for military supplies. Op on (a) is correct: In Bombay, the first co on mill was set up as a

The problem faced by the Indian tex le industry in its early years: spinning mill in 1854. Bombay was close to the vast black soil tract of

During the first few decades of its founda on, the Indian tex le the western part of India, where co on was grown in abundance;
industry had to face severe problems. For Indian tex les, compe ng hence Bombay evolved as an important port that exported raw
with cheap tex les imported from Britain was very difficult. Most materials to England and China. The co on tex les were established
countries' governments imposed heavy du es on imports and helped in Bombay; hence there was no shortage of supplies of raw materials.
local industries to flourish. The colonial government had refused any Around 1900, over 84 mills were running in Bombay. A lot of these
kind of protec on to local industries. mills were established by Parsi and Gujara businessmen. These
businessmen accumulated their capital through trade with China.
35. थम िव यु के दौरान भारत म कपड़ा उ ोग के िवकास का कारण
Mills were established in other ci es too. The first mill in Ahmedabad
था: was established in 1861, followed by the mill established in Kanpur,

(a) ि टे न से कपड़े के आयात म वृ United Province.

36. हथकरघा उ ोग के िवनाश के बाद हथकरघा बुनकरों को नई सूती िमलों


(b) भारतीय ािम वाले कारखानों की सं ा म वृ
म काम िमला। 1900 तक, भारत म सूती िमल थािपत की गईं:
(c) सै आपूित के िलए भारतीय कारखानों ारा उ ादन
(a) बॉ े, कानपुर और अहमदाबाद
(d) शहरी आबादी म तेजी से कमी
(b) म ास, पुणे और बॉ े
(e) उपरो म से कोई नहीं/उपरो म से एक से अिधक
Topic Name: Modern History
(c) बनारस, मथुरा और लखनऊ headman was given the responsibility of collec ng the revenue and
paying it to the company. Zamindar was not considered in this system.
(d) अमृतसर, बॉ े और पुणे
37. हा मैकज़ी ारा तैयार की गई महालवाड़ी बंदोब ी म:
(e) उपरो म से कोई नहीं/उपरो म से एक से अिधक (a) राज की मां ग थायी प से तय की गई थी।
(b) राज की मां ग को समय-समय पर संशोिधत िकया जाना था, थायी प
उ र: (a)
से तय नहीं िकया गया था।
िवक (a) सही है :
(c) राज की मां ग को या तो समय-समय पर संशोिधत िकया जाना था या
बंबई म, पहली कपास िमल 1854 म कताई िमल के प म थािपत की गई थायी प से तय िकया जाना था।
(d) राज की मां ग को न तो समय-समय पर संशोिधत िकया जाना था और न
थी। बॉ े भारत के पि मी िह े के िवशाल काली िम ी के इलाके के करीब
ही थायी प से तय िकया जाना था।
था, जहां कपास ब तायत म उगाई जाती थी; इसिलए बंबई एक मह पूण (e) उपरो म से कोई नहीं/उपरो म से एक से अिधक

बंदरगाह के प म िवकिसत आ जो इं ड और चीन को क े माल का उ र: (b)


िवक (b) सही है :
िनयात करता था। सूती व बंबई म थािपत िकए गए थे; इसिलए क े माल
एक अं ेज, हो मैकज़ी ने एक नई णाली तैयार की, जो 1822 म लागू ई।
की आपूित म कोई कमी नहीं थी।
इसे महलवारी बंदोब कहा जाता था। यह बंगाल ेसीडसी के उ र-पि मी
1900 के आसपास, बॉ े म 84 से अिधक िमल चल रही थीं। इनम से कई िमलों
ां तों म चालू हो गया (इस े का अिधकां श िह ा अब उ र दे श म है )
की थापना पारसी और गुजराती ापा रयों ने की थी। इन ापा रयों ने चीन के
उनके अनुसार, उ र भारतीय समाज म, गाँ व एक मह पूण सामािजक सं था
साथ ापार करके अपनी पूंजी जमा की। अ शहरों म भी िमल थािपत की
थी और इसिलए उसे संरि त रखा जाना था। उनकी दे खरे ख म कले र
गईं। अहमदाबाद म पहली िमल की थापना 1861 म ई थी, उसके बाद
गाँ व-गाँ व घूमे, भूिम का िनरी ण िकया और खेत की नाप-तौल की तथा िविभ
संयु ां त के कानपुर म िमल की थापना ई।
समूहों के रीित- रवाजों और अिधकारों पर भी िवचार िकया। ेक गां व (महल)

Q37. In the Mahalwari se lement devised by Holt Mackenzie: को भुगतान करने के िलए राज की गणना करने के िलए ेक भूखंड का

(a) Demand for revenue was permanently fixed. अनुमािनत राज जोड़ा गया था। राज िनि त नहीं था और समय-समय पर
(b) Demand for revenue was to be revised periodically, not
संशोिधत िकया जाना था। गाँ व के मु खया को राज एक करने और कंपनी
permanently fixed.
(c) Demand for revenue was either to be revised periodically or को भुगतान करने की िज ेदारी दी गई थी। इस व था म जमींदार को नहीं

permanently fixed. माना जाता था|


(d) Demand for revenue was neither to be revised periodically nor
permanently fixed.
Q38. In the context of Indian Economic History, which of the
(e) None of the above/More than one of the above
following statements is not correct about the “Drain of Wealth”
Answer: (b)
theory?
Op on (b) is correct: Holt Mackenzie, an Englishmen, devised a new
(a) The Drain of Wealth theory was systemically ini ated by
system, which came into effect in 1822. It was called the Mahalwari
Dadabhai Naoroji.
se lement. It became opera onal in the North-Western Provinces of
(b) Through this theory, Naoroji sought to prove that Britain was
the Bengal Presidency (most of this area is now in U ar Pradesh)
draining money out of India.
According to him, in North Indian society, the village was an important
(c) The drain of wealth helped in capital forma on in the Indian
social ins tu on and therefore had to be preserved. Under his
Economy.
supervision, the collectors travelled across villages, inspected the land,
(d) The drain theory was put forward in the book Poverty and
and measured the field, and the custom and rights of different groups
Un-Bri sh Rule in India.
were also considered. The es mated revenue of each plot was added
(e) None of the above/More than one of the above
up to calculate the revenue each village (Mahal) had to pay. The
Answer: (c)
revenue was not fixed and was to be periodically revised. The village
Op on (c) is correct: The phrase 'economic drain' refers to a
percentage of India's na onal product that is not available for
Topic Name: Modern History
consump on by its people but is instead drained away to Britain for भारत म पूंजी िनमाण को धीमा कर िदया गया, जबिक धन के उसी िह े ने
poli cal reasons, with India receiving insufficient economic or material
ि िटश अथ व था के िवकास को गित दी। ि िटश अथ व था से अिधशेष ने
returns. Dadabhai Naoroji put forward the drain theory in his book
Poverty and Un-Bri sh Rule in India. Salaries and pensions of civil and िव पूंजी के प म भारत म िफर से वेश िकया, िजससे भारत की संपि और

military officials, interest on loans taken out by the Indian


भी अिधक िनकल गई। इसका भारत के राज और रोजगार की संभावनाओं
Government from abroad, profits from foreign investment in India,
पर भारी भाव पड़ा|
stores purchased in Britain for civil and military departments,
payments to be made for shipping, banking, and insurance services, Q39. Which of the following statements correctly explains the term
all of which stunted the growth of Indian enterprise in these areas, ‘Investments’ in terms of the trade of the East India Company in the
were the major components of this drain. The drain of wealth checked 18th century in Modern Indian History?
and retarded capital forma on in India, while the same por on of (a) It was the money invested in land produc vity by the East India
wealth accelerated the growth of the Bri sh economy. The surplus Company in India.

from the Bri sh economy re-entered India as finance capital, draining (b) It was the money invested by the King of England with the
India's wealth even more. This had a huge impact on India's revenue approval of the Parliament of England.

and employment prospects. (c) It was the money invested by the Indian Kings in the Indian

38. भारतीय आिथक इितहास के स भ म, िन िल खत म से कौन सा ventures of the East India Company.
(d) It was the purchases of Indian goods to export by the Company
कथन "धन की िनकासी" िस ां त के बारे म सही नहीं है ?
out of the revenue of Bengal.
(a) दादाभाई नौरोजी ारा व थत प से डे न ऑफ वे ोरी शु की गई (e) None of the above/More than one of the above
Answer: (d)
थी।
Op on (d) is correct: In 1765, the Company acquired the Diwani of
(b) इस िस ां त के मा म से, नौरोजी ने यह सािबत करने की कोिशश की िक Bengal and thus gained control over its revenues. The Company, even

ि टे न भारत से पैसे िनकाल रहा था। more than its servants, soon directly organised the Drain of wealth. It
began to purchase Indian goods out of the revenue of Bengal and
(c) धन की िनकासी ने भारतीय अथ व था म पूंजी िनमाण म मदद की।
export them. These purchases were known as ‘Investments.’
(d) डे न ोरी को पॉवट एं ड अन-ि िटश ल इन इं िडया नामक पु क म Thus, through ‘Investments’, the Bengal Revenue was sent to England.

By the end of the 18th century, the Drain cons tuted nearly 9% of
ुत िकया गया था।
India’s na onal income. The actual Drain was even more, as a large
(e) उपरो म से कोई नहीं/उपरो म से एक से अिधक
part of the salaries and other incomes of English Officials and the
उ र: (c) trading fortunes of English Merchants also found their way into
England.
िवक (c) सही है :
39. िन िल खत म से कौन सा कथन आधुिनक भारतीय इितहास म 18वीं
मुहावरा 'आिथक िनकासी' भारत के रा ीय उ ाद के ितशत को संदिभत
शता ीमई इं िडया कंपनी के ापार के संदभ म 'िनवेश' श की सही
करता है जो इसके लोगों ारा उपभोग के िलए उपल नहीं है , ब भारत
ा ा करता है ?
को अपया आिथक या भौितक रटन ा करने के साथ, राजनीितक कारणों
(a) यह भारत म ई इं िडया कंपनी ारा भूिम उ ादकता म िनवेश िकया गया
से ि टे न को भेज िदया जाता है । दादाभाई नौरोजी ने अपनी पु क पॉवट एं ड
धन था।
अन-ि िटश ल इन इं िडया म पलायन िस ां त को ितपािदत िकया। नाग रक
(b) यह इं ड के राजा ारा इं ड की संसद के अनुमोदन के साथ िनवेश
और सै अिधका रयों के वेतन और पशन, भारत सरकार ारा िवदे शों से िलए
िकया गया धन था।
गए ऋण पर ाज, भारत म िवदे शी िनवेश से लाभ, नाग रक और सै िवभागों
(c) यह ई इं िडया कंपनी के भारतीय उ मों म भारतीय राजाओं ारा िनवेश
के िलए ि टे न म खरीदे गए ोर, िशिपंग, बिकंग और बीमा सेवाओं के िलए
िकया गया धन था।
िकए जाने वाले भुगतान इन सभी ने इन े ों म भारतीय उ म के िवकास को
(d) यह बंगाल के राज से कंपनी ारा िनयात करने के िलए भारतीय व ु ओं
रोक िदया, जो इस नाले के मुख घटक थे। धन के िनकास को रोका गया और
की खरीद थी।
Topic Name: Modern History
(e) उपरो म से कोई नहीं/उपरो म से एक से अिधक उ र: (e)

उ र: (d) िवक (e) सही है :

िवक (d) सही है : 1859 म बंगाल म " ू िव ोह" के बाद, इं िडगो उ ादन णाली की जां च के

1765 म, कंपनी ने बंगाल की दीवानी का अिध हण िकया और इस कार िलए एक इं िडगो आयोग का गठन िकया गया था। आयोग ारा ां टस को दोषी

अपने राज पर िनयं ण ा िकया। कंपनी ने, यहां तक िक अपने नौकरों से पाया गया, और उ इं िडगो उ ादकों के साथ उनकी आ ामक रणनीित के

भी अिधक, ज ही प से धन की िनकासी का आयोजन िकया। इसने िलए दं िडत िकया गया। इसने कहा िक रै यत नील के उ ादन से लाभ नहीं उठा

बंगाल के राज से भारतीय व ुओं को खरीदना और उनका िनयात करना सकते थे। आयोग ने रै यतों को सलाह दी िक यिद वे अपने मौजूदा दािय ों को

शु िकया। इन खरीदों को 'िनवेश' के प म जाना जाता था। पूरा नहीं करते ह तो वे भिव म नील का उ ादन करने से मना कर सकते ह।

इस कार, 'िनवेश' के मा म से, बंगाल राज इं ड भेजा गया था। 18वीं िव ोह के बाद बंगाल का नील उ ादन िगर गया। हालाँ िक, बागान मािलक अब

शता ी के अंत तक, नाली भारत की रा ीय आय का लगभग 9% थी। िबहार चले गए ह। 1800 के अंत म िसंथेिटक रं गों की खोज होने पर उनके

वा िवक िनकासी और भी अिधक थी, ोंिक अं ेजी अिधका रयों के वेतन वसाय को गंभीर नुकसान आ था, लेिकन वे अभी भी िनमाण करने म

और अ आय का एक बड़ा िह ा और अं ेजी ापा रयों के ापा रक भा कामयाब रहे ।

ने भी इं ड म अपना रा ा खोज िलया।


Q41. Which one of the following novels elaborately discussed the
Sanyasi Revolt?
Q40. What happened a er the “Blue Rebellion” in 1859 in Bengal?
(a) Durgeshnandini
(a) Indigo produc on collapsed in Bengal
(b) Anandamath
(b) Planters shi ed their opera on to Bihar
(c) Pather Dabi
(c) Planters shi ed their opera on to Madras
(d) Chokher Bali
(d) Indigo Commission was set up
(e) None of the above/More than one of the above
(e) None of the above/More than one of the above
Answer: (b)
Answer: (e)
Op on (b) is correct: Sanyasi Revolt (1763-1800). Sanyasis were
Op on (e) is correct: Following the "Blue Rebellion" in Bengal in 1859,
originally peasants, and some were even evicted from their land
an Indigo Commission was formed to inves gate the indigo producing
forcefully. The disastrous famine of the 1770s and the economic
system. The planters were found guilty by the Commission, and they
policies of the Bri sh compelled them to fight against the Bri sh.
were chas sed for their aggressive tac cs with indigo producers. It
Sanyasis were joined by small zamindars, disbanded soldiers and rural
said that ryots could not profit from indigo produc on. The
poor. They fought the forces of the company and raided the treasuries
Commission advised the ryots that they may refuse to produce indigo
and the factories. Warren Has ngs was able to subdue these sanyasis,
in the future if they didn't fulfil their exis ng obliga ons.
but only a er a lot of struggle.
Bengal's indigo produc on plummeted following the uprising.
Both Hindus and Muslims par cipated equally in what is o en called
However, the planters have now relocated to Bihar. Their business was
the Fakir Rebellion. Some important leaders were Majnum Shah (or
seriously harmed when synthe c dyes were discovered in the late
Majnu Shah), Chirag Ali, Musa Shah, Bhawani Pathak and Debi
1800s, but they s ll managed to grow to manufacture.
Chaudhurani. Debi Chaudhurani’s par cipa on recognises the
40. 1859 म बंगाल म "नीला िव ोह" के बाद ा आ?
women’s role in early resistance against the Bri sh. Based on the
(a) बंगाल म इं िडगो उ ादन िगर गया Sanyasi Revolt, Bankim Chandra Cha erjee wrote a Semi-Historical
novel called Anandmath. Another novel he wrote was Devi
(b) ां टस ने अपना ऑपरे शन िबहार म थानां त रत कर िदया
Chaudhrani, as he saw the importance of women in the struggle
(c) ां टस ने अपना ऑपरे शन म ास म थानां त रत कर िदया
against the alien rule that posed threats to tradi onal Indian values.
(d) इं िडगो आयोग की थापना की गई थी 41. िन िल खत म से िकस उप ास म स ासी िव ोह पर िव ार से चचा की

(e) उपरो म से कोई नहीं/उपरो म से एक से अिधक गई है ?


Topic Name: Modern History
(a) दु गशनंिदनी According to the Bri sh governor Vansi art, the zamindars of
Midnapore sided with the ryots in case of conflict between the ryots
(b) आनंदमठ
and the English revenue collec ng officials. The zamindars of
(c) पाथेर डाबी Dhalbhum, Manbhum, Raipur, Panchet, Jha buni, Karnagarh, and
Bagri, living in the vast tract of Jungle Mahals of west and north-west
(d) चोखेर बाली
Midnapore—were ul mately dispossessed of their zamindaries by
(e) उपरो म से कोई नहीं/उपरो म से एक से अिधक
1800s. The important leaders of the uprisings were Damodar Singh
उ र: (b) and Jagannath Dhal.

िवक (b) सही है : 42. िमदनापुर और धालभूम म िव ोह 1760 के दशक के बाद िकसके िव

स ासी िव ोह (1763-1800)। स ासी मूल प से िकसान थे, और कुछ को आ था?

ज़बरद ी उनकी ज़मीन से बेदखल भी कर िदया गया था। 1770 के (a) अं ेजों के कठोर आिथक आदे श

िवनाशकारी अकाल और अं ेजों की आिथक नीितयों ने उ अं ेजों के (b) एक नई भूिम राज णाली का प रचय

खलाफ लड़ने के िलए मजबूर कर िदया। स ािसयों के साथ छोटे जमींदार, (c) अं ेजों ारा चाय बागान की शु आत

िव थािपत सैिनक और ामीण गरीब शािमल ए। उ ोंने कंपनी की ताकतों (d) कृिष जमींदारी और सामंती व था

का मुकाबला िकया और कोषागारों और कारखानों पर धावा बोल िदया। वारे न (e) उपरो म से कोई नहीं/उपरो म से एक से अिधक

हे ं इन स ािसयों को अपने वश म करने म सफल रहा, लेिकन ब त उ र: (b)

संघष के बाद ही। िवक (b) सही है :

िजसे अ र फकीर िव ोह कहा जाता है , उसम िहं दुओं और मुसलमानों दोनों िमदनापुर और धालभूम म िव ोह (1766-74) - अं ेजों ने 1760 म िमदनापुर

ने समान प से भाग िलया। कुछ मह पूण नेता मजनू शाह (या मजनू शाह), पर क ा कर िलया था, और उस समय लगभग 3,000 जमींदार और

िचराग अली, मूसा शाह, भवानी पाठक और दे वी चौधुरानी थे। दे बी चौधुरानी की तालुकदार थे, िजनके रै यतों के साथ सौहादपूण संबंध थे। लेिकन 1772 म

भागीदारी अं ेजों के खलाफ शु आती ितरोध म मिहलाओं की भूिमका को अं ेजों ारा एक नई भूिम राज णाली की शु आत के बाद यह

पहचानती है । स ासी िव ोह पर आधा रत, बंिकम चं चटज ने आनंदमठ सामंज पूण प र बदल गया।

नामक एक अध-ऐितहािसक उप ास िलखा। उनका िलखा एक और उप ास ि िटश गवनर वंिसटाट के अनुसार, िमदनापुर के जमींदारों ने रै यतों और

दे वी चौधरानी था, ोंिक उ ोंने पारं प रक भारतीय मू ों के िलए खतरा पैदा अं ेजी राज एक करने वाले अिधका रयों के बीच संघष के मामले म रै यतों

करने वाले िवदे शी शासन के खलाफ संघष म मिहलाओं के मह को दे खा। का प िलया। पि म और उ र-पि म िमदनापुर के जंगल महलों के िवशाल

पथ म रहने वाले धालभूम, मानभूम, रायपुर, पंचेत, झटीबुनी, कणगढ़ और

Q42. Revolt in Midnapore and Dhalbhum took place a er the 1760s बागड़ी के जमींदारों को अंततः 1800 के दशक तक उनकी जमींदा रयों से
against the:
बेदखल कर िदया गया था। िव ोह के मह पूण नेता दामोदर िसंह और
(a) Harsh economic order of the Bri sh
(b) Introduc on of a new land revenue system जग ाथ ढल थे।
(c) Introduc on of tea planta on by Bri sh Q43. Which one of the following provinces was related to the Pabna
(d) Agrarian landlordism and the feudal system Agrarian Movement?
(e) None of the above/More than one of the above (a) Madras
Answer: (b) (b) North-West Fron er Province
Op on (b) is correct: Revolt in Midnapore and Dhalbhum (c) Bengal
(1766-74)-The English took hold of Midnapore in 1760, and at that (d) Central Province
me, there were about 3,000 zamindars and talukdars having cordial (e) None of the above/More than one of the above
rela ons with their ryots. But this harmonious scenario changed a er Answer: (c)
the introduc on of a new land revenue system by the English in 1772.
Topic Name: Modern History
Op on (c) is correct: Shah Chandra Roy, Shambhu Pal, and Khoodi nazrana were the norm for the bulk of the cul vators. Food and other
Mollah led the Pabna Agrarian Uprising, which was supported by B.C. basics had become more expensive as a result of the First World War.
Cha erjee and R.C. Du (1873; Pabna district, East Bengal, now in This deteriorated the situa on of UP peasants.
Bangladesh). During the 1870s and 1880s, agrarian rebellion erupted 44. अवध म िकसान आं दोलनों के संदभ म 'नजराना' श का अथ है :
across most of Eastern Bengal as a result of zamindars' repressive
(a) उ िकराए
tac cs. The zamindars raised rents above what was permi ed,
preven ng tenants from obtaining occupancy rights under Act X of (b) अवैध लेवी
1859. The zamindars used various aspects to achieve their goals, like
(c) कृिष कर
forcible evic ons, ca le and crop seizures, and lengthy, expensive
(d) नवीनीकरण शु
li ga on in courts where the poor peasant was at a disadvantage.

43. िन िल खत म से कौन सा ां त पाबना कृिष आं दोलन से संबंिधत था? (e) उपरो म से कोई नहीं/उपरो म से एक से अिधक

(a) म ास उ र: (d)

(b) उ र-पि म सीमा ां त िवक (d) सही है :

(c) बंगाल 1857 के िव ोह के बाद, अवध तालुकदारों ने अपनी भूिम पर पुनः दावा िकया।

(d) म ां त इसके प रणाम प तालुकदारों या बड़े जमींदारों ने सूबे के कृषक समाज पर

(e) उपरो म से कोई नहीं/उपरो म से एक से अिधक अपनी पकड़ मजबूत कर ली। अिधक लगान, संि बेदखली (बेदखली),

उ र: (c) गैर-कानूनी शु , नवीनीकरण शु और नज़राना, िकसानों के बड़े िह े के

िवक (c) सही है : िलए आदश थे। थम िव यु के प रणाम प भोजन और अ बुिनयादी

शाह चं रॉय, शंभू पाल और खुदी मो ा ने पाबना कृिष िव ोह का नेतृ चीज अिधक महं गी हो गई थीं। इससे यूपी के िकसानों की हालत खराब हो गई।

िकया, िजसे बी.सी. चटज और आर.सी. द (1873; पाबना िजला, पूव बंगाल,
Q45. During the 19th century, Tribal people considered
अब बां ादे श म)। 1870 और 1880 के दशक के दौरान, जमींदारों की
moneylenders and traders as evil outsiders and the cause of their
दमनकारी रणनीित के प रणाम प पूव बंगाल के अिधकां श िह ों म कृिष misery. What was the reason?
(a) They were dependent on traders and moneylenders
िव ोह भड़क उठे । 1859 के अिधिनयम X के तहत िकरायेदारों को अिधभोग
(b) High prices charged by traders for goods and high rate of interest
अिधकार ा करने से रोकते ए जमींदारों ने अनुमित से अिधक लगान बढ़ा on the loans given by moneylenders

िदया। जमींदारों ने अपने ल ों को ा करने के िलए िविभ पहलुओं का (c) Moneylenders and trader wanted to buy forest produce
(d) statements (a) and (c) is correct
इ ेमाल िकया, जैसे जबरन बेदखली, मवेिशयों और फसल की ज ी, और
(e) None of the above/More than one of the above
अदालतों म लंबी, महं गी मुकदमेबाजी, जहां गरीब िकसान घाटे म था। Answer: (b)

Q44. The term ‘nazrana’ in the context of Peasant movements in Op on (b) is correct: During the nineteenth century, tribal tribes
Awadh refers to: frequently had to buy and sell in order to obtain products that were

(a) High rents not produced locally. As a result, they were reliant on dealers and

(b) Illegal levies moneylenders. Traders came about with items for sale and charged

(c) Farm tax exorbitant prices. Moneylenders provided loans to the tribals,

(d) Renewal fees allowing them to meet their cash demands while also increasing their

(e) None of the above/More than one of the above earnings. However, the interest rates on the loans were frequently

Answer: (d) extremely expensive. As a result, market and commerce were

Op on (d) is correct: Following the 1857 revolt, the Awadh taluqdars frequently associated with debt and poverty among the tribals. As a
reclaimed their lands. The talukdars, or big landlords, ghtened their result, they grew to regard the moneylender and dealer as malevolent
grip on the province's agrarian society as a result. High rents, outsiders who were to blame for their plight.
summary evic ons (bedakhali), unlawful levies, renewal fees, and
Topic Name: Modern History
45. 19वीं शता ी के दौरान आिदवासी लोग सा कारों और ापा रयों को between Bhagalpur and Rajmahal as autonomous. The rebellion was
suppressed by 1856.
दु बाहरी और अपनी दु दशा का कारण मानते थे। ा कारण था?
46. िस संथाल िव ोह िन िल खत म से िकस े से संबंिधत है ?
a) वे ापा रयों और सा कारों पर िनभर थे
(a) पंजाब और ह रयाणा
b) ापा रयों ारा माल के िलए उ कीमत और सा कारों ारा िदए गए
(b) गुजरात
ऋण पर उ ाज दर
(c) पि म बंगाल
c) सा कार और ापारी वन उपज खरीदना चाहते थे
(d) पंजाब
d) कथन (ए) और (सी) सही ह
(e) उपरो म से कोई नहीं/उपरो म से एक से अिधक
e) उपरो म से कोई नहीं/उपरो म से एक से अिधक
उ र: (e)
उ र: (b)
िवक (e) सही है :
िवक (b) सही है :
संथालों के िनरं तर उ ीड़न, एक कृिष समुदाय जो राजमहल पहािड़यों (िबहार)
उ ीसवीं शता ी के दौरान, आिदवासी जनजाितयों को थानीय र पर
के मैदानी इलाकों म बसने के िलए भाग गए थे, ने 1855-56 म और उसके
उ ािदत नहीं होने वाले उ ादों को ा करने के िलए अ र खरीदना और
आसपास जमींदारों के खलाफ संथाल िव ोह का नेतृ िकया।
बेचना पड़ता था। प रणाम प, वे डीलरों और सा कारों पर िनभर थे।
सा कारों, िज दू सरों के साथ-साथ पुिलस का भी समथन ा था, ने िकसानों
ापारी िब ी के िलए सामान लेकर आए और मनमानी कीमत वसूल की।
से दमना क वसूली और भूिम से बेदखली करने के िलए जमींदारों का साथ
सा कारों ने आिदवािसयों को ऋण दान िकया, िजससे उ अपनी आय बढ़ाने
िदया। िव ोह एक ि िटश िवरोधी आं दोलन म बदल गया। दो भाइयों, िसद् धू
के साथ-साथ अपनी नकद मां गों को पूरा करने की अनुमित िमली। हालां िक,
और का के अधीन, संथालों ने कंपनी शासन के अंत की घोषणा की और
ऋणों पर ाज दर अ र ब त महं गी थीं। प रणाम प, आिदवािसयों के
भागलपुर और राजमहल के बीच के े को ाय घोिषत कर िदया। 1856
बीच बाजार और वािण अ र कज और गरीबी से जुड़े थे। प रणाम प,
तक िव ोह को दबा िदया गया था।
वे सा कार और ापारी को दु बाहरी लोगों के प म मानने लगे जो उनकी

दु दशा के िलए िज ेदार थे। Q47. Which of the following persons is/are associated with the
Santhal revolt?

Q46. The famous Santhal Rebellion is related to which of the (a) Sidhu
following regions? (b) kanhu

(a) Punjab and Haryana (c) Raj Kumar Shukla

(b) Gujraat (d) Birsa Munda

(c) West Bengal (e) None of the above/More than one of the above

(d) Punjab Answer: (e)

(e) None of the above/More than one of the above Op on (e) is correct: Santhals are the tribals of Chota Nagpur plateau

Answer: (e) and Rajmahal hills. Santhal revolt was directed against the ‘Dikus’ (the

Op on (e) is correct: The con nued oppression of the Santhals, an outsiders), which included money lenders, zamindars, police etc. They
agricultural community who had fled to se le in the plains of the perceived them as direct reasons for their misery. The rebellion
Rajmahal hills (Bihar), led to the Santhal rebellion against the turned into an an -Bri sh movement. Under Sidhu and Kanhu, two
zamindars in and around 1855-56. brothers, the Santhals proclaimed an end to Company rule and

The money-lenders who had the support of the police, among others, declared the area between Bhagalpur and Rajmahal autonomous. The
had joined the zamindars to subject the peasants to oppressive rebellion was suppressed by 1856.
exac ons and dispossession of lands. The rebellion turned into an 47. िन िल खत म से कौन सा/से संथाल िव ोह से संबंिधत
an -Bri sh movement. Under Sidhu and Kanhu, two brothers, the
है /ह?
Santhals proclaimed an end to Company rule and declared the area
Topic Name: Modern History
(a) िसद् धू ने ि िटश राजनीितक एजट की मृ ु दे खी। मुख िनरं ग िफडू ने 1843 म एक

(b) का िव ोह का नेतृ िकया, िजसम ि िटश चौकी पर हमला और कई सैिनकों की

(c) राजकुमार शु ा मौत शािमल थी।

(d) िबरसा मुंडा


Q49. At Delhi, the nominal and symbolic leadership of the revolt of
(e) उपरो म से कोई नही/ं उपरो म से एक से अिधक
1857 belonged to the Mughal emperor, Bahadur Shah, but the real
उ र: (e) command lay with a court of soldiers headed by:
(a) Birjis Qadir
िवक (e) सही है :
(b) Khan Bahadur
संथाल छोटा नागपुर पठार और राजमहल पहािड़यों के आिदवासी ह।
(c) Bakht Khan
संथाल िव ोह 'िदकु' (बाहरी) के खलाफ िनदिशत था, िजसम सा कार, (d) Shah Mal
(e) None of the above/More than one of the above
जमी ंदार, पुिलस आिद शािमल थे। वे उ अपने दु ख के कारणों के
Answer: (c)
प म मानते थे। िव ोह एक ि िटश िवरोधी आं दोलन म बदल गया। Op on (c) is correct: The Mughal emperor, Bahadur Shah, was

िसद् धू और का , दो भाइयों के अधीन, संथालों ने कंपनी शासन के अंत nominally and symbolically in charge of the insurrec on of 1857 in
Delhi, but the true command was in the hands of a court of soldiers
की घोषणा की और भागलपुर और राजमहल के बीच के े को ाय
led by General Bakht Khan, who had led the revolt of Bareilly troops
घोिषत कर िदया। 1856 तक िव ोह को दबा िदया गया था। and brought them to Delhi. The court was made up of ten members:

Q48. The Singphos Rebellion was concentrated in which of the six from the army and four from the civil service. In the name of the
following regions? emperor, the court managed the state's affairs. Emperor Bahadur

(a) Punjab and North West Shah was probably the weakest link in the revolt's chain of command.

(b) Bihar His frail personality, advanced age, and lack of leadership quali es

(c) Coromandel Coast resulted in poli cal weakness at the heart of the revolu on, causing

(d) The Western Ghats irreparable damage.

(e) None of the above/More than one of the above 49. िद ी म, 1857 के िव ोह का नाममा और तीका क नेतृ मुग़ल
Answer: (e)
स ाट बहादु र शाह का था, लेिकन वा िवक कमान सैिनकों के एक दरबार के
Op on (e) is correct: The rebellion of the Singphos in Assam in early
1830 was immediately quelled, but they con nued to organise revolts. पास थी िजसका नेतृ :
An uprising in 1839 saw the death of the Bri sh poli cal agent. Chief (a) िबरिजस कािदर
Nirang Phidu led an uprising in 1843, which involved an a ack on the
(b) खान बहादु र
Bri sh garrison and the death of many soldiers.

48. िसंगफोस िव ोह िन िल खत म से िकस े म कि त था? (c) ब खान

(a) पंजाब और उ र पि म (d) शाह मल

(b) िबहार (e) उपरो म से कोई नहीं/उपरो म से एक से अिधक

(c) कोरोमंडल तट उ र: (c)

(d) पि मी घाट िवक (c) सही है :

(e) उपरो म से कोई नहीं/उपरो म से एक से अिधक मुगल स ाट, बहादु र शाह, िद ी म 1857 के िव ोह के नाममा और

उ र: (e) तीका क प से भारी थे, लेिकन स ी कमान जनरल ब खान के नेतृ

िवक (e) सही है : वाले सैिनकों के एक दरबार के हाथों म थी, िज ोंने नेतृ िकया था बरे ली के

1830 की शु आत म असम म िसंगफोस के िव ोह को तुरंत दबा िदया गया सैिनकों का िव ोह और उ िद ी ले आया। अदालत दस सद ों से बनी थी:

था, लेिकन उ ोंने िव ोहों को संगिठत करना जारी रखा। 1839 म एक िव ोह छह सेना से और चार िसिवल सेवा से। स ाट के नाम पर, दरबार रा के
Topic Name: Modern History
मामलों का बंधन करता था। िव ोह की कमान की ृंखला म स ाट बहादु र Q51. The land and property of Muslims were confiscated on a large
scale a er the revolt of 1857. This is due to:
शाह शायद सबसे कमजोर कड़ी थे। उनके कमजोर , उ त उ और
(a) Protect landlords and zamindars who supported the Bri sh
नेतृ के गुणों की कमी के कारण ां ित के क म राजनीितक कमजोरी आई, during the revolt
(b) Respect the customary religious and social prac ces of the
िजससे अपूरणीय ित ई।
people in India
Q50. Who was the Governor-General of India when the revolt of
(c) Bri sh considered Muslims responsible for the rebellion in a big
1857 took place?
way
(a) Lord Canning
(d) Hold their kingdoms as subordinates of the Bri sh Crown
(b) Lord Dalhousie
(e) None of the above/More than one of the above
(c) Lord Hardinge
Answer: (c)
(d) Lord Ellenborough
Op on (c) is correct: At all levels of the 1857 revolt—people, soldiers,
(e) None of the above/More than one of the above
and leaders—there was total coopera on between Hindus and
Answer: (a)
Muslims. All rebels recognised Bahadur Shah Zafar, a Muslim, as
Op on (a) is correct: The following major events took place during
emperor, and the Hindu sepoys in Meerut immediately began
the tenure of Lord Canning (1856-1862)
marching to Delhi, the Mughal imperial capital. According to Maulana
● Establishment of three universi es at Calcu a, Madras and
Azad, “Two facts stand out clearly in the midst of the tangled story of
Bombay in 1857.
the Rising of 1857. The first is the remarkable sense of unity among
● Revolt of 1857.
the Hindus and the Muslims of India in this period. The other is the
● Transfer of control from East India Company to the Crown, the
deep loyalty which the people felt for the Mughal Crown.” Rebels and
Government of India Act, 1858. (He became first Viceroy of India)
sepoys, both Hindu and Muslim, respected each other’s sen ments.
● ‘White Mu ny’ by European troops in 1859.
Muslims' land and possessions were confiscated on a huge scale a er
● Indian Councils Act of 1861.
the revolu on, and they were treated with mistrust and contempt.
50. 1857 की ां ित के समय भारत का गवनर जनरल कौन था?
The Bri sh claimed they were directly responsible for the insurrec on.
(a) लॉड कैिनंग 51. 1857 के िव ोह के बाद मुसलमानों की भूिम और संपि को बड़े पैमाने

(b) लॉड डलहौजी पर ज कर िलया गया। इसका कारण है :

(c) लॉड हािडग (a) िव ोह के दौरान अं ेजों का समथन करने वाले जमींदारों और जमींदारों की

(d) लॉड एलेनबरो र ा करना

(e) उपरो म से कोई नहीं/उपरो म से एक से अिधक (b) भारत म लोगों की परं परागत धािमक और सामािजक थाओं का स ान

उ र: (a) कर

िवक (a) सही है : (c) अं ेज मुसलमानों को िव ोह के िलए बड़े पैमाने पर िज ेदार मानते थे

लॉड कैिनंग (1856-1862) के कायकाल के दौरान िन िल खत मुख घटनाएं (d) उनके रा ों को ि िटश ताज के अधीन रखना

ईं (e) उपरो म से कोई नहीं/उपरो म से एक से अिधक

● 1857 म कलक ा, म ास और बॉ े म तीन िव िव ालयों की थापना। उ र: (c)

● 1857 का िव ोह। िवक (c) सही है :

● ई इं िडया कंपनी से ाउन को िनयं ण का ह ां तरण, भारत सरकार 1857 के िव ोह के सभी रों पर- लोग, सैिनक और नेता- िहं दुओं और

अिधिनयम, 1858। (वह भारत का पहला वायसराय बना) मुसलमानों के बीच पूण सहयोग था। सभी िव ोिहयों ने बहादु र शाह जफर, एक

● 1859 म यूरोपीय सैिनकों ारा ' ेत िव ोह'। मु म, को स ाट के प म मा ता दी, और मेरठ म िहं दू िसपािहयों ने तुरंत

● भारतीय प रषद अिधिनयम 1861। मुगल सा ा की राजधानी िद ी की ओर माच करना शु कर िदया।

मौलाना आज़ाद के अनुसार, “1857 के िव ोह की पेचीदा कहानी के बीच दो


Topic Name: Modern History
त प से सामने आते ह। पहला इस अविध म भारत के िहं दुओं और (e) उपरो म से कोई नहीं/उपरो म से एक से अिधक

मुसलमानों के बीच एकता की उ ेखनीय भावना है । दू सरा वह गहरी िन ा है उ र: (c)

जो लोगों ने मुगल ताज के िलए महसूस की। िव ोही और िसपाही, िहं दू और िवक (c) सही है :

मुसलमान दोनों, एक-दू सरे की भावनाओं का स ान करते थे। बहादु र शाह 1857 के िव ोह का तीक बन गया, और िद ी इसका क बन

ां ित के बाद बड़े पैमाने पर मुसलमानों की भूिम और संपि को ज कर गया। बहादु र शाह को नेता बनाने का यह कदम इस त की ीकृित थी िक

िलया गया, और उनके साथ अिव ास और अवमानना की गई। अं ेजों ने दावा मुगल वंश का लंबा शासन भारत की राजनीितक एकता का पारं प रक तीक

िकया िक वे िव ोह के िलए सीधे तौर पर िज ेदार थे। बन गया था। िसपािहयों के इस कृ ने सैिनकों के िव ोह को एक ां ितकारी

यु म बदल िदया, जबिक िव ोह म भाग लेने वाले सभी भारतीय मुखों ने


Q52. During the revolt of 1857, why was Bahadur Shah Zafar
मुगल स ाट के ित अपनी वफादारी की घोषणा करने म ज बाजी की। यह
proclaimed the Emperor of India?
(a) He was a capable leader. इस त का भी ितिबंब है िक िव ोही राजनीित से े रत थे।

(b) Delhi was the capital of the Bri sh empire, and the Mughals held
मुगलों ने दे श के एक ब त बड़े िह े, सरदारों और अ छोटे शासकों पर
Delhi as their power centre.
शासन िकया था और मुगल शासकों की ओर से िविभ े ों को िनयंि त िकया
(c) Long reign of the Mughal dynasty had become the tradi onal
symbol of India’s poli cal unity. था। ि िटश िव ार ने छोटे शासकों के िलए एक कार का खतरा पैदा कर
(d) He was quite willing to challenge the mighty Bri sh power.
िदया था, और उनम से कई ने महसूस िकया था िक यिद मुगल स ाट िफर से
(e) None of the above/More than one of the above
Answer: (c) शासन कर सकते ह, तो वे भी अपने े ों को एक बार िफर से शासन करने के

Op on (c) is correct: Bahadur Shah became the symbol of the 1857 िलए वापस ा कर सकते ह, हालां िक मुगल अिधकार के अधीन।
Revolt, and Delhi became its centre. This move of making Bahadur
Shah the leader was an acceptance of the fact that the long reign of
Q53. Which among the following were the poli cal causes leading to
the Mughal dynasty had become the tradi onal symbol of India’s
the 1857 revolt?
poli cal unity. This act of the sepoys transformed a mu ny of soldiers
(a) The policy of Doctrine of Lapse was introduced by Lord
into a revolu onary war, while all Indian chiefs who par cipated in
Dalhousie.
revolt hastened to proclaim their loyalty to the Mughal Emperor. It is
(b) The annexa on of Awadh on charges of maladministra on.
also a reflec on of the fact rebels were poli cally mo vated.
(c) The policy of Subsidiary Alliance by Lord Wellesley.
The Mughals had ruled a very large part of the country, the chie ains
(d) Absentee Sovereignship
and other small rulers and controlled different regions on behalf of
(e) None of the above/More than one of the above
the Mughal rulers. The Bri sh expansion posed a kind of threat to
Answer: (e)
small rulers, and many of them had felt that if the Mughal Emperor
Op on (e) is correct: There were a number of poli cal causes which
could rule again, they could also get their territories back to rule once
were func onal behind the rising of this great rebellion. All the
again, though under Mughal authority.
above-men oned causes are correct- the policy of doctrine of lapse
52. 1857 के िव ोह के दौरान बहादु र शाह जफर को भारत का स ाट ों introduced by Dalhousie to annex states with no legal heir, the

घोिषत िकया गया था? annexa on of Awadh in 1856 on the grounds of maladministra on
and the policy of subsidiary alliance were all filling Indians with fury
(a) वह एक स म नेता थे।
understanding the disastrous Bri sh policies to expand their territorial
(b) िद ी ि िटश सा ा की राजधानी थी, और मुगलों ने िद ी को अपने reach and poli cal conquests in India. Hence, all the statements are

श क के प म आयोिजत िकया था। correct.

53. 1857 के िव ोह के िलए िन िल खत म से कौन से राजनीितक कारण थे?


(c) मुगल वंश का लंबा शासन भारत की राजनीितक एकता का पारं प रक
(a) डॉ न ऑफ लै की नीित लॉड डलहौजी ारा पेश की गई थी।
तीक बन गया था।
(b) कुशासन के आरोप म अवध का िवलय।
(d) वह श शाली ि िटश श को चुनौती दे ने के िलए काफी तैयार था।
Topic Name: Modern History
(c) लॉड वेले े ारा सहायक गठबंधन की नीित। िवक (c) सही है :

(d) अनुप थत सं भुता भारत के सै मामलों को दे खने के िलए पील आयोग िनयु िकया गया था।

(e) उपरो म से कोई नहीं/उपरो म से एक से अिधक इसने िसफा रश की िक "दे शी सेना िविभ रा ीयताओं और जाितयों से बनी

उ र: (e) होनी चािहए, और एक सामा िनयम के प म, ेक रे िजमट के मा म से

िवक (e) सही है : िमि त प से िमि त होनी चािहए"। इसिलए, अगले कुछ वष के दौरान, िजन

इस महान िव ोह के उठने के पीछे कई राजनीितक कारण थे जो काया क रे िजमटों ने िव ोह िकया था, उ भंग कर िदया गया था, सभी रे िजमटों म

थे। उपयु सभी कारण सही ह- डलहौजी ारा िबना िकसी कानूनी जाितयों को अिधक समान प से िमलाया गया था, और भत पंजाब पर कि त

उ रािधकारी वाले रा ों को जोड़ने के िलए पगत के िस ां त की नीित, रही, जो िव ोह के दौरान वफादार रहे ।

1856 म कुशासन के आधार पर अवध का िवलय और सहायक गठबंधन की Q55. During whose Viceroyship did the ‘White Mu ny’ take place?
(a) Lord Has ngs
नीित सभी भारतीयों को रोष समझ से भर रही थी िवनाशकारी ि िटश नीितयां
(b) Lord Hardinge
भारत म अपनी े ीय प ं च और राजनीितक िवजय का िव ार करने के िलए। (c) Lord Dalhousie

(d) Lord Warren Has ngs


अतः सभी कथन सही ह।
(e) None of the above/More than one of the above
Answer: (e)
Q54. A er the Revolt of 1857, the Peel Commission was formed to?
Op on (e) is correct: The 1859 White Mu ny of European troops took
(a) Facilitate entry of more Indians into civil services by lowering the
place during Viceroy Lord Canning. With the Queen's Proclama on of
age
1858, India's administra on was handed over to the Bri sh Crown.
(b) Lay down the founda on for Western educa on in India
But the transfer of power that the Soldiers of the three Presidency
(c) Look into the military affairs of India
Armies would now require making over their allegiance to the Queen
(d) Blueprint future rela ons between the Crown and the Princely
and the Royal Army. This led to resentment among the European
States
troops as while serving the East India Company (EIC), they used to
(e) None of the above/More than one of the above
receive ‘ba a' (Extra allowance) to cover the various expenditures
Answer: (c)
rela ng to the opera ons outside their home territories, but a er
Op on (c) is correct: The Peel Commission was appointed to look into
shi ing their allegiance to the Bri sh Army, they did not get any extra
the military affairs of India. It recommended that “the na ve army
payment. Also, the officers of the Bri sh Army were senior in age to
should be composed of different na onali es and castes, and as a
those of the same rank in the EIC unit.
general rule, mixed promiscuously through each regiment”.
55. ' ेत िव ोह' िकसके वायसराय काल म आ था?
Therefore, during the next few years, regiments that had mu nied
were disbanded, castes were more evenly mixed across the regiments, (a) लॉड हे ं
and recruitment remained focused on Punjab, which remained loyal
(b) लॉड हािडग
during the mu ny.
(c) लॉड डलहौजी
54. 1857 के िव ोह के बाद पील आयोग का गठन िकया गया था?
(d) लॉड वारे न हे ं
(a) आयु कम करके अिधक भारतीयों को िसिवल सेवाओं म वेश की सुिवधा
(e) उपरो म से कोई नहीं/उपरो म से एक से अिधक
दान करना
उ र: (e)
(b) भारत म पि मी िश ा की नींव रखी
िवक (e) सही है :
(c) भारत के सै मामलों को दे ख
वायसराय लॉड कैिनंग के दौरान यूरोपीय सैिनकों का 1859 का ेत िव ोह आ
(d) ाउन और रयासतों के बीच भिव के संबंधों का खाका
था। 1858 की रानी की उद् घोषणा के साथ, भारत का शासन ि िटश ाउन
(e) उपरो म से कोई नहीं/उपरो म से एक से अिधक
को सौंप िदया गया था। लेिकन स ा के ह ां तरण के िलए तीन ेसीडसी
उ र: (c)
Topic Name: Modern History
सेनाओं के सैिनकों को अब रानी और शाही सेना के ित अपनी िन ा को अकाली आं दोलन िसंह सभा आं दोलन का िव ार था। इसे गु ारा सुधार

बदलने की आव कता होगी। इससे यूरोपीय सैिनकों म नाराजगी पैदा ई आं दोलन के नाम से भी जाना जाता था। इसका ल िसख गु ारों को

ोंिक ई इं िडया कंपनी (ईआईसी) म सेवा करते समय उ अपने गृह े ों उदासी महं तों के चंगुल से मु कराना था। उस समय तक उदासी महं तों की

के बाहर संचालन से संबंिधत िविभ खच को कवर करने के िलए 'ब ा' थित वंशानुगत हो गई थी। ये महं त िन ावान और िति यावादी थे िज

(अित र भ ा) ा होता था, लेिकन अपनी िन ा को थानां त रत करने के सरकारी समथन का लाभ िमलता था।

बाद ि िटश सेना, उ कोई अित र भुगतान नहीं िमला। साथ ही, ि िटश शासन ने 1921 म अकािलयों के अिहं सक असहयोग स ा ह के खलाफ

सेना के अिधकारी EIC इकाई म समान रक के अिधका रयों से उ म व र थे। कठोर कदम उठाए, लेिकन लोकि य मां गों के आगे झुकने के िलए मजबूर

िकया गया। 1922 म, सरकार ने िसख गु ारा अिधिनयम को मंजूरी दी और


Q56. The basic aim of the Akali movement of the 1920s was:
1925 म इसम संशोधन िकया, िजसने िशरोमिण गु ारा बंधक सिमित
(a) to provide modern western educa on to the Sikhs
(b) to counter the prosely sing ac vi es of Chris an missionaries (SGPC) को शीष िनकाय के प म िसख जनता को गु ारों का िनयं ण दान

(c) libera ng the Sikh gurudwaras from the control of corrupt


िकया।
mahants
अकाली आं दोलन एक े ीय आं दोलन था। यह कोई सा दाियक आ ोलन
(d) To establish a network of Khalsa schools
(e) None of the above/More than one of the above नहीं था। कुछ असहमत आवाजों के बावजूद, अकाली नेताओं ने रा ीय मु
Answer: (c)
संघष म मह पूण भूिमका िनभाई।
Op on (c) is correct: Akali movement was an extension of the Singh
Sabha Movement. It was also known as the Gurdwara Reform
Q57. Consider the following:
Movement. Its goal was to free Sikh gurudwaras from the grip of
1. Naoroji Furdonji
corrupt Udasi mahants. By that me, the posi on of Udasi mahants
2. Dadabhai Naoroji
had become hereditary. These mahants were loyalists and
3. S.S. Bengalee
reac onaries who benefited from Government support.
Who of the above was/were associated with Rahnumai
The administra on employed coercive measures against the Akalis'
Mazdayasnan Sabha?
nonviolent non-coopera on satyagraha in 1921 but was forced to bow
(a) 1 only
before the popular demands. In 1922, the government approved the
(b) 1 and 2 only
Sikh Gurdwaras Act and amended it in 1925, which granted the Sikh
(c) 2 and 3 only
masses control of the gurudwaras, with the Shiromani Gurudwara
(d) 1,and 3
Prabandhak Commi ee (SGPC) as the top body.
(e) None of the above/More than one of the above
The Akali Movement was a regional movement. It was not a
Answer: (e)
communal movement. Despite some disagreeing voices, the Akali
Op on (e) is correct: The Rahnumai Mazdayasnan Sabha, or the
leaders played a significant role in the na onal libera on struggle.
Religious Reform Associa on, was created in 1851 by a group of
56. 1920 के दशक के अकाली आ ोलन का मूल उ े था:
English educated Parsis with the goal of “regenera on of the social
(a) िसखों को आधुिनक पि मी िश ा दान करने के िलए condi ons of the Parsis and the restora on of the Zoroastrian religion

(b) ईसाई िमशन रयों की धमातरण गितिविधयों का मुकाबला करने के िलए to its pris ne purity”. Naoroji Furdonji, Dadabhai Naoroji, K.R. Cama,
and S.S. Bengalee were the leaders of this movement. The newspaper
(c) िसख गु ारों को महं तों के िनयं ण से मु कराना
Rast Go ar (Truth-Teller) promoted the reform message. The Parsi
(d) खालसा ू लों का नेटवक थािपत करना religious ceremonies and customs were changed and redefined.
In the social domain, efforts were undertaken to improve the posi on
(e) उपरो म से कोई नहीं/उपरो म से एक से अिधक
of Parsi women by abolishing the purdah rule, promo ng educa on,
उ र: (c) and raising the marriage age. As a result of this movement, the Parsis

िवक (c) सही है : gradually became the most westernised segment of Indian society.

57. िन िल खत को धयान मे रखते ए:


Topic Name: Modern History
1. नौरोजी फरदु नजी From 1840 onwards, the movement turned revolu onary under the

2. दादाभाई नौरोजी leadership of Haji's son, Dudu Mian. From the village to the provincial
level. He established an organisa onal structure for the movement,
3. एस. एस. बंगाली
with a khalifa or authorised deputy at each level. Apart from the
उपरो म से कौन रहनुमाई मजदाय ान सभा से संबंिधत था/थे?
indigo growers, the Fara'idis organised a paramilitary force equipped
(a) केवल 1 with clubs to combat the zamindars, who were largely Hindu
landowners, but there were some Muslim landlords too.
(b) केवल 1 और 2
58. फ़राज़ी आं दोलन, िजसने िव ास के इ ामी ंभों पर जोर िदया, िकसके
(c) केवल 2 और 3
ारा थािपत िकया गया था?
(d) 1, और 3
(a) हाजी शरीअतु ाह
(e) उपरो म से कोई नहीं/उपरो म से एक से अिधक
(b) िमजा गुलाम अहमद
उ र: (e)
(c) सैयद अहमद खान
िवक (e) सही है :
(d) महमूद-उल-हसन
रहनुमाई मजदाय ान सभा, या धािमक सुधार संघ, 1851 म अं ेजी िशि त
(e) उपरो म से कोई नहीं/उपरो म से एक से अिधक
पारिसयों के एक समूह ारा "पारिसयों की सामािजक प र थितयों के उ ान
उ र: (a)
और पारसी धम की बहाली" के ल के साथ बनाया गया था। इसकी ाचीन
िवक (a) सही है :
शु ता ”। नौरोजी फरदोनजी, दादाभाई नौरोजी, के.आर. कामा और एस.एस.
हाजी शरीअतु ा ने 1818 म फ़राज़ी आं दोलन की थापना की थी। इ ािमक
बगाली इस आं दोलन के नेता थे। समाचार प रा गो ार (स बताने वाला)
धम पर ज़ोर दे ने के कारण इस आं दोलन को फ़राज़ी आं दोलन के प म भी
ने सुधार संदेश को बढ़ावा िदया। पारसी धािमक समारोहों और रीित- रवाजों को
जाना जाता था। इसकी गितिविध पूव बंगाल म े के मु म समुदाय के बीच
बदला और पुनप रभािषत िकया गया।
सामािजक नवाचारों या गैर-इ ामी गितिविधयों को ख करने के ल के
सामािजक े म पदा था को समा कर, िश ा को बढ़ावा दे कर और िववाह
साथ ई थी। यह मुसलमानों के प म उनकी िज ेदा रयों पर भी उनका
की आयु बढ़ाकर पारसी मिहलाओं की थित म सुधार के यास िकए गए। इस
ान आकिषत करता है ।
आं दोलन के प रणाम प, पारसी धीरे -धीरे भारतीय समाज का सबसे पि मी
1840 के बाद हाजी के पु दू दू िमयां के नेतृ म आं दोलन ां ितकारी हो गया।
िह ा बन गए।
गां व से लेकर खंड र तक। उ ोंने ेक र पर खलीफा या अिधकृत

Q58. Faraizi Movement, which led to an emphasis on the Islamic िड ी के साथ आं दोलन के िलए एक संगठना क संरचना की थापना की।
pillars of faith, was founded by-
इं िडगो उ ादकों के अलावा, फरीिदयों ने जमींदारों का मुकाबला करने के िलए
(a) Haji Shariatullah
बों से लैस एक अधसैिनक बल का गठन िकया, जो बड़े पैमाने पर िहं दू
(b) Mirza Ghulam Ahmad
(c) Syed Ahmed Khan जमींदार थे, लेिकन कुछ मु म जमींदार भी थे।
(d) Mahmud-ul-Hasan
Q59. Which one of the following is the characteris c of the Sree
(e) None of the above/More than one of the above
Narayana Guru Dharma Paripalana (SNDP) Movement?
Answer: (a)
(a) It sought to introduce proper management of Hindu religious
Op on (a) is correct: Haji Shariatullah founded the Faraizi Movement
ins tu ons.
in 1818. The movement was also known as the Faraizi Movement
(b) It stood for the preserva on of the status quo in socio-religious
because of its emphasis on the Islamic religion. Its ac vity took place
ma ers.
in East Bengal with the goal of eradica ng social innova ons or
(c) It specialised in taking care of those women who were exploited
un-Islamic ac vi es among the Muslim community of the area. It also
and then discarded by society.
draws their a en on to their responsibili es as Muslims.
(d) It was a regional movement born out of the conflict between the
depressed classes and upper castes.
Topic Name: Modern History
(e) None of the above/More than one of the above 1888 म िशवराि पर, एझावा नारायण गु ने ने र नदी से एक प र िलया
Answer: (d)
और इसे अ िव ुरम म िशविलंग के प म रख िदया। यह दिशत करने के
Op on (d) is correct: The Sree Narayana Guru Dharma Paripalana
(SNDP) movement was a regional movement born out of strife िलए था िक मूित समपण उ जाितयों तक सीिमत नहीं था। इसके साथ, उ ोंने

between the lower and upper castes. It was founded by Sree


एक आं दोलन की शु आत की िजसने केरल के समाज म कई कार के
Narayana Guru Swamy (1856-1928) among the Ezhavas of Kerala. The
पूवा हों को गायब होते दे खा। ात किव कुमारन असन नारायण गु के
Ezhavas of Kerala are a disadvantaged caste of toddy-tappers who
were deemed untouchables and were forbidden access to educa on अनुयायी के प म आं दोलन (अ िव ुरम आं दोलन) म शािमल ए।
and temples. The Ezhavas were Kerala's most populous caste,
अ िव ुरम े योगम की थापना 1889 म गरीबों की सहायता के िलए एक
accoun ng for 26% of the state's total popula on.
On Sivaratri in 1888, Narayana Guru, an Ezhava, took a stone from the बड़े संगठन के प म िवकिसत होने के इरादे से की गई थी।

Neyyar river and placed it as a Sivalinga at Aruvippuram. It was meant


to demonstrate that idol dedica on was not limited to the upper Q60. In modern Indian history, Begum Rokeya Sakhawat Hossain is
castes. With this, he sparked a movement that saw many forms of well known for:
prejudice in Kerala's society vanish. The eminent poet Kumaran Asan (a) reinterpreta on of verses from the Koran to argue for women’s
joined the movement (Aruvippuram movement) as a follower of educa on
Narayana Guru. The Aruvippuram Kshetra Yogam was founded in 1889 (b) star ng schools for Muslim girls in Gaya and Calcu a
with the inten on of growing into a large organisa on to aid the poor. (c) publica on of the book, Stripurushtulna based on the social

59. िन िल खत म से कौन सा ी नारायण गु धम प रपालन (एसएनडीपी) differences between men and women.
(d) the founda on of a widows’ home at Poona to provide shelter to
आं दोलन की िवशेषता है ?
widows
(a) इसने िहं दू धािमक सं थानों के उिचत बंधन की शु आत करने की मां ग (e) None of the above/More than one of the above

की। Answer: (e)


Op on (e) is correct: Muslim women, such as the Begums of Bhopal,
(b) यह सामािजक-धािमक मामलों म यथा थित के संर ण के िलए खड़ा था।
have played a significant role in encouraging women's educa on since
(c) यह उन मिहलाओं की दे खभाल करने म िविश है िजनका शोषण िकया the early twen eth century. In Aligarh, they established a primary
school for females.
गया था और िफर समाज ारा ाग िदया गया था।
In Patna and Calcu a, Begum Rokeya Sakhawat Hossain established
(d) यह दिलत वग और उ जाितयों के बीच संघष से पैदा आ एक े ीय Muslim girls' schools. She was a feisty opponent of conserva ve

आं दोलन था। ideals, claiming that religious leaders of all faiths treated women as
second-class ci zens.
(e) उपरो म से कोई नहीं/उपरो म से एक से अिधक
60.आधुिनक भारतीय इितहास म, बेगम कैया सखावत सैन को िन िल खत
उ र: (d)
के िलए जाना जाता है :
िवक (d) सही है :
(a) मिहलाओं की िश ा के िलए बहस करने के िलए कुरान से छं दों की
ी नारायण गु धम प रपालन (एसएनडीपी) आं दोलन िनचली और उ
पुन ा ा
जाितयों के बीच संघष से पैदा आ एक े ीय आं दोलन था। इसकी थापना
(b) गया और कलक ा म मु म लड़िकयों के िलए ू ल शु करना
केरल के एझावाओं के बीच ी नारायण गु ामी (1856-1928) ने की थी।
(c) पु षों और मिहलाओं के बीच सामािजक अंतर पर आधा रत पु क,
केरल के एझावा ताड़ी िनकालने वालों की एक वंिचत जाित ह िज अछूत
ीपु षतुलना का काशन।
समझा जाता था और िश ा और मंिदरों म जाने की मनाही थी। एझावा केरल
(d) िवधवाओं को आ य दान करने के िलए पूना म एक िवधवा के घर की नींव
की सबसे अिधक आबादी वाली जाित थी, जो रा की कुल आबादी का 26%
(e) उपरो म से कोई नहीं/उपरो म से एक से अिधक
िह ा है ।
उ र: (e)
Topic Name: Modern History
िवक (e) सही है : Q62. Malhar Joshi founded the Social Service League in Bombay with
the aim-
बीसवीं सदी की शु आत से ही मु म मिहलाओं, जैसे भोपाल की बेगमों ने
(a) To secure for the masses be er and reasonable condi ons of life
मिहलाओं की िश ा को ो ािहत करने म मह पूण भूिमका िनभाई है । and work
(b) To train na onal missionaries for the service of India
अलीगढ़ म, उ ोंने मिहलाओं के िलए एक ाथिमक िव ालय की थापना की।
(c) To prepare a cadre of selfless workers
बेगम कैया सखावत सैन ने पटना और कलक ा म मु म लड़िकयों के
(d) To spread educa on among women and lower caste people
ू लों की थापना की। वह िढ़वादी आदश की बल िवरोधी थीं, उनका (e) None of the above/More than one of the above
Answer: (a)
दावा था िक सभी धम के धािमक नेता मिहलाओं को दू सरे दज के नाग रक के
Op on (a) is correct: Narayan Malhar Joshi, a devotee of Gopal
प म मानते ह। Krishna Gokhale, founded the Social Service League in Bombay with
Q61. Which one of the following sets of personali es were the goal of ensuring be er and more acceptable living and working
associated with the Jus ce Movement? condi ons for the masses. Many schools, libraries, reading rooms, day
(a) E.V. Ramaswamy Naicker, Vokkaliga Sangha, Sree Narayana Guru nurseries, and coopera ve organisa ons were established because of
(b) Radhakanta Deb, Shiv Narayan Agnihotri, Behramji Malabari their efforts. Police court agents' work, legal help, and counsel to the
(c) Narayan Malhar Joshi, Gopal Krishna Gokhale, Gopal Ganesh poor and uneducated, slum dweller excursions, gymnasium and
Agarkar theatrical performance facili es, sanitary work, medical relief, and
(d) C.N. Mudaliar, T.M. Nair, P. Tyagaraja boys' clubs and scout corps were among their other opera ons. Joshi
(e) None of the above/More than one of the above also established the All India Trade Union Congress (1920).
Answer: (d) 62. म ार जोशी ने बंबई म सोशल सिवस लीग की थापना इस उ े से
Op on (d) is correct: C.N. Mudaliar, T.M. Nair, and P. Tyagaraja
की थी-
founded the Jus ce Movement in the Madras Presidency. It was
aimed to secure jobs and representa on for non-brahmins in the (a) जनता के िलए जीवन और काय की बेहतर और उिचत थित सुिनि त

legislature. The Madras Presidency Associa on was founded in 1917 करना


to demand separate representa on in the assembly for the lower
(b) भारत की सेवा के िलए रा ीय िमशन रयों को िशि त करने के िलए
castes.

61. िन िल खत म से कौन सा ाय आं दोलन से जुड़ा था? (c) िनः ाथ कायकताओं का संवग तैयार करना

(a) ई.वी. रामा ामी नायकर, वो ािलगा संघ, ी नारायण गु (d) मिहलाओं और िनचली जाित के लोगों म िश ा का सार करना

(b) राधाकां त दे ब, िशव नारायण अि हो ी, बेहरामजी मालाबारी (e) उपरो म से कोई नहीं/उपरो म से एक से अिधक

(c) नारायण म ार जोशी, गोपाल कृ गोखले, गोपाल गणेश आगरकर उ र: (a)

(d) सी.एन. मुदिलयार, टी.एम. नायर, पी ागराज िवक (a) सही है :

(e) उपरो म से कोई नहीं/उपरो म से एक से अिधक गोपाल कृ गोखले के भ नारायण म ार जोशी ने जनता के िलए बेहतर

उ र: (d) और अिधक ीकाय रहने और काम करने की थित सुिनि त करने के ल

िवक (d) सही है : के साथ बॉ े म सोशल सिवस लीग की थापना की। उनके यासों के कारण

सी.एन. मुदिलयार, टी.एम. नायर और पी. ागराज ने म ास ेसीडसी म ाय कई ू ल, पु कालय, वाचनालय, डे नसरी और सहकारी संगठन थािपत

आं दोलन की थापना की। इसका उ े िवधाियका म गैर- ा णों के िलए िकए गए। पुिलस अदालत एजटों के काम, कानूनी मदद, और गरीबों और

नौक रयों और ितिनिध को सुरि त करना था। म ास ेसीडसी अिशि त, झु ी-झोपिड़यों म रहने वालों के िलए परामश, ायामशाला और

एसोिसएशन की थापना 1917 म िनचली जाितयों के िलए िवधानसभा म अलग ना दशन सुिवधाएं , ता काय, िचिक ा राहत, और लड़कों के ब

ितिनिध की मां ग के िलए की गई थी। और ाउट कोर उनके अ काय म शािमल थे। जोशी ने अ खल भारतीय

टे ड यूिनयन कां ेस (1920) की भी थापना की।


Topic Name: Modern History
pushed for widows' right to remarry. The Age of Consent Act (1891)
Q63. Ramakrishna Mission was founded by Swami Vivekananda in: was enacted as a result of the reless efforts of a Parsi reformer B.M.
(a) 1867 AD Malabari. He outlawed the marriage of girls under the age of 12.
(b) 1875 AD 64. जे.ई.डी. बे ून, करसोनदास मूलजी और बी.एम. मालाबारी िन िल खत
(c) 1896 AD
म से िकससे संबंिधत था?
(d) 1879 AD
(e) None of the above/More than one of the above (a) मिहलाओं के िलए थितयों की बेहतरी के िलए काम िकया

Answer: (e)
(b) पंजाब े म रा वादी िवचारों को फैलाने म मदद की
Op on (e) is correct: A er Ramakrishna's death in 1897, Swami
(c) तं ता के बाद भारत के िलए आिथक योजना िवकिसत की
Vivekananda established the Ramakrishna Mission. The Ramakrishna
Ma h and Mission's headquarters are at Belur, near Calcu a. The (d) भारत म सा वादी िवचारों के चार म मदद की
Ramakrishna Mission emphasised that salva on is a result of selfless
(e) उपरो म से कोई नहीं/उपरो म से एक से अिधक
service and ac on.
उ र: (a)
63. रामकृ िमशन की थापना ामी िववेकानंद ने की थी:
िवक (a) सही है :
(a) 1867 ई
बे ून ू ल मिहलाओं की िश ा के िलए महान आं दोलन का पहला फल था
(b) 1875 ई
जो 1840 और 1850 के दशक म शु आ था और इसकी थापना जे.ई.डी.
(c) 1896 ई
बे ून जो 1849 म कलक ा म िश ा प रषद के अ थे। गुजराती म,
(d) 1879 ई
करसोनदास मूलजी ने िवधवा पुनिववाह को बढ़ावा दे ने के िलए 1852 म स
(e) उपरो म से कोई नहीं/उपरो म से एक से अिधक
काश की थापना की। बी.एम. मालाबारी, नमद (नमदाशंकर लाभशंकर दवे),
उ र: (e)
ायमूित गोिवंद महादे व रानाडे और के. नटराजन उन लोगों म शािमल थे
िवक (e) सही है :
िज ोंने िवधवाओं के पुनिववाह के अिधकार के िलए जोर िदया। सहमित की
1897 म रामकृ की मृ ु के बाद, ामी िववेकानंद ने रामकृ िमशन की
आयु अिधिनयम (1891) एक पारसी सुधारक बी.एम. के अथक यासों के
थापना की। रामकृ मठ और िमशन का मु ालय कलक ा के पास बेलूर
प रणाम प अिधिनयिमत िकया गया था। मालाबारी। उ ोंने 12 साल से
म है । रामकृ िमशन ने इस बात पर जोर िदया िक मो िनः ाथ सेवा और
कम उ की लड़िकयों की शादी को गैरकानूनी करार िदया।
कम का प रणाम है ।

Q64. J.E.D. Bethune, Karsondas Mulji and B.M. Malabari was


Q65. With which one of the following objec ves was the All India
associated with which of the following?
Women’s Conference (AIWC) founded?
(a) Worked for the be erment of condi ons for women
(a) To promote educa on for women
(b) Helped in spreading the na onalist ideas in the Punjab
(b) To abolish the purdah system
region
(c) To improve the educa onal syllabus of Indian women
(c) Developed the economic plan for India a er Independence
(d) To work for a society based on principles of social jus ce,
(d) Helped to propagate the communist ideas in India
integrity, equal rights and opportuni es
(e) None of the above/More than one of the above
(e) None of the above/More than one of the above
Answer: (a)
Answer: (d)
Op on (a) is correct: The Bethune School was the first fruit of the
Op on (d) is correct: Margaret Cousins launched the All India
great movement for women's educa on that erupted in the 1840s
Women's Conference (AIWC) in 1927, which was possibly the first
and 1850s and was founded by J.E.D. Bethune who was president of
women's organisa on to take an egalitarian stance. Fergusson College
the Council of Educa on in Calcu a in 1849. In Gujara , Karsondas
in Pune hosted the first conference. Maharani Chimnabai Gaekwad,
Mulji founded Satya Prakash in 1852 to promote widow remarriage.
Rani Sahiba of Sangli, Sarojini Naidu, Kamla Devi Cha opadhyaya, and
B.M. Malabari, Narmad (Narmadashankar Labhshankar Dave), Jus ce
Lady Dorab Tata were among the founding members. Its goals were to
Govind Mahadeo Ranade, and K. Natarajan were among those who
Topic Name: Modern History
fight for a society founded on principles of social jus ce, integrity, रखरखाव अिधिनयम (1956), मिहला अिधिनयम म अनैितक ापार का दमन
equal rights and opportuni es and to ensure that every human being
(1958), मातृ लाभ अिधिनयम (1961), दहे ज िनषेध अिधिनयम (1961)।
had access to the necessi es of life, which were not decided by
chance or sex but by planned social distribu on. Q66. Veeresalingam Pantulu is related to which of the following

The AIWC worked for various legisla ve reforms before and a er organiza ons?
India's independence, including the Sarda Act (1929), Hindu Women's (a) Prarthana Samaj
Right to Property Act (1937), Factory Act (1947), Hindu Marriage and (b) Brahmo Samaj
Divorce Act (1954), Special Marriage Act (1954), Hindu Minority and (c) Sadharan Brahmo Samaj
Guardianship Act (1956), Hindu Adop on and Maintenance Act (d) Arya Samaj
(1956), Suppression of Immoral Traffic in Women Act (1958), (e) None of the above/More than one of the above
Maternity Benefits Act (1961), Dowry Prohibi on Act (1961). Answer: (a)
Op on (a) is correct: Veeresalingam Pantulu was a Bri sh Indian
65। िन िल खत म से िकस उ े से अ खल भारतीय मिहला स ेलन
social reformer and writer who lived in the Madras Presidency. He is
(AIWC) की थापना की गई थी? o en regarded as the father of the Telugu Renaissance. He

(a) मिहलाओं के िलए िश ा को बढ़ावा दे ने के िलए encouraged Prarthana Samaj in south India. The earliest Prarthana
Samaj in Andhra was established in 1878 by Veeresalingam. He was a
(b) पदा था को समा करना
pioneering social reformer who advocated for women's educa on and
(c) भारतीय मिहलाओं के शैि क पा म म सुधार करने के िलए widow remarriage (which was not supported by society during his
me). He established the 'Brahmo Mandir' temple in Andhra Pradesh,
(d) सामािजक ाय, अखंडता, समान अिधकारों और अवसरों के िस ां तों के
as well as the 'Hitkarini School' in 1908. He is o en considered the
आधार पर समाज के िलए काम करना
Raja Rammohan Roy of Andhra.
(e) उपरो म से कोई नहीं/उपरो म से एक से अिधक 66. वीरे सिलंगम पंतुलु िन िल खत म से िकस संगठन से संबंिधत है ?

उ र: (d) (a) ाथना समाज

िवक (d) सही है : (b) समाज

मागरे ट किज ने 1927 म अ खल भारतीय मिहला स ेलन (एआईड ूसी) (c) साधारण समाज

का शुभारं भ िकया, जो संभवत: समतावादी ख अपनाने वाला पहला मिहला (d) आय समाज

संगठन था। पुणे म फ ूसन कॉलेज ने पहले स ेलन की मेजबानी की। (e) उपरो म से कोई नहीं/उपरो म से एक से अिधक

सं थापक सद ों म महारानी िचमनाबाई गायकवाड़, सां गली की रानी सािहबा, उ र: (a)

सरोिजनी नायडू, कमला दे वी च ोपा ाय और लेडी दोराब टाटा शािमल थीं। िवक (a) सही है :

इसका ल सामािजक ाय, अखंडता, समान अिधकारों और अवसरों के वीरे सिलंगम पंतुलु एक ि िटश भारतीय समाज सुधारक और लेखक थे जो

िस ां तों पर थािपत समाज के िलए लड़ना था और यह सुिनि त करना था िक म ास ेसीडसी म रहते थे। उ अ र तेलुगु पुनजागरण का जनक माना

ेक मनु के पास जीवन की आव कताओं तक प ं च हो, जो िक संयोग जाता है । उ ोंने दि ण भारत म ाथना समाज को ो ािहत िकया। आं म

या िलंग ारा नहीं ब िनयोिजत सामािजक िवतरण ारा तय की गई थी। सबसे पहला ाथना समाज 1878 म वीरे शिलंगम ारा थािपत िकया गया था।

AIWC ने भारत की तं ता से पहले और बाद म िविभ िवधायी सुधारों के िलए वह एक अ णी समाज सुधारक थे, िज ोंने मिहलाओं की िश ा और िवधवा

काम िकया, िजसम सारदा अिधिनयम (1929), िहं दू मिहलाओं का संपि का पुनिववाह (जो उनके समय म समाज ारा समिथत नहीं था) की वकालत की।

अिधकार अिधिनयम (1937), कारखाना अिधिनयम (1947), िहं दू िववाह और उ ोंने आं दे श म ' ो मंिदर' मंिदर की थापना की, साथ ही 1908 म

तलाक अिधिनयम (1954), िवशेष िववाह अिधिनयम ( 1954), िहं दू 'िहतका रणी ू ल' की भी थापना की। उ अ र आं का राजा राममोहन

अ सं क और संर कता अिधिनयम (1956), िहं दू द क हण और राय माना जाता है ।


Topic Name: Modern History
Q67. What was the purpose of the ‘Society for Social Reform’, which 2. Aboli on of Sa in India.
was organised by Veerasalingam Pantulu? 3. Aboli on of slavery in India.
(a) To secure tenancy rights for the peasants in Madras Presidency What is the sequence of the above events?
(b) To break the caste rules in Maharashtra (a) 2-1-3
(c) To secure the right to use a road for untouchables in Kerala (b) 3-2-1
(d) Support of widow remarriage in Madras. (c) 1-2-3
(e) None of the above/More than one of the above (d) 2-3-1
Answer: (d) (e) None of the above/More than one of the above
Op on (d) is correct: In the Telugu-speaking areas of the Madras Answer: (d)
Presidency, the reform movement in support of widow remarriage Op on (d) is correct: Slavery was abolished in Britain in 1820. Indian
was started by Veerasa-lingam Pantulu, who founded a Society for Slavery Act, also known as the Act of 1843, declared slavery to be
Social Reform for this purpose in 1878. illegal in India and all exis ng slaves were emancipated without any
The first widow remarriage in the region was officiated by him in 1881 compensa on to the slave owners. The Charter Act of 1833 instructed
in his hometown Rajahmundry, in the face of s ff opposi on. the government of India to abolish slavery.
Gradually, support for the reform increased, and in 1891 a Widow With the efforts of Ishwar Chandra Vidyasagar resulted in the passing
Remarriage Associa on was formed with the patronage of the of the Hindu Widows Remarriage Act of 1856, which legalised window
prominent ci zens of the town. marriages in India. In 1866 Vishnushastri Pandit started a society for

67. वीरसिलंगम पंतुलु ारा आयोिजत 'सोसाइटी फॉर सोशल रफॉम' का the encouragement of widow-remarriage.
The regula on of 1829 declared the prac ce of Sa illegal and
उ े ा था?
punishable by the criminal courts as culpable homicide. Raja Ram
(a) म ास ेसीडसी म िकसानों के िलए िकरायेदारी के अिधकारों को सुरि त Mohan Roy played a pivotal role in forcing the Bri sh government to

pass an act against the Sa .


करने के िलए
68. 19वीं शता ी के दौरान िकए गए सामािजक-धािमक सुधारों के संदभ म
(b) महारा म जाित के िनयमों को तोड़ने के िलए
िन िल खत पर िवचार कर:
(c) केरल म अछूतों के िलए सड़क का उपयोग करने का अिधकार सुरि त
1. िवधवा पुनिववाह को वैध बनाना।
करने के िलए
2. भारत म सती था का उ ूलन।
(d) म ास म िवधवा पुनिववाह का समथन।
3. भारत म गुलामी का उ ूलन।
(e) उपरो म से कोई नहीं/उपरो म से एक से अिधक
उपरो घटनाओं का म ा है ?
उ र: (d)
(a) 2-1-3
िवक (d) सही है :
(b) 3-2-1
म ास ेसीडसी के तेलुगु भाषी े ों म, िवधवा पुनिववाह के समथन म सुधार (c) 1-2-3
(d) 2-3-1
आं दोलन वीरासा-िलंगम पंतुलु ारा शु िकया गया था, िज ोंने 1878 म इस
(e) उपरो म से कोई नहीं/उपरो म से एक से अिधक
उ े के िलए एक समाज सुधार सोसायटी की थापना की थी।
उ र: (d)
इस े म पहला िवधवा पुनिववाह 1881 म उनके गृहनगर राजमुंदरी म कड़े
िवक (d) सही है :
िवरोध के बावजूद संप आ था। धीरे -धीरे सुधार के िलए समथन बढ़ता गया
1820 म ि टे न म दासता को समा कर िदया गया था। भारतीय दासता
और 1891 म शहर के मुख नाग रकों के संर ण म एक िवधवा पुनिववाह संघ
अिधिनयम, िजसे 1843 के अिधिनयम के प म भी जाना जाता है , ने भारत म
का गठन िकया गया।
दासता को अवैध घोिषत कर िदया और सभी मौजूदा दासों को दास मािलकों

Q68. Consider the following in the context of the socio-religious को िबना िकसी मुआवजे के मु कर िदया गया। 1833 के चाटर अिधिनयम ने

reforms taken during the 19th century: भारत सरकार को गुलामी को समा करने का िनदश िदया।
1. Legalisa on of widow remarriage.
Topic Name: Modern History
ई र चं िव ासागर के यासों के प रणाम प 1856 के िहं दू िवधवा की वकालत की और कुलीनवाद और ब िववाह का िवरोध िकया। इसने लोगों

पुनिववाह अिधिनयम को पा रत िकया गया, िजसने भारत म खड़की िववाहों को बाल िववाह को ितबंिधत करने की शपथ लेने के िलए े रत करने के िलए

को वैध कर िदया। 1866 म िव ुशा ी पंिडत ने िवधवा-पुनिववाह को बढ़ावा िस " ित ा आं दोलन" शु िकया।

दे ने के िलए एक सोसाइटी की शु आत की।


Q70. ‘Notes on Infant Marriage and Enforced Widowhood’ is
1829 के अिधिनयम ने सती था को अवैध घोिषत िकया और आपरािधक
associated with:
अदालतों ारा दं डनीय ह ा के प म दं डनीय घोिषत िकया। राजा राम मोहन (a) Surendra Nath Bannerjee

(b) R. P. Du
राय ने ि िटश सरकार को सती के खलाफ अिधिनयम पा रत करने के िलए
(c) Bahramji Malabari
मजबूर करने म मह पूण भूिमका िनभाई।
(d) Veeresalingam
(e) None of the above/More than one of the above
Q69. Indian (Na onal) Social Conference was founded by: Answer: (c)
(a) Savitribai Phule Op on (c) is correct: Bahrami Merwanji Malabari (1853–1912) was an
(b) Mahadev Govind Ranade Indian poet, publicist, author, and social reformer best known for his
(c) Raghunath Rao. ardent advocacy for the protec on of the rights of women and his
(d) Maharshi Karve ac vi es against child marriage. He is a Parsi reformer and a journalist
(e) None of the above/More than one of the above from Bombay. In August 1884, Malabari published a set of ‘Notes on
Answer: (e) Infant Marriage and Enforced Widowhood’ that he sent to 4,000
Op on (e) is correct: Indian (Na onal) Social Conference was founded leading Englishmen and Hindus. In it, Malabari deplored the ‘social
by M.G. Ranade and Raghunath Rao. It was virtually the social reform evil’ of ‘baby marriage’ and demanded a legislature to prevent it.
cell of the Indian Na onal Congress. Its first session was held in Although he stayed away from the Indian Na onal Congress as an
Madras in December 1887. The Conference met annually as a organiza on, Malabari a ended the Indian Na onal Congress in
subsidiary conven on of the Indian Na onal Congress at the same Bombay in 1885.
venue and focused a en on on social reform. The Conference 70. 'िशशु िववाह और जबरन िवधवापन पर नोट् स' संबंिधत है :
advocated marriages and opposed kulinism and polygamy. It launched
(a) सुर नाथ बनज
the famous “Pledge Movement” to inspire people to take an oath to
prohibit child marriage. (b) आर. पी. द

69. भारतीय (रा ीय) सामािजक स ेलन ारा थािपत िकया गया था: (c) बहरामजी मालाबारी

(a) सािव ीबाई फुले (d) वीरे शिलंगम

(b) महादे व गोिवंद रानाडे (e) उपरो म से कोई नहीं/उपरो म से एक से अिधक

(c) रघुनाथ राव उ र: (c)

(d) महिष कव िवक (c) सही है :

(e) उपरो म से कोई नहीं/उपरो म से एक से अिधक बहरामी मेरवानजी मालाबारी (1853-1912) एक भारतीय किव, चारक,

उ र: (e) लेखक और समाज सुधारक थे, िज मिहलाओं के अिधकारों की र ा और

िवक (e) सही है : बाल िववाह के खलाफ उनकी गितिविधयों के िलए उनकी बल वकालत के

भारतीय (रा ीय) सामािजक स ेलन की थापना एम.जी. रानाडे और रघुनाथ िलए जाना जाता है । वह एक पारसी सुधारक और बंबई के एक प कार ह।

राव। व ुतः यह भारतीय रा ीय कां ेस का सामािजक सुधार को था। अग 1884 म, मालाबारी ने 'िशशु िववाह और जबरन िवधवापन पर नोट् स'

इसका पहला स िदसंबर 1887 म म ास म आयोिजत िकया गया था। स ेलन का एक सेट कािशत िकया, िजसे उ ोंने 4,000 मुख अं ेजों और िहं दुओं को

की वािषक बैठक भारतीय रा ीय कां ेस के सहायक स ेलन के प म उसी भेजा। इसम मालाबारी ने 'बाल िववाह' की 'सामािजक बुराई' की भ ना की

थान पर ई और सामािजक सुधार पर ान कि त िकया। स ेलन ने िववाहों


Topic Name: Modern History
और इसे रोकने के िलए िवधाियका की मां ग की। य िप वह एक संगठन के प and the cultures of the empire they sought to rule. The Asia c Society
of Bengal, founded by Sir William Jones in 1784, undertook the
म भारतीय रा ीय कां ेस से दू र रहे , मालाबारी ने 1885 म बंबई म भारतीय
edi ng, prin ng and transla on of many Indian manuscripts.
रा ीय कां ेस म भाग िलया। 72. 1784 म एिशयािटक सोसाइटी ऑफ बंगाल की थापना का मु उ े

िन िल खत म से कौन सा था?
Q71. Who among the following started the Bhoodan Movement in
India? (a) भारतीय कानून म अं ेजी िसिवल सेवकों को िशि त करना।

(a) Jawaharlal Nehru (b) भारतीय भाषाओं और शा ों के अ यन को बढ़ावा दे ना।


(b) Mo lal Nehru
(c) पुराता क सव ण और उ नन करना।
(c) Gyaani Gyan Singh
(d) Acharya Vinoba Bhave (d) भारत म अं ेजी रीित- रवाजों और परं पराओं को बढ़ावा दे ना।

(e) None of the above/More than one of the above


(e) उपरो म से कोई नहीं/उपरो म से एक से अिधक
Answer: (d)
उ र: (b)
Op on (d) is correct: Acharya Vinoba Bhave started the Bhoodan
Movement in India. Vinoba asked the landlords of the village to come िवक (b) सही है :
forward and save the Harijans, and a landlord got up and offered the
औपिनवेिशक काल के दौरान, ि िटश शासकों ने भारतीय इितहास का
required land. This incident added a new chapter in the history of
sacrifices and non-violence. अ यन करना शु िकया और उपमहा ीप के बारे म ान का एक सं ह

71. िन िल खत म से िकसने भारत म भूदान आं दोलन की शु आत की थी? तैयार िकया तािक वे उस सा ा के लोगों और सं ृ ितयों को बेहतर ढं ग से

(a) जवाहरलाल नेह समझ सक, िजस पर वे शासन करना चाहते थे। 1784 म सर िविलयम जो

(b) मोतीलाल नेह ारा थािपत एिशयािटक सोसाइटी ऑफ बंगाल ने कई भारतीय पां डुिलिपयों

(c) ानी ान िसंह का संपादन, मु ण और अनुवाद िकया।

(d) आचाय िवनोबा भावे


Q73. Which of the following statements is not correct with respect
(e) उपरो म से कोई नहीं/उपरो म से एक से अिधक
to Paramhansa Mandali?
उ र: (d) (a) It was founded in 1849 in Maharashtra

िवक (d) सही है : (b) The ideology of the society was closely linked to that of the
Manav Dharma Sabha.
आचाय िवनोबा भावे ने भारत म भूदान आं दोलन की शु आत की। िवनोबा ने
(c) It believed that one God should be worshipped.
गाँ व के जमींदारों को आगे आने और ह रजनों को बचाने के िलए कहा, और एक (d) It was primarily founded by Rajnarayan Bose and A K Du .
(e) None of the above/More than one of the above
जमींदार ने उठकर आव क भूिम की पेशकश की। इस घटना ने बिलदान
Answer: (d)
और अिहं सा के इितहास म एक नया अ ाय जोड़ा। Op on (d) is not correct: Founded in 1849 in Maharashtra, the
Q72. Which one of the following was the main objec ve for se ng founders of the Paramahansa Mandali—Dadoba Pandurang, Mehtaji
up the Asia c Society of Bengal in 1784? Durgaram and others—began as a secret society that worked to
(a) Training the English Civil Servants in Indian law. reform Hindu religion and society in general. The ideology of the
(b) Promo ng the study of Indian languages and scriptures. society was closely linked to that of the Manav Dharma Sabha.
(c) Conduc ng archaeological surveys and excava ons. Besides believing that one God should be worshipped, society also
(d) Promo ng the English customs and tradi ons in India. said real religion is based on love and moral conduct. Freedom of
(e) None of the above/More than one of the above thought was encouraged, as was ra onality.
Answer: (b) The founders of the mandali were primarily interested in breaking
Op on (b) is correct: During the colonial period, Bri sh administrators caste rules. At their mee ngs, food cooked by lower caste people was
began to study Indian history and create an archive of knowledge taken by the members. These mandalis also advocated widow
about the subcon nent to help them be er understand the people
Topic Name: Modern History
remarriage and women’s educa on. Branches of Paramahansa aim was to purify Islam by removing improper prac ces and
Mandali existed in Poona, Satara, and other towns of Maharashtra. beliefs.

73. परमहं स मंडली के संबंध म िन िल खत म से कौन सा कथन सही नहीं है ? ● Nadwat-ul-ulama movement was founded by Shibli Numani
and had its presence in Lucknow. They aimed at recas ng
(a) इसकी थापना 1849 म महारा म ई थी
Muslim Educa onal System.
(b) समाज की िवचारधारा मानव धम सभा से िनकटता से जुड़ी ई थी। ● The Ahmadiyya Movement was concentrated in Punjab. It
defended Islam from a acks from missionaries etc.
(c) यह माना जाता था िक एक भगवान की पूजा की जानी चािहए।
74. िन िल खत म से कौन सा मु म धािमक आं दोलन और उनके
(d) यह मु प से राजनारायण बोस और ए के द ारा थािपत िकया गया
संबंिधत भाव े सही सुमेिलत नहीं है ?
था।
(a) वहाबी आं दोलन - पटना
(e) उपरो म से कोई नहीं/उपरो म से एक से अिधक
(b) फराइजी - बंगाल
उ र: (d)
(c) नदवत-उल-उलमा-लखनऊ
िवक (d) सही नहीं है :
(d) अहमिदया - पंजाब
महारा म 1849 म थािपत, परमहं स मंडली के सं थापक-दाडोबा पां डुरं ग,
(e) उपरो म से कोई नहीं/उपरो म से एक से अिधक
मेहताजी दु गाराम और अ -ने एक गु समाज के प म शु आत की, िजसने
उ र: (e)
सामा प से िहं दू धम और समाज म सुधार के िलए काम िकया। समाज की
िवक (e) सही है : मु म धािमक आं दोलन:
िवचारधारा मानव धम सभा से घिन प से जुड़ी ई थी। यह मानने के अलावा
● वहाबी आं दोलन का मह पूण क पटना था। इसके नेता शाह वलीउ ाह
िक एक ई र की पूजा की जानी चािहए, समाज ने यह भी कहा िक वा िवक
थे। इसका काय इ ाम को शु और पुनज िवत करना था।
धम ेम और नैितक आचरण पर आधा रत है । तकसंगतता के प म िवचार
● फ़राज़ी आं दोलन बंगाल म कि त था। हाजी शरीयत उ ाह फ़राज़ी
की तं ता को ो ािहत िकया गया था।
आं दोलन के सं थापक थे। इसका उ े अनुिचत थाओं और िव ासों को
मंडली के सं थापक मु प से जाित के िनयमों को तोड़ने म िच रखते थे।
हटाकर इ ाम को शु करना था।
उनकी बैठकों म, सद ों ारा िन जाित के लोगों ारा पकाया गया भोजन
● नदवत-उल-उलेमा आं दोलन की थापना िशबली नुमानी ने की थी और
हण िकया जाता था। इन मंडिलयों ने िवधवा पुनिववाह और मिहला िश ा की
इसकी उप थित लखनऊ म थी। उनका उ े मु म िश ा णाली को
भी वकालत की। परमहं स मंडली की शाखाएँ पूना, सतारा और महारा के
पुनगिठत करना था।
अ शहरों म मौजूद थीं।
● अहमिदया आं दोलन पंजाब म कि त था। इसने िमशन रयों आिद के हमलों

Q74. Which of the following Muslim Religious Movements and their से इ ाम की र ा की।
respec ve areas of influence is not correctly matched?
(a) Wahabi Movement - Patna Q75. Who among the following leaders believed that the
(b) Farazi - Bengal ‘Safety-Valve’ theory was behind the forma on of the Indian
(c) Nadwat-ul-ulama - Lucknow Na onal Congress?
(d) Ahmadiyya - Punjab (a) Lala Lajpat Rai
(e) None of the above/More than one of the above (b) Jawahar Lal Nehru
Answer: (e) (c) Rajendra Prasad
Op on (e) is correct: Muslim Religious Movements: (d) Kadambini Ganguly
● The important centre of the Wahhabi Movement was Patna. (e) None of the above/More than one of the above
Its leader was Shah Waliullah. Its func on was to purify and Answer: (a)
revive Islam. Op on (a) is correct: According to the 'Safety-Valve' theory, A. O.
● The Faraizi Movement was concentrated in Bengal. Haji Hume founded the Congress with the inten on of serving as a safety
Shariat Ullah was the founder of the Faraizi Movement. Its
Topic Name: Modern History
valve' for the Indians' rising dissa sfac on. For that purpose, he stance hardened even more. The government openly condemned the
persuaded Lord Dufferin not to block the Congress' establishment. The Congress, calling the na onalists "sedi ous brahmins," "disloyal
'Safety-Valve' doctrine was believed by hardline leaders such as Lala babus," and other derogatory terms. The Congress was dubbed "a
Lajpat Rai. The 'conspiracy hypotheses of Marxist historians was an factory of sedi on" by Lord Dufferin. Later, the government used a
offshoot of the safety valve' concept. R. P. Du , for example, claimed "divide and conquer" strategy against Congress.
that the Indian Na onal Congress sprang from a plot to put down a 76. िन िल खत म से िकसने भारतीय रा ीय कां ेस को "राज ोह का
popular movement in India and that the bourgeois leaders were
कारखाना" कहा था?
complicit.

75. िन िल खत म से िकस नेता का मानना था िक भारतीय रा ीय कां ेस के (a) लॉड डफ़ रन

गठन के पीछे 'से ी-वा ' िस ां त था? (b) लॉड कजन

(a) लाला लाजपत राय (c) लॉड ए न II

(b) जवाहर लाल नेह (d) लॉड लसडाउन

(c) राज साद (e) उपरो म से कोई नहीं/उपरो म से एक से अिधक

(d) काद नी गां गुली उ र: (a)

(e) उपरो म से कोई नहीं/उपरो म से एक से अिधक िवक (a) सही है :

उ र: (a) कां ेस के उदारवादी ि कोण के बावजूद, ि िटश भारतीय सरकार शु से

िवक (a) सही है : ही भारतीय रा ीय कां ेस का िवरोध कर रही थी। 1887 म कां ेस को

'से ी-वा ' िस ां त के अनुसार, ए.ओ. ह्यूम ने भारतीयों के बढ़ते असंतोष के सामािजक मामलों तक सीिमत रखने के िलए राजी करने म िवफल रहने के

िलए 'से ी वा ' के प म काम करने के इरादे से कां ेस की थापना की। बाद, जब कां ेस औपिनवेिशक शासन की लगातार आलोचना कर रही थी,

उस उ े के िलए, उ ोंने लॉड डफ़ रन को कां ेस की थापना को अव आिधका रक ख और भी स हो गया। सरकार ने खुले तौर पर कां ेस की

न करने के िलए राजी िकया। लाला लाजपत राय जैसे क र नेता 'से ी-वा ' िनंदा की, रा वािदयों को "दे श ोही ा ण," "बेवफा बाबू," और अ

िस ां त को मानते थे। मा वादी इितहासकारों की 'ष ं प रक ना सुर ा अपमानजनक श कहा। लॉड डफ रन ने कां ेस को "दे श ोह का

वा ' अवधारणा की एक शाखा थी। उदाहरण के िलए, आर.पी. द ने दावा कारखाना" करार िदया था। बाद म, सरकार ने कां ेस के खलाफ "फूट डालो

िकया िक भारतीय रा ीय कां ेस भारत म एक लोकि य आं दोलन को कुचलने और जीतो" की रणनीित का इ ेमाल िकया।

की सािजश से पैदा ई थी और बुजुआ नेताओं की िमलीभगत थी।


Q77. Which of the following statements is correct?
(a) The moderates wanted to abolish the Indian Council
Q76. Who among the following called the Indian Na onal Congress
(b) The moderates’ immediate demand was for full self-government
“a factory of sedi on”?
or democracy
(a) Lord Dufferin
(c) In 1898, the House of Commons passed a resolu on for
(b) Lord Curzon
simultaneous examina on for ICS in London and in India
(c) Lord Elgin II
(d) Jus ce Mahadev Govind Ranade started the Bombay Chronicle,
(d) Lord Lansdowne
an English-language weekly newspaper.
(e) None of the above/More than one of the above
(e) None of the above/More than one of the above
Answer: (a)
Answer: (a)
Op on (a) is correct: Despite the Congress's moderate approach, the
Op on (a) is correct: One of the most important poli cal demands of
Bri sh Indian Government was opposed to the Indian Na onal
the Moderates was to abolish the Indian Council, which prevented the
Congress from the start. A er failing to persuade the Congress to
Secretary of State from ini a ng liberal policies in India. Their
restrict itself to social ma ers in 1887, when the Congress was
immediate demand was not for full self-government or democracy;
growing increasingly cri cal of colonial administra on, the official
they demanded democra c rights only for the educated members of
Topic Name: Modern History
the Indian society, who would subs tute for the masses. In 1893, the Op on (e) is correct: Bal Gangadhar Tilak characterised the policies of
House of Commons passed a resolu on for simultaneous examina on moderate leaders as poli cal mendicancy. In the ini al phase of the
for ICS in London and in India. Pherozeshah Mehta was nominated to Congress, moderate leaders adopted methods of prayers, pe ons,
the Bombay Legisla ve Council in 1887 and, in 1893, a member of the and protests which were termed "poli cal mendicancy" by the
Imperial Legisla ve Council. He was chosen as the President of the extremist leaders.
Indian Na onal Congress in 1890. In 1910, he started the Bombay 78. िन िल खत म से िकसने ारं िभक कां ेस नेताओं ारा राजनीितक
Chronicle, an English-language weekly newspaper.
िभ ावृि के प म अपनाए गए तरीकों की आलोचना की?
77. िन िल खत कथनों म से कौन सही है ?
(a) लाला लाजपत राय
(a) उदारवादी भारतीय प रषद को ख करना चाहते थे
(b) भगत िसंह
(b) नरमपंिथयों की त ाल मां ग पूण शासन या लोकतं के िलए थी
(c) गोपाल कृ गोखले
(c) 1898 म, हाउस ऑफ कॉम ने लंदन और भारत म ICS के िलए एक साथ
(d) रामकृ परमहं स
परी ा के िलए एक ाव पा रत िकया
(e) उपरो म से कोई नहीं/उपरो म से एक से अिधक
(d) ायमूित महादे व गोिवंद रानाडे ने बॉ े ॉिनकल, एक अं ेजी भाषा का
उ र: (e)
सा ािहक समाचार प शु िकया।
िवक (e) सही है :
(e) उपरो म से कोई नहीं/उपरो म से एक से अिधक
बाल गंगाधर ितलक ने नरमपंथी नेताओं की नीितयों को राजनीितक िभखारी
उ र: (a)
बताया। कां ेस के ारं िभक चरण म, नरमपंथी नेताओं ने ाथनाओं, यािचकाओं
िवक (a) सही है :
और िवरोध के तरीकों को अपनाया, िज चरमपंथी नेताओं ारा "राजनीितक
नरमपंिथयों की सबसे मह पूण राजनीितक मां गों म से एक भारतीय प रषद
िभ ावृि " कहा गया।
को समा करना था, िजसने रा सिचव को भारत म उदार नीितयों को शु

करने से रोक िदया था। उनकी त ाल मां ग पूण शासन या लोकतं की नहीं Q79. The merger of the Indian Na onal Congress and the Indian

थी; उ ोंने केवल भारतीय समाज के िशि त सद ों के िलए लोकतां ि क Na onal Conference happened in which of the following sessions of
the Congress?
अिधकारों की मां ग की, जो जनता का िवक होगा। 1893 म, हाउस ऑफ
(a) 1885, Bombay
कॉम ने लंदन और भारत म आईसीएस के िलए एक साथ परी ा के िलए एक (b) 1886, Calcu a

(c) 1887, Madras


ाव पा रत िकया। िफ़रोज़शाह मेहता को 1887 म बॉ े लेिज ेिटव
(d) 1888, Allahabad
काउं िसल के िलए और 1893 म इं पी रयल लेिज ेिटव काउं िसल का सद
(e) None of the above/More than one of the above
नािमत िकया गया था। उ 1890 म भारतीय रा ीय कां ेस के अ के प Answer: (b)
Op on (b) is correct: One of the foremost pre-congress organisa ons
म चुना गया था। 1910 म, उ ोंने अं ेजी भाषा के सा ािहक समाचार प
to discuss na onal issues was the Indian Na onal Conference, formed
बॉ े ॉिनकल की शु आत की। by Surendranath Banerjee and Ananda Mohan Bose in 1883. It
merged with the Indian Na onal Congress at the second session of
Q78. Who among the following cri cised the methods adopted by the Congress at Calcu a in 1886, under the presidency of Dadabhai
the early Congress leaders as poli cal mendicancy? Nauroji.
(a) Lala Lajpat Rai 79. भारतीय रा ीय कां ेस और भारतीय रा ीय स ेलन का िवलय कां ेस के
(b) Bhagat Singh
िन िल खत म से िकस अिधवेशन म आ था?
(c) Gopal Krishna Gokhale
(d) Ramkrishan paramhansh (a) 1885, बॉ े

(e) None of the above/More than one of the above (b) 1886, कलक ा
Answer: (e)
(c) 1887, म ास
Topic Name: Modern History
(d) 1888, इलाहाबाद एक ापक मा ता है िक भारतीय रा ीय कां ेस (आईएनसी) के गठन के

(e) उपरो म से कोई नहीं/उपरो म से एक से अिधक पीछे का कारण सुर ा वा िस ां त था, यानी ि िटश भारतीय जनता के बीच

उ र: (b) बढ़ते असंतोष को आईएनसी के मा म से शां त करना चाहते थे। भारतीयों के

िवक (b) सही है : बीच असंतोष का कारण था वना ुलर ेस ए के पा रत होने, इ ट िबल

1883 म सुर नाथ बनज और आनंद मोहन बोस ारा गिठत भारतीय रा ीय िववाद (1883), सामा भेदभाव आिद जैसे मु ों पर, इस पृ भूिम म, INC का

स ेलन रा ीय मु ों पर चचा करने के िलए सबसे मह पूण पूव-कां ेस गठन सेवािनवृ ि िटश िसिवल सेवक ए.ओ. ह्यूम।

संगठनों म से एक था। कां ेस के दू सरे स म इसका भारतीय रा ीय कां ेस म


Q81. With respect to the a tude posed by the Bri sh Government
िवलय हो गया। 1886 म दादाभाई नौरोजी की अ ता म कलक ा म।
towards the Congress, which of the following statements is not
correct?
Q80. “To prevent another poli cal crisis like 1857, a vent was (a) The Bri sh Indian Government was hos le to the Congress from
created to channelize the discontent of Indians. For this, the re red the beginning despite the la er’s moderate methods.
Civil Servant A. O. Hume founded the Indian Na onal Congress.” This (b) The official a tude so ened a er 1887, when the Government
theory was popularly called as the: was failed in persuading the Congress to confine itself to social
(a) Safety Valve Theory issues.
(b) Drain Theory of India (c) Dufferin called the Congress ‘a factory of sedi on’.
(c) Theory of Eclipse (d) The Government adopted a ‘divide and rule’ policy toward
(d) Energy Vent Theory Congress.
(e) None of the above/More than one of the above (e) None of the above/More than one of the above
Answer: (a) Answer: (e)
Op on (a) is correct: There is a widespread belief that the reason Op on (e) is not correct: The Bri sh Indian Government was hos le
behind the forma on of Indian Na onal Congress (INC) was safety to the Congress from the beginning despite the la er’s moderate
valve theory, i.e. Bri shers wanted to pacify the rising discontent methods and emphasis on loyalty to the Bri sh Crown. The official
among Indian masses through INC. The discontent among Indians was a tude s ffened further a er 1887 when the Government failed to
due to issues like the passing of the Vernacular Press Act, the Illbert persuade the Congress to confine itself to social issues when the
bill controversy (1883), general discrimina on etc. In this background, Congress was becoming increasingly cri cal of colonial rule. Now, the
the INC was formed by re red Bri sh Civil Servant A.O. Hume. Government resorted to open condemna on of the Congress, calling
80. “1857 जैसे एक और राजनीितक संकट को रोकने के िलए, भारतीयों के the na onalists ‘sedi ous brahmins’, ‘disloyal babus’, etc. Dufferin

called the Congress ‘a factory of sedi on’. Later, the Government


असंतोष को दू र करने के िलए एक रा ा बनाया गया था। इसके िलए,
adopted a ‘divide and rule’ policy toward Congress. The officials
सेवािनवृ िसिवल सेवक ए.ओ. ह्यूम ने भारतीय रा ीय कां ेस की थापना
encouraged reac onary elements like Sir Syed Ahmed Khan and Raja
की। इस िस ां त को लोकि य प से कहा जाता था: Shiv Prasad Singh of Benaras to organise the United Indian Patrio c
Associa on to counter Congress propaganda.
(a) सुर ा वा िस ां त
81. कां ेस के ित ि िटश सरकार के रवैये के संबंध म, िन िल खत म से कौन
(b) डे न ोरी ऑफ इं िडया
सा कथन सही नहीं है ?
(c) हण का िस ां त
(a) ि िटश भारतीय सरकार कां ेस के उदारवादी तरीकों के बावजूद शु से
(d) एनज वट ोरी
ही उसके ित श ुतापूण थी।
(e) उपरो म से कोई नहीं/उपरो म से एक से अिधक
(b) 1887 के बाद आिधका रक रवैया नरम पड़ गया, जब सरकार कां ेस को
उ र: (a)
सामािजक मु ों तक ही सीिमत रखने के िलए राजी करने म िवफल रही।
िवक (a) सही है :
(c) डफ रन ने कां ेस को 'राज ोह का कारखाना' कहा।
Topic Name: Modern History
(d) सरकार ने कां ेस के ित 'फूट डालो और राज करो' की नीित अपनाई। ● Aurobindo Ghosh was in favour of extending the movement to
the rest of India. He was named principal of Bengal Na onal
(e) उपरो म से कोई नहीं/उपरो म से एक से अिधक
College, which was established in 1906 to promote patrio sm
उ र: (e) and an educa on system based on Indian condi ons and culture.
He was also the editor of Vande Mataram, and in the spirit of the
िवक (e) सही नहीं है :
Swadeshi Movement, he advocated strikes, na onal educa on,
कां ेस के उदारवादी तरीकों और ि िटश ताज के ित वफादारी पर जोर दे ने
and other ac vi es through his ar cles. Ja ndranath Bannerji
के बावजूद ि िटश भारतीय सरकार शु आत से ही कां ेस के ित श ुतापूण and Barindrakumar Ghosh assisted him (who managed the
Anushilan Sami ).
थी। 1887 के बाद आिधका रक रवैया और भी कठोर हो गया, जब सरकार
82. दे शी आं दोलन के दौरान िन िल खत म से िकसने कुछ दे शी ोर
कां ेस को सामािजक मु ों तक ही सीिमत रखने के िलए राजी करने म िवफल
थािपत िकए और र ाबंधन मनाने का आ ान िकया?
रही, जब कां ेस औपिनवेिशक शासन की लगातार आलोचना कर रही थी।
(a) सुर नाथ बनज
अब, सरकार ने रा वािदयों को 'दे श ोही ा ण', 'दे श ोही बाबु' आिद
(b) अि नी कुमार द
कहकर कां ेस की खुली िनंदा की। डफ रन ने कां ेस को 'दे श ोह का
(c) रवीं नाथ टै गोर
कारखाना' कहा। बाद म सरकार ने कां ेस के ित 'फूट डालो और राज करो'
(d) अरिबंदो घोष
की नीित अपनाई। अिधका रयों ने सर सैयद अहमद खान और बनारस के राजा
(e) उपरो म से कोई नहीं/उपरो म से एक से अिधक
िशव साद िसंह जैसे िति यावादी त ों को कां ेस के चार का मुकाबला
उ र (c)
करने के िलए यूनाइटे ड इं िडयन पैिटयोिटक एसोिसएशन को संगिठत करने के
िवक (c) सही है :
िलए ो ािहत िकया।
रवीं नाथ टै गोर ने लोगों को तं ता के िलए लड़ने के िलए े रत करने के

Q82. Who among the following set up some Swadeshi stores and िलए िविभ गीतों की रचना की और बंगाली लोक संगीत को पुनज िवत करके
called for the observance of Raksha Bandhan during the Swadeshi रा ीय गौरव को जगाया। उ ोंने कुछ दे शी ोर भी थािपत िकए और र ा
Movement?
बंधन (भाईचारे की िनशानी के प म एक-दू सरे की कलाई पर धागे बां धने की
(a) Surendranath Banerjea
(b) Ashwini Kumar Du था) को बढ़ावा िदया।
(c) Rabindranath Tagore
दे शी आं दोलन से जुड़े :
(d) Aurobindo Ghosh
(e) None of the above/More than one of the above ● नरमपंथी रा वादी सुर नाथ बनज ने द बंगाली जैसे समाचार प ों के मा म

Answer (c) से सफल ेस अिभयान चलाए और बड़ी भीड़ को संबोिधत िकया। कृ कुमार
Op on (c) is correct: Rabindranath Tagore composed various songs to
िम ा और नर कुमार सेन ने उनकी सहायता की।
inspire people to fight for freedom and resurrected Bengali folk music
to ins l na onal pride. He also established some Swadeshi stores and ● अि नी कुमार द , एक ू ल िश क, ने दे शी आं दोलन का चार करने

promoted Raksha Bandhan (the prac ce of tying threads on each


के िलए दे श बंधब सिमित की थापना की और उनके िवरोध म बा रसल के
other's wrists as a sign of brotherhood).
मु म िकसानों का नेतृ िकया।
Personali es Associated with Swadeshi Movement:
● Surendranath Banerjea, a moderate na onalist, organised ● अरिबंदो घोष आं दोलन को शेष भारत म फैलाने के प म थे। उ बंगाल
successful press campaigns and addressed large crowds through
नेशनल कॉलेज का ि ंिसपल नािमत िकया गया था, िजसे 1906 म भारतीय
newspapers like The Bengalee. Krishnakumar Mitra and Narendra
Kumar Sen assisted him. प र थितयों और सं ृ ित के आधार पर दे शभ और िश ा णाली को

● Ashwini Kumar Du , a school teacher, set up Swadesh Bandhab बढ़ावा दे ने के िलए थािपत िकया गया था। वे वंदे मातरम के संपादक भी थे
Sami to propagate the Swadeshi Movement and led the Muslim
और दे शी आं दोलन की भावना से उ ोंने अपने लेखों के मा म से हड़तालों,
peasants of Barisal in their protests.
Topic Name: Modern History
रा ीय िश ा और अ गितिविधयों की वकालत की। जितं नाथ बनज और ● मुह द शफी और फ़ज़ल-ए- सैन पंजाब म एक मु म समूह के नेता थे जो

ब रं कुमार घोष ने उनकी सहायता की (िज ोंने अनुशीलन सिमित का बंधन बिह ार के बजाय रचना क दे शी म शािमल थे।

िकया)। ● कुंवरजी मेहता और क ाणजी मेहता ने पाटीदार युवक मंडल के मा म से

संगठना क काय शु िकया।


Q83. Who among the following founded Punjab Na onal Bank and
● जोग चं ा ने छा ों को तकनीकी और औ ोिगक िश ण के िलए िवदे श
promoted Swadeshi Movement in Punjab through the
Brahmo-leaning group? जाने की सुिवधा के िलए धन जुटाने के िलए माच 1904 म एक संघ की थापना

(a) Lala Harkishan Lal


की।
(b) Muhammed Shafi
Q84. The `Swadeshi' and 'Boyco ' were adopted as methods of
(c) Kunwarji Mehta
struggle for the first me during the:
(d) Jogendra Chandra
(a) Non-Coopera on Movement in India
(e) None of the above/More than one of the above
(b) Agita on against the Par on of Pakistan
Answer: (a)
(c) Agita on against the Par on of Bengal
Op on (a) is correct: Lala Harkishan Lal promoted Swadeshi
(d) Agita on against the Jallianwala bagh massacre
Movement in Punjab through the Brahmo-leaning group, which began
(e) None of the above/More than one of the above
the Tribune newspaper and founded the Punjab Na onal Bank.
Answer: (c)
Personali es Associated with Swadeshi Movement:
Op on (c) is correct: The Swadeshi and Boyco movement began as
● Muhammed Shafi and Fazal-i-Husain were leaders of a Muslim
an agita on to oppose the Bengal par on, which later turned into a
group in Punjab involved in construc ve swadeshi rather than
mass movement throughout the country. The formal proclama on of
boyco .
the Swadeshi Movement was made on 7th August 1905, in a mee ng
● Kunwarji Mehta and Kalyanji Mehta began organisa onal work
held at the Calcu a Town Hall. In the mee ng, the famous Boyco
through the Pa dar Yuvak Mandal.
Resolu on was passed.
● Jogendra Chandra set up an associa on in March 1904 to raise
84. ' दे शी' और 'बिह ार' को संघष के तरीकों के प म पहली बार
funds to facilitate students to go abroad for technical and
industrial training. अपनाया गया था:

83. िन िल खत म से िकसने पंजाब नेशनल बक की थापना की और (a) भारत म असहयोग आं दोलन

ा ो-लीन समूह के मा म से पंजाब म दे शी आं दोलन को बढ़ावा िदया? (b) पािक ान के िवभाजन के खलाफ आं दोलन

(a) लाला हरिकशन लाल (c) बंगाल के िवभाजन के खलाफ आं दोलन

(b) मोह द शफी (d) जिलयां वाला बाग ह ाकां ड के खलाफ आं दोलन

(c) कुंवरजी मेहता (e) उपरो म से कोई नहीं/उपरो म से एक से अिधक

(d) जोग चं उ र: (c)

(e) उपरो म से कोई नहीं/उपरो म से एक से अिधक िवक (c) सही है :

उ र: (a) दे शी और बिह ार आं दोलन बंगाल िवभाजन के िवरोध म एक आं दोलन के

िवक (a) सही है : प म शु आ, जो बाद म पूरे दे श म एक जन आं दोलन म बदल गया।

लाला हरिकशन लाल ने -झुकाव वाले समूह के मा म से पंजाब म दे शी दे शी आं दोलन की औपचा रक घोषणा 7 अग 1905 को कलक ा टाउन

आं दोलन को बढ़ावा िदया, िजसने िट ून अखबार शु िकया और पंजाब हॉल म आयोिजत एक बैठक म की गई थी। बैठक म िस बिह ार ाव

नेशनल बक की थापना की। पा रत िकया गया।

दे शी आं दोलन से जुड़े :

Q85. Who was the first Indian na onalist to embrace Swaraj as the
des ny of the na on?
Topic Name: Modern History
(a) Dadabhai Naoroji government policy on tariffs, currency, and famine. Tilak was arrested
(b) Mahatma Gandhi following the murder of W.C.Rand on the basis of a poem tled
(c) Swami vivekanand "Shivaji's U erances" published in Kesari and a speech Tilak gave at
(d) Lala Lajpat Rai the Shivaji fes val, praising Shivaji's murder of Afzal Khan. The
(e) None of the above/More than one of the above prosecu on presented Tilak's defence of Shivaji's assassina on of
Answer: (e) Afzal Khan as an invita on to a ack Bri sh officers. Tilak was found
Op on (e) is correct: Bal Gangadhar Tilak was the first Indian guilty and sentenced to eighteen months in prison.
na onalist to embrace Swaraj as the des ny of the na on. Tilak 86. िन िल खत म से कौन पूना म ेग सिमित के अ की ह ा से संबंिधत
deeply opposed the then Bri sh educa on system that ignored and
था?
defamed India's culture, history, and values.

85. राज को रा की िनयित के पम ीकार करने वाला पहला भारतीय (a) चापेकर भाइयों

रा वादी कौन था? (b) सुर नाथ बनज

(a) दादाभाई नौरोजी (c) वी डी सावरकर

(b) महा ा गां धी (d) अफजल खान

(c) ामी िववेकानंद (e) उपरो म से कोई नहीं/उपरो म से एक से अिधक

(d) लाला लाजपत राय उ र (a)

(e) उपरो म से कोई नहीं/उपरो म से एक से अिधक िवक (a) सही है :

उ र: (e) 1897 म, ेग ने पूना पर आ मण िकया, और जबिक बाल गंगाधर ितलक ने

िवक (e) सही है : बीमारी से िनपटने के िलए सरकारी रणनीित का समथन िकया, अलगाव और

बाल गंगाधर ितलक राज को रा की िनयित के प म अपनाने वाले पहले घर िनरी ण जैसी ू र और कठोर तकनीकों का ापक सावजिनक िवरोध

भारतीय रा वादी थे। ितलक ने त ालीन ि िटश िश ा णाली का घोर िवरोध आ। लोकि य िव ोह के प रणाम प चापेकर बंधुओं ने पूना ेग सिमित

िकया िजसने भारत की सं ृ ित, इितहास और मू ों की उपे ा की और उ के अ की ह ा कर दी। बालकृ ह र चापेकर, वासुदेव ह र चापेकर और

बदनाम िकया। दामोदर ह र चापेकर चापेकर भाई थे। तीनों म सबसे बड़े दामोदर ह र चापेकर

Q86. Who among the following was associated with the murder of थे।
the chairman of the Plague Commi ee in Poona?
चापेकर बंधु ि िटश शासन से नाराज थे और उ ोंने भारतीयों के साथ कैसा
(a) Chapekar brothers
वहार िकया था। इस लोकि य दु नी को टै रफ, मु ा और अकाल पर
(b) Surendranath Banerjea
(c) V. D. Savarkar सरकार की नीित से भी बढ़ावा िमला। ितलक को ड ू.सी.रड की ह ा के
(d) Afzal Khan
बाद केसरी म कािशत "िशवाजी के कथन" नामक एक किवता के आधार पर
(e) None of the above/More than one of the above
Answer (a) िगर ार िकया गया था और एक भाषण ितलक ने िशवाजी उ व म िदया था,

Op on (a) is correct: In 1897, plague invaded Poona, and while Bal िजसम िशवाजी ारा अफजल खान की ह ा की शंसा की गई थी। अिभयोजन
Gangadhar Tilak backed government tac cs to combat the disease,
प ने ि िटश अिधका रयों पर हमला करने के िनमं ण के प म अफ़ज़ल
there was widespread public opposi on to cruel and harsh techniques
like segrega on and house inspec ons. The Chapekar brothers खान की िशवाजी की ह ा के ितलक के बचाव को ुत िकया। ितलक को
assassinated the chairman of the Poona Plague Commi ee as a result
दोषी पाया गया और उ अठारह महीने जेल की सजा सुनाई गई।
of the popular uprising. Balkrishna Hari Chapekar, Vasudeo Hari
Q87. The Surat Split was the spli ng of the Indian Na onal Congress
Chapekar, and Damodar Hari Chapekar were the Chapekar brothers.
into two groups - the extremists and the moderates at the Surat
The eldest of the three was Damodar Hari Chapekar.
session in 1907. Which of the following was the reason behind the
The Chapekar brothers were angered by Bri sh rule and how they
same?
treated Indians. This popular animosity was also fueled by
Topic Name: Modern History
(a) Council act of 1892 (d) The property of the villagers was seized and transferred in the
(b) Lucknow Session 1916 name of the Bri sh crown.
(c) Non -Coopera on Movement (e) None of the above/More than one of the above
(d) Boyco Movement Answer: (c)
(e) None of the above/More than one of the above Op on (c) is correct: In Kheda, Gujarat, famines and droughts were
Answer: (d) common and had destroyed the agrarian economy of the region.
Op on (d) is correct: The Indian Na onal Congress (INC) split into two Despite this, the Bri sh administra on did not reduce the taxes and
groups - the extremists and the moderates, at the Surat Session in threatened to seize the property if the taxes were not paid. Due to
1907. The split happened because extremists wanted to con nue the this, the Kheda Satyagraha was launched by Mahatma Gandhi.
Swadeshi and Boyco movement outside Bengal, while moderates 88. गुजरात म खेड़ा स ा ह शु करने के पीछे असली कारण ा था?
wanted to restrict it to Bengal. The disagreement over the spread of
(a) ि िटश शासन ने वहां थायी बंदोब शु करने का फैसला िकया।
this movement led to a split in INC at the Surat session.

87. सूरत का िवभाजन 1907 म सूरत अिधवेशन म भारतीय रा ीय कां ेस का (b) े से िनयात के कारण गुजरात की अथ व था बुरी तरह भािवत ई थी।

दो समूहों म िवभाजन था - चरमपंथी और नरमपंथी। इसके पीछे िन िल खत म (c) अकाल और सूखे के बावजूद शासन ने भू-राज को िनलंिबत नहीं िकया।

से कौन सा कारण था? (d) ामीणों की संपि को ज कर िलया गया और ि िटश ताज के नाम पर

(a) 1892 का प रषद अिधिनयम थानां त रत कर िदया गया।

(b) लखनऊ स 1916 (e) उपरो म से कोई नहीं/उपरो म से एक से अिधक

(c) असहयोग आं दोलन उ र: (c)

(d) बिह ार आं दोलन िवक (c) सही है :

(e) उपरो म से कोई नहीं/उपरो म से एक से अिधक खेड़ा, गुजरात म, अकाल और सूखा आम बात थी और इसने े की कृिष

उ र: (d) अथ व था को न कर िदया था। इसके बावजूद, ि िटश शासन ने करों को

िवक (d) सही है : कम नहीं िकया और करों का भुगतान न करने पर संपि को ज करने की

1907 म सूरत अिधवेशन म भारतीय रा ीय कां ेस (आईएनसी) दो समूहों म धमकी दी। इसी के चलते महा ा गां धी ने खेड़ा स ा ह की शु आत की थी।

िवभािजत हो गई - चरमपंथी और नरमपंथी। िवभाजन इसिलए आ ोंिक


Q89. Which of the following satyagraha/movement that led to a
चरमपंथी बंगाल के बाहर दे शी और बिह ार आं दोलन जारी रखना चाहते suspension by Mahatma Gandhi is known as the 'Himalayan

थे, जबिक उदारवादी इसे बंगाल तक ही सीिमत रखना चाहते थे। इस आं दोलन Blunder'?
(a) Boyco Movement
के सार पर असहमित के कारण सूरत अिधवेशन म कां ेस म िवभाजन हो
(b) Non-coopera on movement
गया। (c) Indigo revolt
(d) Civil disobedience movement
(e) None of the above/More than one of the above

Q88. What was the real cause behind star ng the Kheda Satyagraha Answer: (e)
in Gujarat? Op on (e) is correct: Rowla satyagraha was in response to the

(a) The Bri sh administra on decided to introduce Permanent Bri sh government enac ng the Anarchical and Revolu onary Crimes
Se lement there. Act of 1919, popularly known as the Rowla Act. Gandhiji called for a

(b) The Gujarat economy was severely affected due to the exports countrywide campaign against the Rowla Act. Deciding that Indians
from the region. were not ready to make a stand consistent with the principle of

(c) The administra on did not suspend the land revenue despite the nonviolence, an integral part of satyagraha (disobeying the Bri sh
famine and droughts. government law without violence), Gandhi suspended the resistance.
The suspension of the same is known as 'Himalayan Blunder’.
Topic Name: Modern History
89. िन िल खत स ा ह/आं दोलन म से कौन सा स ा ह/आं दोलन िजसके (b) चंपारण स ा ह

कारण महा ा गां धी को िनलंबन का सामना करना पड़ा, 'िहमालयन ंडर' के (c) रोलेट स ा ह

प म जाना जाता है ? (d) बनारस िहं दू िव िव ालय की थापना

(a) बिह ार आं दोलन (e) उपरो म से कोई नहीं/उपरो म से एक से अिधक

(b) असहयोग आं दोलन उ र: (d)

(c) इं िडगो िव ोह िवक (d) सही है :

(d) सिवनय अव ा आं दोलन बनारस िहं दू िव िव ालय की थापना वह समय था जब गां धी ने अपनी पहली

(e) उपरो म से कोई नहीं/उपरो म से एक से अिधक सावजिनक उप थित दज की थी। उनके पहले श ों ने दशकों को झकझोर

उ र: (e) कर रख िदया। 'यह हमारे िलए गहरे अपमान और शम की बात है ,' उ ोंने

िवक (e) सही है : कहा, 'िक म इस शाम को इस महान कॉलेज की छाया के नीचे, पिव शहर म,

रौलट स ा ह ि िटश सरकार ारा 1919 के अराजक और ां ितकारी इस महान कॉलेज की छाया को संबोिधत करने के िलए, पिव शहर म मजबूर

अपराध अिधिनयम को अिधिनयिमत करने के जवाब म था, िजसे रौलट ं । , अपने दे शवािसयों को उस भाषा म संबोिधत करना जो मेरे िलए िवदे शी है ।'

अिधिनयम के प म जाना जाता है । गां धीजी ने रोलेट ए के खलाफ यह एक बम िव ोट था। िकसी ने कभी अं ेजी भाषा के खलाफ बोलने की

दे श ापी अिभयान चलाने का आ ान िकया। यह िनणय लेते ए िक भारतीय िह त नहीं की थी।

अिहं सा के िस ां त के अनु प खड़े होने के िलए तैयार नहीं थे, स ा ह (िबना


Q91. Which of the following is not a theory given regarding the
िहं सा के ि िटश सरकार के कानून की अव ा) का एक अिभ अंग था, गां धी ने
suspension of the non-coopera on movement?
ितरोध को िनलंिबत कर िदया। उसी के िनलंबन को 'िहमालयन ंडर' के (a) Gandhi was afraid of the Bri sh suppression a er the Chauri

Chaura incident and hence, suspended the movement.


नाम से जाना जाता है ।
(b) Gandhi was afraid because the leadership of the movement
could have slipped from the hands of the Congress to the
Q90. In which among the following places did Gandhi make his first
peasants and hence suspended the movement.
public appearance?
(c) Differences between Gandhi and Jinnah were increasing, and
(a) Kheda Satyagraha
hence, the movement got suspended.
(b) Champaran Satyagraha
(d) The movement was one of Gandhi's first organized acts of
(c) Rowla Satyagraha
large-scale satyagraha.
(d) Establishment of Banaras Hindu University
(e) None of the above/More than one of the above
(e) None of the above/More than one of the above
Answer: (c)
Answer: (d)
Statement (a) is correct. Gandhi was afraid of the violence of Chauri
Op on (d) is correct: The establishment of Banaras Hindu University
Chaura and was afraid of Bri sh suppression. The violence of Chauri
was the me when Gandhi made his first public appearance. His first
Chaura provided an excuse for the Bri sh to crush the movement.
words froze the audience. ‘It is a ma er of deep humilia on and
Hence, Gandhi suspended the movement.
shame for us,’ he said, ‘that I am compelled this evening under the
Statement (b) is correct. The above view is given by Marxist historian
shadow of this great college, in the sacred city, to address the shadow
RP Du .
of this great college, in the sacred city, to address my countrymen in
Statement (c) is not correct: as there is no such theory given by any
language that is foreign to me.’ It was a bombshell. Nobody had ever
historians that the differences between Gandhi and Jinnah led to the
dared to speak against the English language.
suspension of the non-coopera on movement.
90. िन िल खत म से िकस थान पर गां धी ने अपनी पहली सावजिनक
Statement (d) is correct: The movement was one of Gandhi's first
उप थित दज की थी? organized acts of large-scale satyagraha.

(a) खेड़ा स ा ह
Topic Name: Modern History
91. असहयोग आं दोलन के िनलंबन के संबंध म िन िल खत म से कौन सा (a) सिवनय अव ा आं दोलन

िस ां त नहीं िदया गया है ? (b) भारत छोड़ो आं दोलन

(a) गां धी चौरी चौरा घटना के बाद ि िटश दमन से डरते थे और इसिलए, (c) रोलेट अिधिनयम

आं दोलन को थिगत कर िदया। (d) असहयोग आं दोलन

(b) गां धी डर गए थे ोंिक आं दोलन का नेतृ कां ेस के हाथों से िकसानों के (e) उपरो म से कोई नहीं/उपरो म से एक से अिधक

हाथों िफसल सकता था और इसिलए आं दोलन को िनलंिबत कर िदया गया था। उ र: (d)

(c) गां धी और िज ा के बीच मतभेद बढ़ रहे थे, और इसिलए आं दोलन िनलंिबत िवक (d) सही है :

कर िदया गया था। मोपला िव ोह 1921 म शु ए असहयोग आं दोलन की एक शाखा थी। 1921

(d) आं दोलन बड़े पैमाने पर स ा ह के गां धी के पहले संगिठत काय म से एक तक, मोपला ने मालाबार म सबसे बड़ा और सबसे तेजी से बढ़ने वाला समुदाय

था। बना िलया। दस लाख की आबादी के साथ, कुल िमलाकर मालाबार का 32%,

(e) उपरो म से कोई नहीं/उपरो म से एक से अिधक मोपला दि ण मालाबार म कि त थे।

उ र: (c) Q93. With which among the following na ons did the Bri sh sign
the Treaty of Sevres?
कथन (a) सही है । गां धी चौरी चौरा की िहं सा से डरे ए थे और ि िटश दमन से
(a) Turkey
डरते थे। चौरी-चौरा की िहं सा ने अं ेजों को आं दोलन को कुचलने का बहाना (b) France

(c) Hungary
उपल करा िदया। इसिलए, गां धी ने आं दोलन को थिगत कर िदया।
(d) None of the above
कथन (b) सही है । उपरो मा वादी इितहासकार आरपी द ारा
(e) None of the above/More than one of the above
िदया गया है । Answer: (a)
Op on (a) is correct: The Treaty of Sèvres was a 1920 treaty signed
कथन (c) सही नहीं है : ोंिक िकसी भी इितहासकार ारा ऐसा कोई िस ां त
between the Allies of World War I and the O oman Empire. The
नहीं िदया गया है िक गां धी और िज ा के बीच मतभेदों के कारण असहयोग treaty ceded large parts of the O oman territory to France, the

आं दोलन को थिगत कर िदया गया। United Kingdom, Greece and Italy, as well as created large occupa on
zones within the O oman Empire. The Bri sh signed the Treaty of
कथन (d) सही है : आं दोलन बड़े पैमाने पर स ा ह के गां धी के पहले संगिठत
Sevres with Turkey. It was in 1920 which completely dismembered
कृ ों म से एक था। Turkey. This also led to the Khilafat Movement across the world.

Q92. The Moplah rebellion was an offshoot of which, among the 93. िन िल खत म से िकस दे श के साथ अं ेजों ने सेवरे स की संिध पर
following pan India movements?
ह ा र िकए थे?
(a) Civil Disobedience movement
(b) Quit India movement (a)टक

(c) Rowla Act (b) ां स


(d) Non-Coopera on movement
(c) हं गरी
(e) None of the above/More than one of the above
Answer: (d) (d) उपयु म से कोई नहीं

Op on (d) is correct: The Moplah rebellion was an offshoot of the (e) उपरो म से कोई नहीं/उपरो म से एक से अिधक
Non-Coopera on movement that started in 1921. By 1921, the
उ र: (a)
Moplahs formed the largest and fastest-growing community in
Malabar. With a popula on of one million, 32% of that of Malabar as a िवक (a) सही है :
whole, the Moplahs were concentrated in South Malabar.
सेवस की संिध 1920 की एक संिध थी िजस पर थम िव यु के सहयोिगयों
92. िन िल खत अ खल भारतीय आं दोलनों म मोपला िव ोह िकसकी शाखा
और तुक सा ा के बीच ह ा र िकए गए थे। संिध ने ओटोमन े के बड़े
थी?
Topic Name: Modern History
िह े को ां स, यूनाइटे ड िकंगडम, ीस और इटली को सौंप िदया, साथ ही (c) Sukhdev Thapar
(d) Ashfaqullah Khan
ओटोमन सा ा के भीतर बड़े क े वाले े भी बनाए। अं ेजों ने तुक के
(e) None of the above/More than one of the above
साथ सेवस की संिध पर ह ा र िकए। यह 1920 म था िजसने तुक को पूरी Answer: (c)

Op on (c) is correct: The most important ac on of the Hindustan


तरह से िवघिटत कर िदया था। इसने दु िनया भर म खलाफत आं दोलन को भी
Republican Associa on (HRA) was the Kakori robbery. The men held
ज िदया।
up the 8-Down train at Kakori, an obscure village near Lucknow, and
Q94. Rabindranath Tagore renounced his knighthood in protest of: looted its official railway cash. Government crackdown a er the Kakori
(a) Chauri Chaura violence robbery led to arrests of many, of whom 17 were jailed, four
(b) Kakori conspiracy transported for life, and 4—Ram Prasad Bismil, Ashfaq Ullah Khan,
(c) India's par cipa on in World War I Roshan Singh, and Rajendra Lahiri—were hanged. Kakori proved to be
(d) : Jallianwala massacre a setback.
(e) None of the above/More than one of the above Determined to overcome the Kakori setback, the younger
Answer: (d) revolu onaries, inspired by socialist ideas, set out to reorganise the
Op on(d) is correct: Jallianwala massacre Hindustan Republican Associa on at a historic mee ng in the ruins of
● Rabindranath Tagore renounced his knighthood in protest of the Ferozshah Kotla in Delhi (September 1928). Under the leadership of
Jallianwala massacre. Chandrashekhar Azad, the name of HRA was changed to the
● Jallianwala massacre: A large crowd of people had gathered in a Hindustan Socialist Republican Associa on (HSRA). The par cipants
small park (Jallianwala) to protest against the arrest of their leaders included Bhagat Singh, Sukhdev Thapar (who was always known as
Saifuddin Kitchlew and Satyapal. The Army blocked the only exit point Sukhdev), Bhagwa Charan Vohra from Punjab and Bejoy Kumar
and opened fire on the unarmed crowd killing around 1000 people. Sinha, Shiv Verma, and Jaidev Kapoor from the United Provinces. The
94. रवीं नाथ टै गोर ने िकसके िवरोध म अपनी नाइट ड का ाग िकया: HSRA decided to work under collec ve leadership and adopted
socialism as its official goal.
(a) चौरी-चौरा िहं सा
95. िन िल खत म से िकसे काकोरी डकैती (1925) का म फाँ सी नहीं दी
(b) काकोरी सािजश
गई थी?
(c) थम िव यु म भारत की भागीदारी
(a) राज लािहड़ी
(d) : जिलयां वाला ह ाकां ड
(b) रोशन िसंह
(e) उपरो म से कोई नहीं/उपरो म से एक से अिधक
(c) सुखदे व थापर
उ र: (d)
(d) अशफाकउ ा खान
िवक (d) सही है :
(e) उपरो म से कोई नहीं/उपरो म से एक से अिधक
जिलयां वाला ह ाकां ड
उ र: (c)
● रवीं नाथ टै गोर ने जिलयां वाला ह ाकां ड के िवरोध म अपनी नाइट ड का
िवक (c) सही है :
ाग कर िदया।
िहं दु ान रप कन एसोिसएशन (एचआरए) की सबसे मह पूण कारवाई
● जिलयां वाला ह ाकां ड: अपने नेताओं सैफु ीन िकचलू और स पाल की
काकोरी डकैती थी। पु षों ने लखनऊ के पास एक अ ात गां व काकोरी म
िगर ारी के िवरोध म एक छोटे से पाक (जिलयां वाला) म लोगों की भारी भीड़
8-डाउन टे न को पकड़ िलया और इसकी आिधका रक रे लवे नकदी लूट ली।
जमा हो गई थी। सेना ने एकमा िनकास िबंदु को अव कर िदया और
काकोरी डकैती के बाद सरकार की कारवाई म कई लोगों को िगर ार िकया
िनह े भीड़ पर गोिलयां चला दीं िजसम लगभग 1000 लोग मारे गए।
गया, िजनम से 17 को जेल ई, चार को आजीवन कारावास, और 4- राम साद
Q95. Who among the following was not hanged in the Kakori
Robbery (1925) case? िब ल, अशफाक उ ा खान, रोशन िसंह और राज लािहड़ी को फां सी दी

(a) Rajendra Lahiri गई। काकोरी एक झटका सािबत ई।


(b) Roshan Singh
Topic Name: Modern History
काकोरी झटके से उबरने के िलए युवा ां ितका रयों ने समाजवादी िवचारों से अनुसार, क ीय िवधानमंडल म 18% सीट और ां तीय िवधानमंडल म 148 सीट

े रत होकर, िद ी म िफरोजशाह कोटला के खंडहर (िसतंबर 1928) म एक दिलत वग के िलए आरि त की जानी थीं।

ऐितहािसक बैठक म िहं दु ान रप कन एसोिसएशन को पुनगिठत करने के Q97. In which one of the following Movements, ‘Dandi March’
became a symbol of protest against the Bri sh?
िलए िनधा रत िकया। चं शेखर आज़ाद के नेतृ म HRA का नाम बदलकर
(a) Non-Coopera on Movement
िह दु ान सोशिल रप कन एसोिसएशन (HSRA) कर िदया गया। (b) Swadeshi Movement

(c) Quit India Movement


ितभािगयों म भगत िसंह, सुखदे व थापर (जो हमेशा सुखदे व के नाम से जाने
(d) Civil Disobedience Movement
जाते थे), पंजाब से भगवती चरण वोहरा और संयु ां त से िबजॉय कुमार
(e) None of the above/More than one of the above
िस ा, िशव वमा और जयदे व कपूर शािमल थे। एचएसआरए ने सामूिहक Answer: (d)
Op on (d) is correct: In the Civil Disobedience Movement, the 'Dandi
नेतृ के तहत काम करने का फैसला िकया और समाजवाद को अपने
March' became a symbol of protest against the Bri sh. On March 12,
आिधका रक ल के प म अपनाया। the famous Dandi march, which marked the beginning of the Civil
Q96. What benefits did the 'Poona Pact' offer to the depressed Disobedience Movement, started, and on April 6, 1930, Gandhi
classes? disobeyed the salt ban by picking up a lump of salt at Dandi. The
(a) 67 seats in the central legislature. breaking of the law was interpreted as a sign of the Indian people's
(b) 148 seats in the provincial legislature. will to live free of Bri sh-made laws and, as a result, of Bri sh
(c) 18% seats in the central legislature and 148 seats in the domina on. Gandhi ac vely encouraged people to produce salt from
provincial legislature. saltwater in their own houses, therefore breaking the salt prohibi on.
(d) Reserva on for depressed classes in the ICS exam. 97. िन िल खत म से िकस आं दोलन म 'दां डी माच' अं ेजों के खलाफ िवरोध
(e) None of the above/More than one of the above
का तीक बना?
Answer: (e)
Op on (e) is correct: The Poona Pact was an agreement between (a) असहयोग आं दोलन
Mahatma Gandhi and Dr Ambedkar on behalf of depressed classes (b) दे शी आं दोलन
and upper-caste Hindu leaders on the reserva on of electoral seats
(c) भारत छोड़ो आं दोलन
for the depressed classes in the legislature of the Bri sh India
government in 1930. According to the pact, 18% of seats in the central (d) सिवनय अव ा आं दोलन
legislature and 148 seats in the provincial legislature were to be
(e) उपरो म से कोई नहीं/उपरो म से एक से अिधक
reserved for depressed classes.
उ र: (d)
96. 'पूना पै ' से दिलत वग को ा लाभ आ?
िवक (d) सही है :
(a) क ीय िवधानमंडल म 67 सीट।
सिवनय अव ा आं दोलन म, 'दां डी माच' अं ेजों के खलाफ िवरोध का तीक
(b) ां तीय िवधानमंडल म 148 सीट।
बन गया। 12 माच को, सिवनय अव ा आं दोलन की शु आत को िचि त करने
(c) क ीय िवधानमंडल म 18% सीट और ां तीय िवधानमंडल म 148 सीट।
वाला िस दां डी माच शु आ और 6 अ ैल, 1930 को गां धी ने दां डी म
(d) आईसीएस परी ा म उदास वग के िलए आर ण।
नमक की एक गां ठ उठाकर नमक ितबंध की अव ा की। कानून को तोड़ने
(e) उपरो म से कोई नहीं/उपरो म से एक से अिधक
की ा ा भारतीय लोगों की ि िटश िनिमत कानूनों से मु रहने की इ ा के
उ र: (e)
संकेत के प म की गई थी और इसके प रणाम प ि िटश शासन था।
िवक (e) सही है :
गां धी ने सि य प से लोगों को अपने घरों म खारे पानी से नमक बनाने के िलए
पूना पै 1930 म ि िटश भारत सरकार की िवधाियका म दिलत वग के िलए
ो ािहत िकया, इसिलए नमक िनषेध को तोड़ िदया।
चुनावी सीटों के आर ण पर दिलत वग और उ जाित के िहं दू नेताओं की
Q98. During which one of the following movements the
ओर से महा ा गां धी और डॉ अ ेडकर के बीच एक समझौता था। संिध के
military-style camps ‘Sibirams’ were set up in the Andhra region?
Topic Name: Modern History
(a) Non-Coopera on Movement Op on (a) is correct: On August 16, 1932, Bri sh Prime Minister
(b) Swadeshi Movement Ramsay MacDonald introduced the Communal Award. The Communal
(c) Champaran Satyagraha Award established dis nct electorates and reserved seats for
(d) Civil Disobedience Movement minori es, especially the depressed classes, who were allocated
(e) None of the above/More than one of the above seventy-eight reserved seats based on the conclusions of the Indian
Answer: (d) Franchise Commi ee (also known as the Lothian Commi ee). For
Op on (d) is correct: District salt marches were organised in the several seats in Bombay, this award provided dis nct electorates for
Andhra (east and West Godavari, Krishna and Guntur) area during the Muslims, Europeans, Sikhs, Indian Chris ans, Anglo-Indians, the poor,
Civil Disobedience Movement. The Salt Satyagraha's headquarters and even the Marathas. The award was seen by na onal leaders
were established in a number of Sibirams (military-style camps). headed by Congress as another instance of Britain's divide-and-rule
Merchants gave to Congress finances, while Kamma and Raju policy.
cul vators, the domina ng castes, opposed puni ve tac cs. However, 99. लोिथयन सिमित संबंिधत थी:
there was no widespread support for the movement in the region as
(a) सां दाियक पुर ार
there was during the Non-Coopera on Movement (1921-22).

98. िन िल खत म से िकस आ ोलन के दौरान आ े म सै -शैली के (b) भारत सरकार अिधिनयम, 1919

िशिवर 'साइिबराम' थािपत िकए गए थे? (c) भारत सरकार अिधिनयम, 1935

(a) असहयोग आं दोलन (d) सिवनय अव ा आं दोलन

(b) दे शी आं दोलन (e) उपरो म से कोई नहीं/उपरो म से एक से अिधक

(c) चंपारण स ा ह उ र: (a)

(d) सिवनय अव ा आं दोलन िवक (a) सही है :

(e) उपरो म से कोई नहीं/उपरो म से एक से अिधक 16 अग , 1932 को ि िटश धान मं ी रामसे मैकडोना ने सां दाियक

उ र: (d) पुर ार की शु आत की। सां दाियक पुर ार ने अ सं कों के िलए

िवक (d) सही है : अलग िनवाचक मंडल और आरि त सीटों की थापना की, िवशेष प से

सिवनय अव ा आं दोलन के दौरान आं (पूव और पि म गोदावरी, कृ ा और दिलत वग, िज भारतीय मतािधकार सिमित (िजसे लोिथयन सिमित के पम

गुंटूर) े म िजला नमक माच आयोिजत िकए गए थे। नमक स ा ह का भी जाना जाता है ) के िन ष के आधार पर अठह र आरि त सीट आवंिटत

मु ालय कई िसिबराम (सै शैली के िशिवरों) म थािपत िकया गया था। की गईं। बंबई म कई सीटों के िलए, इस पुर ार ने मुसलमानों, यूरोपीय, िसख,

ापा रयों ने कां ेस के िव को िदया, जबिक क ा और राजू कृषकों, वच भारतीय ईसाई, एं ो-इं िडयन, गरीबों और यहां तक िक मराठों के िलए अलग

वाली जाितयों ने दं डा क रणनीित का िवरोध िकया। हालाँ िक, े म आं दोलन मतदाता दान िकए। इस पुर ार को कां ेस के नेतृ वाले रा ीय नेताओं ने

के िलए कोई ापक समथन नहीं था जैसा िक असहयोग आं दोलन (1921-22) ि टे न की फूट डालो और शासन करो नीित के एक अ उदाहरण के पम

के दौरान आ था। दे खा।

Q100. Who among the following was the viceroy of India when the

Q99. The Lothian Commi ee was associated with the: Round Table Conferences were held?

(a) Communal Award (a) Lord Irwin

(b) Government of India Act, 1919 (b) Lord Linlithgow

(c) Government of India Act, 1935 (c) Lord Lawrence

(d) Civil Disobedience Movement (d) None of the above

(e) None of the above/More than one of the above (e) None of the above/More than one of the above

Answer: (a) Answer: (a)


Topic Name: Modern History
Op on (a) is correct: Lord Irwin was the viceroy of India when the 101. 1930 म, ऐितहािसक 'दां डी माच' के पूरा होने के बाद महा ा गां धी ने
Round Table Conferences were held. Lord Irwin held office from 1926
औपचा रक प से मु ी भर नमक चुनकर सिवनय अव ा आं दोलन शु
to 1931.

100. िन िल खत म से कौन भारत का वायसराय था जब गोलमेज स ेलन िकया:

आयोिजत िकया गया था? (a) सेवा ाम

(a) लॉड इरिवन (b) दां डी

(b) लॉड िलनिलथगो (c) साबरमती

(c) लॉड लॉरस (d) वधा

(d) उपयु म से कोई नहीं (e) उपरो म से कोई नहीं/उपरो म से एक से अिधक

(e) उपरो म से कोई नहीं/उपरो म से एक से अिधक उ र: (c)

उ र: (a) िवक (c) सही है :

िवक (a) सही है : मोहनदास करमचंद गां धी ने 6 अ ैल, 1930 को साबरमती आ म से दां डी तक

लॉड इरिवन भारत के वायसराय थे जब गोलमेज स ेलन आयोिजत िकए गए ऐितहािसक 'दां डी माच' पूरा होने के बाद मु ी भर नमक चुनकर औपचा रक

थे। लॉड इरिवन ने 1926 से 1931 तक कायालय संभाला। प से सिवनय अव ा आं दोलन शु िकया, इस कार लगाए गए नमक

Q101. In 1930, Mahatma Gandhi formally launched the Civil कानून को तोड़ िदया। सरकार की ओर से। वह आं दोलन के पीछे मुख श
Disobedience Movement by picking a handful of salt a er the
थे और उ ोंने तं ता सं ाम म जमीनी र पर भागीदारी को े रत िकया।
comple on of the historic ‘Dandi March’ from:
सिवनय अव ा आं दोलन की शु आत को िचि त करने वाला ऐितहािसक माच
(a) Sevagram
(b) Dandi 12 माच को शु आ और गां धी ने 6 अ ैल को दां डी म नमक की एक गां ठ
(c) Sabarma
उठाकर नमक कानून तोड़ा। कानून के उ ंघन को भारतीय तीक के पम
(d) Wardha
(e) None of the above/More than one of the above दे खा गया। लोगों का ि िटश-िनिमत कानूनों के तहत न जीने का संक और

Answer: (c) इसिलए, ि िटश शासन के अधीन। गां धी ने खुलेआम लोगों से अपने घरों म
Op on (c) is correct: Mohandas Karamchand Gandhi formally
समु ी जल से नमक बनाने को कहा और नमक कानून का उ ंघन िकया।
launched the Civil Disobedience Movement on April 6, 1930, by
picking a handful of salt a er the comple on of the historic ‘Dandi माच, इसकी गित और लोगों पर इसके भाव को समाचार प ों म अ ी तरह
March’ from Sabarma Ashram to Dandi, thus breaking the salt law
से कवर िकया गया था। गां धी की अपील के जवाब म गुजरात म 300 ाम
imposed by the Government. He was the major force behind the
अिधका रयों ने इ ीफा दे िदया। कां ेस कायकताओं ने जमीनी र के
movement and inspired grassroots par cipa on in the freedom
struggle. संगठना क काय म खुद को लगा िलया।
The historic march, marking the launch of the Civil Disobedience
Movement, began on March 12, and Gandhi broke the salt law by Q102. The Congress Ministries resigned from office in November
picking up a lump of salt at Dandi on April 6. The viola on of the law 1939 due to:
was seen as a symbol of the Indian people’s resolve not to live under (a) Failure of Government to manage Infla on.
Bri sh-made laws and, therefore, under Bri sh rule. Gandhi openly (b) Viceroy, on its own, had made India a par cipant in the
asked the people to make salt from seawater in their homes and imperialist war without consul ng Congress.
violated the salt law. The march, its progress and its impact on the (c) Communal riots broke out in Madras Province.
people were well covered in newspapers. In Gujarat, 300 village (d) Non-Coopera on of Muslim League in administra on.
officials resigned in answer to Gandhi’s appeal. Congress workers (e) None of the above/More than one of the above
engaged themselves in grassroots-level organisa onal tasks. Answer: (b)
Topic Name: Modern History
Op on (b) is correct: In November 1939, the Congress Ministries (d) गां धीजी की िगर ारी का िवरोध करना
resigned, claiming that the Viceroy had unilaterally rendered India a
(e) उपरो म से कोई नहीं/उपरो म से एक से अिधक
par cipant in the imperialist war without consul ng the Congress.
This was called "deliverance day" by the Muslim League under Jinnah. उ र: (e)

िवक (e) सही है : अग ाव को अ ीकार करने के बाद महा ा गां धी


102. नवंबर 1939 म कां ेस मंि म लों ने कायालय से इ ीफा दे िदया,
के नेतृ म गत स ा ह शु िकया गया था। यह भारत के ित ि िटश
इसके कारण:
सरकार की नीितयों के खलाफ एक सां केितक िवरोध था। यह 17 अ ू बर,
(a) मु ा ीित का बंधन करने म सरकार की िवफलता।
1940 को शु आ और िवनोबा भावे पहले स ा ही बने, िजसके बाद
(b) वायसराय ने कां ेस से परामश िकए िबना अपने दम पर भारत को
जवाहरलाल नेह आए। इस स ा ह के दौरान गां धी जी ने 'सव दय' श का
सा ा वादी यु म भागीदार बनाया था।
योग िकया था।
(c) म ास ां त म सा दाियक दं गे भड़क उठे ।
Q104. Consider the following statements:
(d) शासन म मु म लीग का असहयोग। 1. Congress demanded dominion status for India for par cipa on
in the second world war.
(e) उपरो म से कोई नहीं/उपरो म से एक से अिधक
2. India was dragged into the war without the consent of congress
उ र: (b) by the viceroy Lord Linlithgow.

िवक (b) सही है : 3. India’s inclusion in the war led to the forma on of different
ministries.
नवंबर 1939 म, कां ेस मं ालयों ने यह दावा करते ए इ ीफा दे िदया िक
4. Winston Churchill was the Bri sh Prime Minister during World
वायसराय ने कां ेस से परामश िकए िबना भारत को एकतरफा प से War II.
Which of the above statements are correct?
सा ा वादी यु म भागीदार बना िदया था। इसे िज ा के अधीन मु म लीग
(a) 1 and 3
ारा "मु िदवस" कहा जाता था।
(b) 2 and 4
Q103. The Individual Satyagraha (1940) was ini ated: (c) 3 and 4
(a) to protest against the par cipa on in WWII (d) 1, 2, and 3
(b) as a reac on against the Kakori conspiracy (e) None of the above/More than one of the above
(c) as a moral protest against the Bri sh Policies a er the Cripps Answer: (b)
Mission Op on (b) is correct: Congress demanded complete independence for
(d) to protest against Gandhiji’s arrest par cipa on in the second world war and not just dominion status.
(e) None of the above/More than one of the above Lord Linlithgow announced India’s par cipa on without Congress’
Answer: (e) consent which led to the resigna on of all Congress provincial
Op on (e) is correct: Individual Satyagraha was started under the cabinets. Winston Churchill was the Bri sh Prime Minister during
leadership of Mahatma Gandhi a er rejec ng the August offer. It was World War II (1939-1945).
a symbolic protest against the policies of the Bri sh Government 104. िन िल खत कथनों पर िवचार कर:
toward India. It started on October 17, 1940, and Vinoba Bhave
1. कां ेस ने ि तीय िव यु म भाग लेने के िलए भारत को अिधरा का दजा
became the first satyagrahi, followed by Jawaharlal Nehru. The word
‘Sarvodaya’ was used by Gandhiji during this satyagraha. दे ने की मां ग की।

103. गत स ा ह (1940) शु िकया गया था: 2. वायसराय लॉड िलनिलथगो ारा कां ेस की सहमित के िबना भारत को यु

(a) WWII म भाग लेने के खलाफ िवरोध करने के िलए म घसीटा गया।

(b) काकोरी सािजश के खलाफ िति या के पम 3. यु म भारत के शािमल होने से िविभ मं ालयों का गठन आ।

(c) ि िमशन के बाद ि िटश नीितयों के खलाफ एक नैितक िवरोध के प 4. ि तीय िव यु के दौरान िवं न चिचल ि टे न के धानमं ी थे।

म उपरो कथनों म से कौन से सही ह?


Topic Name: Modern History
(a) 1 और 3 उ र: (c)

(b) 2 और 4 िवक (c) सही है :

(c) 3 और 4 रोलेट ए ( ैक ए ) 10 माच, 1919 को पा रत िकया गया था, जो सरकार

(d) 1, 2, और 3 को राज ोह गितिविधयों से जुड़े िकसी भी को िबना मुकदमे के कैद या

(e) उपरो म से कोई नहीं/उपरो म से एक से अिधक कैद करने के िलए अिधकृत करता है । इससे दे श ापी अशां ित फैल गई। रौलट

उ र: (b) ए के िवरोध म गां धी ने स ा ह शु िकया।

िवक (b) सही है : कां ेस ने ि तीय िव यु म भाग लेने के िलए पूण तं ता 7 अ ैल, 1919 को गां धी ने रौलट ए का िवरोध करने के तरीकों का वणन

की मां ग की थी न िक केवल डोिमिनयन े टस की। करते ए स ा ही नामक एक लेख कािशत िकया। ि िटश अिधका रयों ने

लॉड िलनिलथगो ने कां ेस की सहमित के िबना भारत की भागीदारी की घोषणा गां धी और स ा ह म भाग लेने वाले अ नेताओं के खलाफ की जाने वाली

की िजसके कारण कां ेस के सभी ां तीय मंि मंडलों ने इ ीफा दे िदया। कारवाई पर आपस म चचा की। गां धी को पंजाब म वेश करने से ितबंिधत

िवं न चिचल ि तीय िव यु (1939-1945) के दौरान ि िटश धान मं ी थे। करने और आदे शों की अवहे लना करने पर उ िगर ार करने के आदे श

Q105.During the Indian Freedom Struggle, why did the Rowla Act जारी िकए गए।
arouse popular indigna on?
(a) It curtained the freedom of religion.
Q106. In reference to the modern history of India, the ‘C.
(b) It suppressed the Indian tradi onal educa on.
Rajagopalachari Formula’ was aimed:
(c) It authorised the Government to imprison people without trial.
(a) To promote Congress-Muslim League coopera on.
(d) It curbed the trade union ac vi es.
(b) To regulate the proceedings of the Congress Working
(e) None of the above/More than one of the above
Commi ee.
Answer: (c)
(c) League to cooperate with Congress in forming a Provisional
Op on (c) is correct: The Rowla Act (Black Act) was passed on March
Government at the centre.
10, 1919, authorising the Government to imprison or confine, without
(d) To readdress the peasant grievances.
a trial, any person associated with sedi ous ac vi es. This led to
(e) None of the above/More than one of the above
na onwide unrest. Gandhi ini ated Satyagraha to protest against the
Answer: (e)
Rowla Act.
Op on (e) is correct: In 1944, senior Congress leader C.
On April 7, 1919, Gandhi published an ar cle called Satyagrahi,
Rajagopalachari (CR) devised a formula for coopera on between the
describing ways to oppose the Rowla Act. The Bri sh authori es
Congress and the Muslim League. The League's demand for Pakistan
discussed amongst themselves the ac ons to be taken against Gandhi
was implicitly accepted. Gandhi was in favour of the formula.
and any other leaders who were par cipa ng in the Satyagraha.
The main points in the CR Plan were:
Orders were issued to prohibit Gandhi from entering Punjab and
● Muslim League to support Congress's demand for independence.
arrest him if he disobeyed the orders.
● League to cooperate with Congress in forming a Provisional
105.भारतीय तं ता सं ाम के दौरान, रोलेट ए ने लोकि य आ ोश ों Government at the centre.

जगाया? ● Following the war's conclusion, the whole popula on of


Muslim-majority districts in North-West and North-East India will
(a) इसने धम की तं ता पर पदा डाला।
vote in a plebiscite on whether or not to create a separate
(b) इसने भारतीय पारं प रक िश ा को दबा िदया। sovereign state.
● In the case of par on, an agreement was to be made jointly to
(c) इसने सरकार को िबना मुकदमे के लोगों को कैद करने के िलए अिधकृत
safeguard the defence, commerce and communica ons.
िकया।
● The above terms were to be opera ve only if England transferred
(d) इसने टे ड यूिनयन गितिविधयों पर अंकुश लगाया। full powers to India.

(e) उपरो म से कोई नहीं/उपरो म से एक से अिधक


Topic Name: Modern History
106. भारत के आधुिनक इितहास के संदभ म, 'सी। राजगोपालाचारी फॉमूला' ● All members of the Execu ve Council were to be Indians
except the Governor-General and the Commander-in-Chief.
का उ े था:
● Caste Muslims and Hindus were to have equal
(a) कां ेस-मु म लीग सहयोग को बढ़ावा दे ने के िलए। representa on.
● The reconstructed Council was to func on within the
(b) कां ेस काय सिमित की कायवाही को िविनयिमत करने के िलए।
framework of the 1935 Act as an interim Government (i.e.,
(c) लीग क म अ थायी सरकार बनाने म कां ेस को सहयोग करे गी।
not responsible to the Central Assembly).
(d) िकसान िशकायतों को दू र करने के िलए। ● The Governor-General was to exercise his veto on the advice
of Ministers.
(e) उपरो म से कोई नहीं/उपरो म से एक से अिधक
● Representa ves from various par es were to submit a joint
उ र: (e) list to the Viceroy for Execu ve Council nomina ons.

िवक (e) सही है : Separate lists were to be submi ed if a combined list could
not be made.
1944 म, कां ेस के व र नेता सी. राजगोपालाचारी (सीआर) ने कां ेस और
● Possibili es for discussions on a new Cons tu on were to
मु म लीग के बीच सहयोग के िलए एक सू तैयार िकया। पािक ान के िलए be kept open once the war was over.

लीग की मां ग को िनिववाद प से ीकार कर िलया गया। गां धी सू के प म 107. भारत म 'वेवेल योजना' िकसके शासनकाल म शु की गई थी:

थे। (a) िवं न चिचल

सीआर योजना म मु िबंदु थे: (b) ेमट एटली

● मु म लीग तं ता के िलए कां ेस की मां ग का समथन करे गी। (c) नेिवल चे रलेन

● लीग क म अ थायी सरकार बनाने म कां ेस को सहयोग करे गी। (d) े नली बा िवन

● यु की समा के बाद, उ र-पि म और उ र-पूव भारत म मु म ब ल (e) उपरो म से कोई नहीं/उपरो म से एक से अिधक

िजलों की पूरी आबादी एक अलग सं भु रा बनाने या न बनाने के िलए उ र: (a)

जनमत सं ह म मतदान करे गी। िवक (a) सही है :

● िवभाजन के मामले म, र ा, वािण और संचार की सुर ा के िलए संयु भारत म िवं न चिचल के कायकाल के दौरान वेवेल योजना पेश की गई थी।

प से एक समझौता िकया जाना था। इस त के बावजूद िक यूरोप म ि तीय िव यु मई 1945 म समा हो गया,

● उपरो शत तभी लागू होंगी जब इं ड भारत को पूण श याँ ह ां त रत जापानी खतरा बना रहा। ि टे न म िवं न चिचल की िढ़वादी सरकार भारत

करे गा। के संवैधािनक संकट का समाधान खोजने के िलए उ ुक थी।

Q107. The ‘Wavell Plan’ was introduced in India during the reign of:
(a) Winston Churchill वेवेल योजना के मु ाव थे:
(b) Clement Atlee
● गवनर-जनरल और कमां डर-इन-चीफ को छोड़कर कायकारी प रषद के
(c) Neville Chamberlain
(d) Stanley Baldwin सभी सद भारतीय थे।

(e) None of the above/More than one of the above ● जाित मुसलमानों और िहं दुओं को समान ितिनिध िमलना था।
Answer: (a)
● पुनिनिमत प रषद को 1935 के अिधिनयम के ढां चे के भीतर एक अंत रम
Op on (a) is correct: During Winston Churchill's tenure in India, the
Wavell Plan was introduced. Despite the fact that the Second World सरकार के प म काय करना था (अथात, क ीय िवधानसभा के िलए

War in Europe ended in May 1945, the Japanese menace remained. िज ेदार नहीं)।
Winston Churchill's Conserva ve government in Britain was eager to
● गवनर-जनरल को मंि यों की सलाह पर अपने वीटो का योग करना था।
find a solu on to India's cons tu onal crisis.
The main proposals of the Wavell Plan were:
Topic Name: Modern History
● िविभ दलों के ितिनिधयों को कायकारी प रषद के नामां कन के िलए उ नस या अ सहायक पदों पर िनयु िकया गया था। उनके साथ पु षों

वायसराय को एक संयु सूची ुत करनी थी। संयु सूची न बन पाने की के समान वहार िकया जाता था और उ कोई िवशेष ान नहीं िदया जाता

थित म अलग-अलग सूिचयां ुत की जानी थी। था।

● यु समा होने के बाद एक नए संिवधान पर चचा की संभावनाएं खुली रखी


Q109. Which among the following was the commission set up to
जानी थीं।
inves gate the Jallianwala Bagh Massacre?
(a) Hunter Commission
Q108. Who among the following commanded the Rani of Jhansi (b) Raleigh Commission
regiment of INA (Indian Na onal Army)? (c) William Commission
(a) Lakshmi Swaminathan (d) Sargent Commission
(b) Kalpana Du (e) None of the above/More than one of the above
(c) Vijaya Lakshmi Pandit Answer: (a)
(d) Sarojini Naidu Op on (a) is correct: On 14 October 1919, a er orders issued by the
(e) None of the above/More than one of the above Secretary of State for India, Edwin Montagu, the Government of India
Answer: (a) announced the forma on of a commi ee of inquiry into the events in
Op on (a) is correct: Subhash Chandra Bose urged women to join the Punjab. Referred to as the Disorders Inquiry Commi ee, it was later
Indian Na onal Army as soldiers in 1943. (INA). This was a very radical more widely known as the Hunter Commission. This proved to be
viewpoint. In 1943, he founded the Rani of Jhansi Unit, a women's eyewash for the Indians.
regiment in the INA. Captain Lakshmi Swaminathan (Sahgal a er
109. जिलयां वाला बाग ह ाकां ड की जां च के िलए िन िल खत म से िकस
marriage) led this regiment. The bulk of troops were trained as
soldiers, but those who were not suited to combat responsibili es आयोग का गठन िकया गया था?
were engaged as nurses or in other support posi ons. They were (a) हं टर आयोग
treated the same as the males and were not given any special
(b) रै ले आयोग
a en on.

108. िन िल खत म से िकसने INA (भारतीय रा ीय सेना) की झाँ सी रे िजमट की (c) िविलयम आयोग

रानी की कमान संभाली थी? (d) साजट कमीशन

(a) ल ी ामीनाथन (e) उपरो म से कोई नहीं/उपरो म से एक से अिधक

(b) क ना द उ र: (a)

(c) िवजया ल ी पंिडत िवक (a) सही है :

(d) सरोिजनी नायडू 14 अ ू बर 1919 को, भारत के रा सिचव, एडिवन मोंटागु ारा जारी िकए

(e) उपरो म से कोई नहीं/उपरो म से एक से अिधक गए आदे शों के बाद, भारत सरकार ने पंजाब म घटनाओं की जाँ च के िलए एक

उ र: (a) सिमित के गठन की घोषणा की। िवकार जां च सिमित के प म संदिभत, इसे

िवक (a) सही है : बाद म ापक प से हं टर आयोग के प म जाना गया। यह भारतीयों के

सुभाष चं बोस ने मिहलाओं से 1943 म भारतीय रा ीय सेना म सैिनकों के िलए आं खों म धूल झोंकने वाला सािबत आ।

प म शािमल होने का आ ह िकया। (आईएनए)। यह ब त ही ां ितकारी


Q110. Consider the following statements is/are correct :
ि कोण था। 1943 म, उ ोंने INA म मिहला रे िजमट, झां सी यूिनट की रानी 1. Gandhiji decided to make salt the main focus of the Quit India

की थापना की। कै न ल ी ामीनाथन (शादी के बाद सहगल) ने इस movement.


2. The Quit India movement started as a reac on to the failure of
रे िजमट का नेतृ िकया। बड़ी सं ा म सैिनकों को सैिनकों के प म
the Cripps Mission.
िशि त िकया गया था, लेिकन जो यु की िज ेदा रयों के अनुकूल नहीं थे, Select the correct answer using the codes given below.
Topic Name: Modern History
(a) 1 only (e) None of the above/More than one of the above
(b) 2 only Answer: (e)
(c) Both 1 and 2 Op on (e) is correct: Royal Indian Navy Mu ny took place in February
(d) Neither 1 nor 2 1946. It was followed by an upheaval in the ci es of Calcu a, Bombay
(e) None of the above/More than one of the above and Karachi, which ul mately expanded to the countryside, followed
Answer: (b) by army and airforce par cipa on. This revolt was significant because
Op on (b) is correct: Statement 1 is not correct: Gandhiji used Salt as it shook the founda on of the Bri sh administra on as it was the
a main symbol of protest during the Civil Disobedience Movement, Navy-Airforce-Army which together strengthened the Bri sh rule in
launched in 1930 & not in Quit India Movement. India. So, this mu ny is considered to be the final nail in the coffin of
Statement 2 is correct: Quit India Movement was launched in August the Bri sh Empire.
1942 as an outcome of the failure of the Cripps Mission & evident 111. िन िल खत म से कौन भारत म ि िटश सा ा के ताबूत म अंितम कील
danger of the Japanese invasion of India.
सािबत आ?
110.िन िल खत कथनों पर िवचार कर सही है /ह:
(a) 1947 के आरआईएन िव ोह
1. गां धीजी ने नमक को भारत छोड़ो आं दोलन का मु फोकस बनाने का
(b) गां धी के भारत छोड़ो आं दोलन करो या मरो के नारे के साथ
फैसला िकया।
(c) भारतीय रा ीय सेना का गठन और दि ण पूव एिशया म अं ेजों की बाद की
2. ि िमशन की िवफलता की िति या के प म भारत छोड़ो आं दोलन
वापसी
शु आ।
(d) िज ा ारा दी गई सीधी कारवाई
नीचे िदए गए कूट का योग कर सही उ र चुिनए।
(e) उपरो म से कोई नहीं/उपरो म से एक से अिधक
(a) केवल 1
उ र: (a)
(b) केवल 2
िवक (a) सही है : रॉयल इं िडयन नेवी िव ोह फरवरी 1946 म आ था। इसके
(c) 1 और 2 दोनों
बाद कलक ा, बॉ े और कराची शहरों म उथल-पुथल ई, जो अंततः ामीण
(d) न तो 1 और न ही 2
इलाकों तक फैल गई, िजसके बाद सेना और वायु सेना की भागीदारी ई। यह
(e) उपरो म से कोई नहीं/उपरो म से एक से अिधक
िव ोह मह पूण था ोंिक इसने ि िटश शासन की नींव िहला दी थी ोंिक
उ र: (b)
यह नेवी-एयरफोस-आम थी िजसने िमलकर भारत म ि िटश शासन को
िवक (b) सही है :
मजबूत िकया था। अतः इस िव ोह को ि िटश सा ा के ताबूत म अंितम
कथन 1 सही नहीं है : गां धीजी ने 1930 म शु िकए गए सिवनय अव ा
कील माना जाता है ।
आं दोलन के दौरान िवरोध के मु तीक के प म नमक का इ ेमाल िकया
Q112. The Radcliffe Commission was appointed to:
था न िक भारत छोड़ो आं दोलन म। (a) Solve the problem of minori es in India

कथन 2 सही है : ि िमशन की िवफलता और भारत पर जापानी आ मण (b) Give effect to the Independence Bill
(c) Delimit the boundaries between India and china
के खतरे के प रणाम प अग 1942 म भारत छोड़ो आं दोलन शु
(d) Enquire into the riots in East Bengal
िकया गया था। (e) None of the above/More than one of the above

Q111. Which of the following proved to be the final nail in the coffin Answer: (e)
of the Bri sh Empire in India? Op on (e) is correct: The Boundary Commission, chaired by Sir Cyril

(a) RIN Mu ny of 1947 Radcliffe, was established by the Bri sh government to demarcate the

(b) Gandhi's Quit India Movement with a do or die slogan borders between India and Pakistan. The Boundary Commissions in

(c) Forma on of the Indian Na onal Army and the subsequent each case were made up of two Muslims and two non-Muslim judges
retreat of the Bri sh in SE Asia who operated under severe me limita ons. Radcliffe was given six

(d) Direct Ac on given by Jinnah weeks to design the boundaries and resolve contested issues, despite
Topic Name: Modern History
having no knowledge of India and relying on out-of-date maps and (a) सश बलों का िवभाजन।
census sta s cs. There is a demarca on line between the Indian and
(b) मु म-ब ल इकाइयों को पािक ान म थानां त रत कर िदया गया था।
Pakistani por ons of the Punjab and Bengal provinces of Bri sh India.

112. रै ड फ आयोग को िनयु िकया गया था: (c) दोनों रा ों के बीच महान मतभेद उ ए।

(a) भारत म अ सं कों की सम ा को हल कर (d) ि िटश सैिनकों ने भारत छोड़ना शु कर िदया

(b) तं ता िवधेयक को भावी कर (e) उपरो म से कोई नहीं/उपरो म से एक से अिधक

(c) भारत और चीन के बीच सीमाओं का प रसीमन कर उ र: (c)

(d) पूव बंगाल म दं गों म पूछताछ िवक (c) सही है :

(e) उपरो म से कोई नहीं/उपरो म से एक से अिधक सु ीम कमां डर के प म ऑिचनलेक के नेतृ म एक संयु र ा प रषद की

उ र: (e) थापना सश बलों और उनके संयं ों, मशीनरी, उपकरण और ॉक के

िवक (e) सही है : सुचा िवभाजन को सुिनि त करने के िलए की गई थी। प रषद ने संक

सर िस रल रै ड फ की अ ता म सीमा आयोग की थापना ि िटश िलया िक मु म-ब ल इकाइयों को पािक ान और गैर-मु म सैिनकों को

सरकार ारा भारत और पािक ान के बीच सीमाओं का सीमां कन करने के भारत को सौंप िदया जाना चािहए, लेिकन दोनों प ों के बीच पया असहमित

िलए की गई थी। ेक मामले म सीमा आयोग दो मुसलमानों और दो के कारण सु ीम कमां डर का पद समा कर िदया गया था। 17 अग , 1947

गैर-मु म ायाधीशों से बना था, जो गंभीर समय सीमाओं के तहत काम को ब त अ व था के बीच, ि िटश सेना ने भारत छोड़ना शु िकया और यह

करते थे। रै ड फ को भारत के बारे म कोई जानकारी नहीं होने और पुराने ि या फरवरी 1948 तक समा हो गई।

Q114. Stands ll Agreement is related to which of the following?


न ों और जनगणना के आं कड़ों पर भरोसा करने के बावजूद सीमाओं को
(a) Ramsay MacDonald's Communal Award
िडजाइन करने और िववािदत मु ों को हल करने के िलए छह स ाह का समय (b) Swadeshi Movement

िदया गया था। ि िटश भारत के पंजाब और बंगाल ां तों के भारतीय और (c) Princely states' reorganisa on
(d) The 1857 sepoy's revolt
पािक ानी भागों के बीच एक सीमां कन रे खा है ।
(e) None of the above/More than one of the above
Q113. The post of Supreme Commander during the par on was Answer: (c)
abolished due to: Op on (c) is correct: Unifying post-par on India & 565 Princely
(a) Division of the armed forces. States under one administra on was perhaps the most important task
(b) Muslim-majority units were transferred to Pakistan. faced by then poli cal leadership. Most princely states joined the
(c) Great differences arose between the two na ons. Cons tuent Assembly, but States like Junagarh, Kashmir, Manipur, and
(d) Bri sh troops started to leave India Hyderabad publicly announced their desire to claim independent
(e) None of the above/More than one of the above status. Among them, Kashmir and Hyderabad signed a Stands ll
Answer: (c) Agreement.
Op on (c) is correct: A combined defence council, led by Auchinleck The ruler of Kashmir, whose state was situated between the two new
as Supreme Commander, was established to ensure a smooth division countries, could not decide which country to join. Hyderabad was the
of the armed forces and their plants, machinery, equipment, and largest Princely state. The Nizam wanted an independent status for
stocks. The council resolved that Muslim-majority units should be Hyderabad & thus entered into the nego a on of a stands ll
handed to Pakistan and non-Muslim troops to India, but the tle of agreement with India.
Supreme Commander was abolished due to substan al disagreements
114. ड ल ए ीमट िन िल खत म से िकससे संबंिधत है ?
between the two par es. The Bri sh forces began to leave India on
August 17, 1947, amid much disarray, and the process was finished by (a) रामसे मैकडोना ् स सां दाियक पुर ार

February 1948. (b) दे शी आं दोलन


113. िवभाजन के दौरान सु ीम कमां डर का पद समा कर िदया गया था:
(c) रयासतों का पुनगठन
Topic Name: Modern History
(d) 1857 का िसपाही िव ोह (d) एक ाकृितक वातावरण के भीतर रचना क िश ा

(e) उपरो म से कोई नहीं/उपरो म से एक से अिधक (e) उपरो म से कोई नहीं/उपरो म से एक से अिधक

उ र: (c) उ र: (e)

िवक (c) सही है : िवक (e) सही है :

िवभाजन के बाद के भारत और 565 रयासतों को एक शासन के तहत रवीं नाथ टै गोर का मानना था िक रचना क िश ा को केवल एक ाकृितक

एकीकृत करना शायद त ालीन राजनीितक नेतृ के िलए सबसे मह पूण सेिटं ग म ही पोिषत िकया जा सकता है । ऐसा करने के िलए, उ ोंने 1901 म

काय था। अिधकां श रयासत संिवधान सभा म शािमल ईं, लेिकन जूनागढ़, शां ितिनकेतन की थापना की। युवा इस माहौल म रचना क और मु हो

क ीर, मिणपुर और है दराबाद जैसे रा ों ने सावजिनक प से तं थित सकते ह। उसे अपनी इ ाओं और िवचारों की जां च करने की अनुमित दी गई

का दावा करने की अपनी इ ा की घोषणा की। उनम से, क ीर और थी। टै गोर का मानना था िक बचपन -िश ा का काल होना चािहए, जो अं ेजों

है दराबाद ने एक ड ल समझौते पर ह ा र िकए। ारा थोपे गए कठोर और ितबंधा क अनुशासन से मु हो। िश कों को

क ीर के शासक, िजसका रा दो नए दे शों के बीच थत था, यह तय नहीं रचना क होना चािहए, ब ों को समझना चािहए और ब े की िज ासा को

कर सका िक िकस दे श म शािमल होना है । है दराबाद सबसे बड़ी रयासत थी। ो ािहत करना चािहए। टै गोर के अनुसार, पारं प रक ू ल एक ब े की

िनज़ाम है दराबाद के िलए एक तं थित चाहता था और इस कार भारत के रचना क होने की अंतिनिहत इ ा के साथ-साथ उसके आ य की भावना का

साथ एक ठहराव समझौते की बातचीत म वेश िकया। भी दम घोंट दे ते ह।

Q115. Which one of the following is the chief characteris c of Q116. Hindu College was established in Benaras in 1791. What was
'Rabindranath Tagore’s view about educa on? the main purpose behind it?
(a) Indian languages ought to be the medium of teaching (a) To encourage the study of ancient Sanskrit texts that would be
(b) Childhood should be a period of self-learning useful for the administra on of the country
(c) Rigid and restric ng discipline of the schooling system (b) To promote the study of Arabic, Persian and Islamic law
(d) Crea ve learning within a natural environment (c) To promote English as the medium of instruc on for higher
(e) None of the above/More than one of the above educa on
Answer: (e) (d) statements (b), and (c) given above is correct
Op on (e) is correct: Rabindranath Tagore believed that crea ve (e) None of the above/More than one of the above
learning could only be nurtured in a natural se ng. In order to do Answer: (a)
this, he founded San niketan in 1901. The youngster might be Op on (a) is correct: Many Company officials suggested that the
crea ve and free in this environment. She was allowed to inves gate Bri sh should foster an Indian rather than a Western educa on
her own desires and thoughts. Tagore believed that childhood should system a er the Asia c Society of Bengal championed the idea of
be a period of self-learning, free from the Bri sh-imposed harsh and reviving India's illustrious past. Ins tu ons should be established to
restric ve discipline of schools. Teachers needed to be crea ve, promote the study of old Indian texts and to educate Sanskrit and
understand the youngster, and encourage the child's curiosity. Persian literature and poetry, they believed. The administrators also
According to Tagore, conven onal schools suffocate a child's inherent believed that Hindus and Muslims should be taught subjects that they
yearning to be crea ve, as well as her sense of wonder. were already familiar with and appreciated, rather than those that

115. िन िल खत म से कौन-सी िश ा के बारे म रवी नाथ टै गोर के ि कोण were unfamiliar to them. Only then, they believed, could the Bri sh
hope to get a foothold in the hearts of the "na ves," and only then
की मुख िवशेषता है ?
could alien rulers expect their subjects to respect them.
(a) भारतीय भाषाओं को िश ण का मा म होना चािहए A madrasa was created in Calcu a in 1781 to promote the study of
Arabic, Persian, and Islamic law, and the Hindu College in Benaras in
(b) बचपन आ -सीखने की अविध होनी चािहए
1791 to s mulate the study of ancient Sanskrit wri ngs that would be
(c) ू ली िश ा णाली के कठोर और ितबंिधत अनुशासन
important for the country's administra on.
Topic Name: Modern History
116. 1791 म बनारस म िह दू कॉलेज की थापना ई। इसके पीछे मु be solely for modern studies. While Western sciences and literature
should be taught to prepare students for jobs, Orientalists believe that
उ े ा था?
tradi onal Indian learning should be expanded. Even the Anglicists
(a) ाचीन सं ृ त ंथों के अ यन को ो ािहत करने के िलए जो दे श के were divided on the issue of medium of instruc on: one party

advocated for the use of English, while the other advocated for the
शासन के िलए उपयोगी होगा
use of Indian languages (vernaculars).
(b) अरबी, फारसी और इ ामी कानून के अ यन को बढ़ावा दे ने के िलए
The renowned Lord Macaulay's Minute concluded the dispute in
(c) उ िश ा के िलए िश ा के मा म के प म अं ेजी को बढ़ावा दे ना favour of Anglicists, sta ng that limited government resources would
be allocated solely to teaching Western sciences and literature in
(d) कथन (b), और (c) ऊपर िदए गए सही ह
English. Lord Macaulay believed that "Indian learning was inferior to
(e) उपरो म से कोई नहीं/उपरो म से एक से अिधक European learning," which was true in the modern stage of scien fic

उ र: (a) and social sciences.

117. भारतीय इितहास के संदभ म ा वादी-आं वादी िववाद आ। इस


िवक (a) सही है :
िववाद का आधार ा था?
कंपनी के कई अिधका रयों ने सुझाव िदया िक बंगाल की एिशयािटक सोसाइटी
(a) भारत म अं ेजी िश ा का प रचय
ारा भारत के शानदार अतीत को पुनज िवत करने के िवचार के बाद अं ेजों
(b) म सुधार आयोग की थापना
को पि मी िश ा णाली के बजाय एक भारतीय को बढ़ावा दे ना चािहए।
(c) दे श के सभी समाज सुधार समूहों को एक साथ एक संगठन के अंतगत लाना
उनका मानना था िक पुराने भारतीय ंथों के अ यन को बढ़ावा दे ने और
(d) उपरो कथन (b), और (c) इस संदभ म सही ह
सं ृ त और फारसी सािह और किवता को िशि त करने के िलए सं थानों की
(e) उपरो म से कोई नहीं/उपरो म से एक से अिधक
थापना की जानी चािहए। शासकों का यह भी मानना था िक िहं दुओं और
उ र: (a)
मुसलमानों को उन िवषयों को पढ़ाया जाना चािहए िजनसे वे पहले से ही
िवक (a) सही है :
प रिचत थे और उनकी सराहना करते थे, न िक वे जो उनके िलए अप रिचत थे।
आं वािदयों ने सावजिनक िनदश पर सामा सिमित म तक िदया िक िश ा
उनका मानना था िक केवल तभी, अं ेज "मूलिनवािसयों" के िदलों म पैर जमाने
पर सरकारी खच पूरी तरह से आधुिनक अ यन के िलए होना चािहए। जबिक
की उ ीद कर सकते ह और तभी िवदे शी शासक अपनी जा से स ान की
छा ों को नौक रयों के िलए तैयार करने के िलए पि मी िव ान और सािह
उ ीद कर सकते ह।
पढ़ाया जाना चािहए, ओ रएं टिल मानते ह िक पारं प रक भारतीय िश ा का
1781 म कलक ा म अरबी, फारसी और इ ामी कानून के अ यन को
िव ार िकया जाना चािहए। यहाँ तक िक िश ा के मा म के मु े पर
बढ़ावा दे ने के िलए एक मदरसा बनाया गया था और 1791 म बनारस म िहं दू
आं वादी भी िवभािजत थे: एक प ने अं ेजी के उपयोग की वकालत की,
कॉलेज ाचीन सं ृ त लेखन के अ यन को ो ािहत करने के िलए बनाया
जबिक दू सरे ने भारतीय भाषाओं ( थानीय भाषाओं) के उपयोग की वकालत
गया था जो दे श के शासन के िलए मह पूण होगा।

Q117. In the context of Indian history, the Orientalist-Anglicist की।


Controversy took place. What was the basis of this controversy? िस लॉड मैकाले के िमनट ने एं िस के प म िववाद का िन ष
(a) Introduc on to English Educa on in India
िनकाला, िजसम कहा गया िक सीिमत सरकारी संसाधनों को केवल पि मी
(b) Establishment of the labour reform commission
(c) Bringing together all the social reform groups of the country िव ान और सािह को अं ेजी म पढ़ाने के िलए आवंिटत िकया जाएगा। लॉड

under one organisa on मैकाले का मानना था िक "भारतीय िश ा यूरोपीय िश ा से हीन थी," जो


(d) statements (b), and (c) given above is correct in this context
वै ािनक और सामािजक िव ान के आधुिनक चरण म सच थी।
(e) None of the above/More than one of the above
Answer: (a)
Op on (a) is correct: The Anglicists argued at the General Commi ee Q118. The Woods dispatch of 1854 is related to:
on Public Instruc on that government spending on educa on should (a) Hindi Educa on in India
Topic Name: Modern History
(b) Introduc on of the India Post एक संब कॉलेज, और कलक ा बॉ े और म ास के ेसीडसी शहरों म
(c) Aboli on of the Sa
िव िव ालयों को संब िकया।
(d) Establishment of Administra on Services in India
(e) None of the above/More than one of the above ● इसने ूल र पर उ अ यन और थानीय भाषाओं के िलए िश ा के

Answer: (e)
मा म के प म अं ेजी की िसफा रश की। इसने मिहला और ावसाियक
Op on (e) is correct: In 1854, Charles Wood prepared a despatch
िश ा और िश कों के िश ण पर जोर िदया।
(Wood’s Despatch) on an educa onal system for India. Considered the
“Magna Carta of English Educa on in India”, this was the first ● इसने िनधा रत िकया िक सरकारी सं थानों म दी जाने वाली िश ा धमिनरपे
comprehensive plan for the spread of educa on in India.
होनी चािहए।
● It asked the Government of India to assume responsibility
for the educa on of the masses, thus repudia ng the ● इसने िनजी उ म को ो ािहत करने के िलए सहायता अनुदान की एक
‘downward filtra on theory’, at least on paper. णाली की िसफा रश की।
● It systema sed the hierarchy from vernacular primary
schools in villages at the bo om, followed by
Q119. Who among the following Viceroys repealed the Vernacular
Anglo-Vernacular High Schools and an affiliated college at
Press Act in 1882?
the district level, and affilia ng universi es in the presidency
(a) Lord Ripon
towns of Calcu a Bombay and Madras.
(b) Lord Ly on
● It recommended English as the medium of instruc on for
(c) Lord Elgin-II
higher studies and vernaculars at the school level. It laid
(d) Lord Curzon
stress on female and voca onal educa on and teachers’
(e) None of the above/More than one of the above
training.
Answer: (a)
● It laid down that the educa on imparted in government
Op on (a) is correct: From its very beginning, the na onal movement
ins tu ons should be secular.
stood for the freedom of the press. The Indian newspapers became
● It recommended a system of grants-in-aid to encourage
highly cri cal of Lord Ly on’s administra on, especially regarding its
private enterprise.
inhuman treatment of vic ms of the famine of 1876-77. The
118. 1854 का वुड्स िड ैच संबंिधत है : Government struck back with the Vernacular Press Act, 1878 (VPA).

(a) भारत म िहं दी िश ा The Act came to be nicknamed “the gagging Act”. The worst features
of this Act were:
(b) भारतीय डाक का प रचय
● Discrimina on between English press and vernacular press.
(c) सती का उ ूलन ● No right of appeal.

(d) भारत म शासन सेवाओं की थापना Under VPA, proceedings were ins tuted against different newspapers
like Som Prakash, Bharat Mihir, Dacca Prakash and Samachar.
(e) उपरो म से कोई नहीं/उपरो म से एक से अिधक
(Incidentally, the Amrita Bazar Patrika( Sisir Kumar Ghose and Mo Lal
उ र: (e) Ghosh) turned overnight into an English newspaper to escape the
VPA.) A er some me, a press commissioner was appointed to supply
िवक (e) सही है :
authen c and accurate news, and the pre-censorship clause was
1854 म, चा वुड ने भारत के िलए एक शैि क णाली पर एक िड ैच repealed. Lord Ripon repealed the act in 1882 due to the strong

(वुड्स िड ैच) तैयार िकया था। "भारत म अं ेजी िश ा का मै ा काटा" माना opposi on
119. िन िल खत म से िकस वायसराय ने 1882 म वना ूलर ेस ए को
जाता है , यह भारत म िश ा के सार के िलए पहली ापक योजना थी।
िनर िकया था?
● इसने भारत सरकार से जनता की िश ा की िज ेदारी लेने के िलए कहा, इस
(a) लॉड रपन
कार कम से कम कागज पर 'डाउनवड िफ े शन ोरी' का खंडन िकया।
(b) लॉड िलटन
● इसने नीचे के गाँ वों म थानीय भाषा के ाथिमक िव ालयों से पदानु म को
(c) लॉड ए न-ि तीय
व थत िकया, इसके बाद एं ो-वना ूलर हाई ू ल और िजला र पर
Topic Name: Modern History
(d) लॉड कजन (d) जी. सु म मअ र

(e) उपरो म से कोई नहीं/उपरो म से एक से अिधक (e) उपरो म से कोई नहीं/उपरो म से एक से अिधक

उ र: (a) उ र: (b)

िवक (a) सही है : िवक (b) सही है :

शु आत से ही, रा ीय आं दोलन ेस की तं ता के िलए खड़ा था। भारतीय 1883 म, सुर नाथ बनज वना ुलर ेस ए , 1878 के तहत कैद होने वाले

समाचार प लॉड िलटन के शासन की अ िधक आलोचना क हो गए, पहले भारतीय प कार बने। बंगाली म एक नाराज संपादकीय म, बनज ने एक

िवशेष प से 1876-77 के अकाल के पीिड़तों के अमानवीय वहार के बारे म बंगािलयों की धािमक भावनाओं के ित असंवेदनशील होने के िलए कलक ा

म। सरकार ने वना ूलर ेस ए , 1878 (वीपीए) के साथ वापसी की। इस उ ायालय के एक ायाधीश की आलोचना की थी। उसके फैसलों की।

अिधिनयम को "द गैिगंग ए " का उपनाम िदया गया। इस अिधिनयम की


Q121. In 1906, which of the following states introduced compulsory
सबसे खराब िवशेषताएं थीं:
primary educa on throughout its territory?
● अं ेजी ेस और वना ूलर ेस के बीच भेदभाव। (a) Baroda
(b) Bengal
● अपील का अिधकार नहीं।
(c) Bhopal
वीपीए के तहत सोम काश, भारत िमिहर, ढाका काश और समाचार जैसे
(d) Madras
िविभ अखबारों के खलाफ कायवाही शु की गई। (संयोग से, अमृत बाजार (e) None of the above/More than one of the above
Answer: (a)
पि का (िशिशर कुमार घोष और मोती लाल घोष) वीपीए से बचने के िलए
Op on (a) is correct: The progressive state of Baroda implemented
रातोंरात एक अं ेजी अखबार म बदल गई।) कुछ समय बाद, ामािणक और obligatory elementary educa on across its en re area in 1906.

सटीक समाचार और पूव-ससरिशप खंड की आपूित के िलए एक ेस आयु Leaders from across the country encouraged the government to do so
for Bri sh India (Gopal Krishan Gokhale made powerful advocacy for it
िनयु िकया गया। िनर कर िदया गया था। लॉड रपन ने 1882 म कड़े
in the Legisla ve Assembly). The government refused to take on the
िवरोध के कारण अिधिनयम को िनर कर िदया| responsibility of compulsory educa on in its 1913 Resolu on on

Q120.Who among the following was the first Indian journalist Educa on Policy but accepted the policy of elimina ng illiteracy and
imprisoned under the Vernacular Press Act, 1878? urged provincial governments to take early steps to provide free

(a) Bal Gangadhar Tilak elementary educa on to the poorer and more backward sec ons. The

(b) Surendranath Banerjea quality of secondary schools was to be improved, and private

(c) Dadabhai Naoroji ini a ves were to be supported.

(d) G. Subramaniya Aiyar 121. 1906 म, िन िल खत म से िकस रा ने अपने पूरे े म अिनवाय


(e) None of the above/More than one of the above
ाथिमक िश ा की शु आत की?
Answer: (b)
Op on (b) is correct: In 1883, Surendranath Banerjea became the first (a) बड़ौदा
Indian journalist to be imprisoned under the Vernacular Press Act, (b) बंगाल
1878. In an angry editorial in The Bengalee, Banerjea had cri cised a
(c) भोपाल
judge of Calcu a High Court for being insensi ve to the religious
sen ments of Bengalis in one of his judgements. (d) म ास

120.िन िल खत म से कौन पहला भारतीय प कार था िजसे वना ूलर ेस (e) उपरो म से कोई नहीं/उपरो म से एक से अिधक

ए , 1878 के तहत कैद िकया गया था? उ र: (a)

(a) बाल गंगाधर ितलक िवक (a) सही है :

(b) सुर नाथ बनज

(c) दादाभाई नौरोजी


Topic Name: Modern History
बड़ौदा के गितशील रा ने 1906 म अपने पूरे े म अिनवाय ारं िभक िवक (c) सही है :

िश ा को लागू िकया। दे श भर के नेताओं ने सरकार को ि िटश भारत के िलए सैडलर िव िव ालय आयोग (1917-19) की थापना कलक ा िव िव ालय म

ऐसा करने के िलए ो ािहत िकया (गोपाल कृ गोखले ने िवधान सभा म मु ों की जां च और रपोट करने के िलए की गई थी, लेिकन इसके सुझाव मोटे

इसके िलए श शाली वकालत की ). सरकार ने िश ा नीित पर अपने 1913 तौर पर अ िव िव ालयों के िलए ासंिगक थे। इसने ाथिमक िव ालय

के संक म अिनवाय िश ा की िज ेदारी लेने से इनकार कर िदया लेिकन िश ा से लेकर उ िश ा तक सब कुछ दे खा। यह माना जाता था िक उ

िनर रता को ख करने की नीित को ीकार कर िलया और ां तीय सरकारों िश ा म सुधार के िलए मा िमक िश ा म सुधार एक मह पूण अ दू त था।

से गरीब और अिधक िपछड़े वग को मु ाथिमक िश ा दान करने के िलए ू ल काय म की अविध 12 वष होनी चािहए। इं टरमीिडएट चरण (मैिटक के

शी कदम उठाने का आ ह िकया। मा िमक िव ालयों की गुणव ा म सुधार बजाय) पूरा करने के बाद, छा ों को तीन साल के िड ी ो ाम को आगे बढ़ाने

िकया जाना था, और िनजी पहलों का समथन िकया जाना था। के िलए िव िव ालय म वेश करना चािहए। यह (ए) िव िव ालय के िलए

Q122. With reference to the reforms of Bri sh, Sadler Commission छा ों को तैयार करने के िलए िकया गया था, (बी) िव िव ालय के मानकों को
was related to:
पूरा नहीं करने वाले छा ों की एक बड़ी सं ा से िव िव ालयों को राहत दे ने
(a) Police Reform
(b) Cons tu onal Reforms के िलए, और (सी) उन यों को कॉलेिजएट िश ा दान करने के िलए

(c) Educa onal Reforms


िकया गया था जो िव िव ालय म भाग लेने का इरादा नहीं रखते थे। मा िमक
(d) Social Reforms
और मा िमक िश ा को एक अलग िश ा बोड ारा शािसत और िविनयिमत
(e) None of the above/More than one of the above
Answer: (c) िकया जाना चािहए।
Op on (c) is correct: Saddler University Commission (1917-19) was Q123. The report made by William Adam is related to one of the
established to inves gate and report on issues at Calcu a University, following?
but its sugges ons were broadly relevant to other universi es. It (a) It was a report related to the study of indigenous educa on in
looked at everything from elementary school educa on to higher India, which talked about the existence of village pathshalas.
educa on. It was believed that improving secondary educa on was a (b) It was a report men oning the role of Indian peasants, who were
vital precursor to improving higher educa on. involved in manufacturing indigo.
The dura on of the school programme should be 12 years. A er (c) It men oned the existence of social evils & recommended
comple ng an intermediate stage (rather than matric), students William Ben nck for the aboli on of Sa .
should enter university to pursue a three-year degree programme. (d) It was concerned with the development of infrastructure in India.
This was done in order to (a) prepare students for university, (b) (e) None of the above/More than one of the above
relieve universi es of a huge number of students who did not meet Answer: (a)
university standards, and (c) provide collegiate educa on to Op on (a) is correct: Lord William Ben nck, the governor-general of
individuals who did not intend to a end university. Secondary and India, appointed Adam in 1835 to survey the state of educa on in
intermediate educa on should be administered and regulated by a Bengal and Bihar and to suggest reforms. Adam submi ed 3 reports
separate board of educa on. (1835-1838). Adam's First report: Educa on was given by religious
122. अं ेजों के सुधारों के स भ म सैडलर आयोग का संबंध था: ins tu ons. There were many schools of these types in Bengal. There

(a) पुिलस सुधार were 100000 such schools in Bihar and Bengal. He submi ed a second
report on 23rd December 1836. There were two types of schools: one
(b) संवैधािनक सुधार
was regular schools that were similar to modern schools, and the
(c) शैि क सुधार other was domes c schools where family members taught the
children or a teacher was appointed to teach.
(d) सामािजक सुधार
123. िविलयम एडम ारा बनाई गई रपोट िन िल खत म से एक से संबंिधत
(e) उपरो म से कोई नहीं/उपरो म से एक से अिधक
है ?
उ र: (c)
Topic Name: Modern History
(a) यह भारत म दे शी िश ा के अ यन से संबंिधत एक रपोट थी, िजसम (b) लॉड हािडग

ामीण पाठशालाओं के अ के बारे म बात की गई थी। (c) लॉड रीिडं ग

(b) यह भारतीय िकसानों की भूिमका का उ ेख करने वाली एक रपोट थी, जो (d) लॉड डलहौजी

इं िडगो के िनमाण म शािमल थे। (e) उपरो म से कोई नहीं/उपरो म से एक से अिधक

(c) इसने सामािजक बुराइयों के अ का उ ेख िकया और सती के उ र: (e)

उ ूलन के िलए िविलयम बिटक की िसफा रश की। िवक (e) सही है :

(d) यह भारत म बुिनयादी ढां चे के िवकास से संबंिधत था। 1835 म चा मेटकाफ ारा भारतीय ेस को ितबंधों से मु कर िदया गया

(e) उपरो म से कोई नहीं/उपरो म से एक से अिधक था। उ ोंने ेस की तं ता की जां च करने वाले 1823 के अि य अ ादे श को

उ र: (a) िनर कर िदया। उ 'भारत ेस का मु दाता' कहा जाता है । इस कदम का

िवक (a) सही है : िशि त भारतीयों ने उ ाहपूवक ागत िकया था। यह एक कारण था िक

भारत के गवनर-जनरल लॉड िविलयम बिटक ने 1835 म एडम को बंगाल और उ ोंने कुछ समय के िलए भारत म ि िटश शासन का समथन िकया था।

िबहार म िश ा की थित का सव ण करने और सुधारों का सुझाव दे ने के िलए


Q125. In the context of the Indian Na onal Movement, Zakir
िनयु िकया था। एडम ने 3 रपोट (1835-1838) ुत की। एडम का थम
Hussain commi ee’s “Learning through Ac vity/Work” principle is
ितवेदन िश ा धािमक सं थाओं ारा दी जाती थी। बंगाल म इस कार के related to which of the following educa onal schemes?

(a) Macaulay’s minute


अनेक िव ालय थे। िबहार और बंगाल म ऐसे 100000 ू ल थे। उ ोंने 23
(b) Wardha Scheme of Basic Educa on
िदसंबर 1836 को दू सरी रपोट ुत की। ू ल दो कार के थे: एक िनयिमत
(c) Sargent Plan of Educa on
ू ल थे जो आधुिनक ू लों के समान थे, और दू सरा घरे लू ू ल थे जहाँ (d) Indian Universi es Act, 1904
(e) None of the above/More than one of the above
प रवार के सद ब ों को पढ़ाते थे या पढ़ाने के िलए एक िश क िनयु
Answer: (b)
िकया जाता था। Op on (b) is correct: The Congress had organised a Na onal
Conference on Educa on in October 1937 in Wardha. In the light of
Q124. Who among the following is known as the ‘liberator of the the resolu ons passed there, the Zakir Hussain commi ee formulated
press’ in Bri sh India? a detailed na onal scheme for basic educa on, which came to be
(a) Lord Ripon known as the Wardha Scheme of Basic Educa on (1937).
(b) Lord Hardinge The main principle behind this scheme was ‘learning through ac vity.
(c) Lord Reading It was based on Gandhi’s ideas published in a series of ar cles in the
(d) Lord Dalhousie weekly Harijan. Gandhi thought that Western educa on had created a
(e) None of the above/More than one of the above gulf between the educated few and the masses and had also made
Answer: (e) the educated elite ineffec ve.
Op on (e) is correct: The Indian press was freed of restric ons by 125. भारतीय रा ीय आ ोलन के स भ म, जािकर सैन सिमित का
Charles Metcalfe in 1835. He repealed the obnoxious 1823 ordinance
"गितिविध/काय के मा म से सीखना" िस ां त िन िल खत म से िकस शैि क
that checked the freedom of the Press. He is called 'Liberator of India
Press'. This step had been welcomed enthusias cally by the educated योजना से संबंिधत है ?
Indians. It was one of the reasons why they had for some me (a) मैकाले का िमनट
supported Bri sh rule in India.
(b) बेिसक िश ा की वधा योजना
124. िन िल खत म से िकसे ि िटश भारत म ' ेस के मु दाता' के पम
(c) िश ा की साजट योजना
जाना जाता है ?
(d) भारतीय िव िव ालय अिधिनयम, 1904
(a) लॉड रपन
Topic Name: Modern History
(e) उपरो म से कोई नहीं/उपरो म से एक से अिधक िव भारती एक सावजिनक अनुसंधान क िव िव ालय और शां ितिनकेतन,

उ र: (b) पि म बंगाल, भारत म थत एक रा ीय मह का सं थान है । इसकी थापना

िवक (b) सही है : रवीं नाथ टै गोर ने की थी, िज ोंने इसे िव भारती कहा, िजसका अथ है भारत

कां ेस ने अ ू बर 1937 म वधा म िश ा पर एक रा ीय स ेलन का आयोजन के साथ दु िनया का जुड़ाव। आजादी के पहले तक यह एक कॉलेज था। तं ता

िकया था। वहां पा रत ावों के आलोक म, जािकर सैन सिमित ने बुिनयादी के तुरंत बाद, सं थान को संसद के एक अिधिनयम ारा 1951 म एक क ीय

िश ा के िलए एक िव ृत रा ीय योजना तैयार की, िजसे बुिनयादी िश ा की िव िव ालय का दजा िदया गया था।

वधा योजना (1937) के प म जाना गया।


Q127. Which of the following Commissions/Commi ees was set up
इस योजना के पीछे मु िस ां त 'गितिविध के मा म से सीखना' था। यह
to go into condi ons and prospects of universi es in India and to
सा ािहक ह रजन म लेखों की एक ृंखला म कािशत गां धी के िवचारों पर suggest measures for improvement in their cons tu on and working

during Bri sh India?


आधा रत था। गां धी ने सोचा था िक पि मी िश ा ने कुछ िशि त लोगों और
(a) Butler Commi ee
जनता के बीच एक खाई पैदा कर दी है और िशि त अिभजात वग को भी
(b) Lee Commission
अ भावी बना िदया है । (c) Raleigh Commission

Q126. Which of the following universi es was founded by (d) Aitchison Commission
Rabindranath Tagore? (e) None of the above/More than one of the above

(a) Bombay University Answer: (c)

(b) Visvabhara Op on (c) is correct: In 1902, Raleigh Commission was set up by Lord

(c) Calcu a University Curzon to go into the condi ons and prospects of universi es in India

(d) Banaras Hindu University and to suggest measures for improvement in their cons tu on and

(e) None of the above/More than one of the above working.

Answer: (b) The commission precluded repor ng on primary or secondary

Op on (b) is correct: Visva-Bhara is a public research centre educa on. Based on its recommenda ons, the Indian Universi es Act
university and an Ins tu on of Na onal Importance located in was passed in 1904. As per the Act,
San niketan, West Bengal, India. It was founded by Rabindranath ● Universi es were to give more a en on to study and

Tagore, who called it Visva-Bhara , which means the communion of research.

the world with India. Un l independence, it was a college. Soon a er ● The number of fellows of a university and their period in the

independence, the ins tu on was given the status of a central office was reduced, and most fellows were to be nominated

university in 1951 by an act of the Parliament. by the Government.


● The Government was to have powers to veto universi es’
126. िन िल खत म से िकस िव िव ालय की थापना रवी नाथ टै गोर ने की
senate regula ons and could amend these regula ons or
थी? pass regula ons on its own.

(a) बॉ े िव िव ालय ● Condi ons were to be made stricter for affilia on of private
colleges.
(b) िव भारती
● Five lakh rupees were to be sanc oned per annum for five
(c) कलक ा िव िव ालय years for the improvement of higher educa on and
universi es.
(d) बनारस िहं दू िव िव ालय
127. िन िल खत म से िकस आयोग/सिमित का गठन भारत म िव िव ालयों
(e) उपरो म से कोई नहीं/उपरो म से एक से अिधक
की थितयों और संभावनाओं पर िवचार करने और ि िटश भारत के दौरान
उ र: (b)
उनके संिवधान और काय णाली म सुधार के उपाय सुझाने के िलए िकया गया
िवक (b) सही है :
था?
Topic Name: Modern History
(a) बटलर सिमित Zafar, the king's family would be relocated from the Red Fort to
another loca on in Delhi.
(b) ली कमीशन
Governor-General Lord Canning declared in 1856 that Bahadur Shah
(c) रै ले आयोग Zafar would be the last Mughal king and that none of his progeny
would be recognised as kings a er his death; instead, they would be
(d) एचीसन आयोग
referred to as princes.
(e) उपरो म से कोई नहीं/उपरो म से एक से अिधक
128. गवनर-जनरल ने फैसला िकया िक बहादु र शाह जफर अंितम मुगल
उ र: (c)
राजा होंगे, और उनकी मृ ु के बाद, उनके वंशजों म से िकसी को भी राजा के
िवक (c) सही है :
प म मा ता नहीं दी जाएगी - उ िसफ राजकुमार कहा जाएगा।
1902 म, भारत म िव िव ालयों की थितयों और संभावनाओं म जाने और
िन िल खत म से िकस गवनर-जनरल ने यह िनणय िलया था?
उनके संिवधान और कामकाज म सुधार के उपाय सुझाने के िलए लॉड कजन
(a) लॉड डलहौजी
ारा रै ले आयोग की थापना की गई थी।
(b) लॉड हािडग-I
आयोग ने ाथिमक या मा िमक िश ा पर रपोिटग पर रोक लगा दी। इसकी
(c) लॉड वारे न हे ं
िसफा रशों के आधार पर, भारतीय िव िव ालय अिधिनयम 1904 म पा रत
(d) लॉड एलेनबोरोग
िकया गया था। अिधिनयम के अनुसार,
(e) उपरो म से कोई नहीं/उपरो म से एक से अिधक
● िव िव ालयों को अ यन और अनुसंधान पर अिधक ान दे ना था।
उ र: (e)
● एक िव िव ालय के अ ेताओं की सं ा और कायालय म उनकी अविध
िवक (e) सही है :
कम कर दी गई, और अिधकां श अ ेताओं को सरकार ारा नािमत िकया
कंपनी ने 1840 के दशक म मुगल वंश को समा करने की सािजश रचनी
जाना था।
शु कर दी थी। कंपनी के िस े से मुगल बादशाह का नाम िमटा िदया गया।
● सरकार के पास िव िव ालयों के सीनेट िविनयमों को वीटो करने की श
गवनर-जनरल डलहौजी ने 1849 म घोषणा की िक बहादु र शाह जफर की मृ ु
थी और वह इन िविनयमों म संशोधन कर सकती थी या यं िनयमों को पा रत
पर, राजा के प रवार को लाल िकले से िद ी म िकसी अ थान पर
कर सकती थी।
थानां त रत कर िदया जाएगा।
● िनजी कॉलेजों की संब ता के िलए शत कड़ी की जानी थीं।
गवनर-जनरल लॉड कैिनंग ने 1856 म घोषणा की िक बहादु र शाह ज़फ़र
●उ िश ा और िव िव ालयों के सुधार के िलए पां च साल के िलए पां च लाख
अंितम मुग़ल राजा होंगे और उनकी मृ ु के बाद उनकी िकसी भी संतान को
पये ितवष ीकृत िकए जाने थे।
राजा के प म मा ता नहीं दी जाएगी; इसके बजाय, उ धानों के पम
Q128. The Governor-General decided that Bahadur Shah Zafar would
संदिभत िकया जाएगा।
be the last Mughal king, and a er his death, none of his descendants
would be recognised as kings – they would just be called princes. Q129. Who among the following Governor Generals advocated a
Who one of the following Governors-General decided this? forward policy which implied that the East India Company

(a) Lord Dalhousie government in India itself had to take ini a ves to protect the

(b) Lord Hardinge-I boundary of Bri sh India from a probable Russian a ack?

(c) Lord Warren has ngs (a) Warren Has ngs

(d) Lord Ellenboroug (b) Lord Auckland

(e) None of the above/More than one of the above (c) Lord Cornwallis

Answer: (e) (d) Lord William Ben nck

Op on (e) is correct: The Company began to plot how to bring the (e) None of the above/More than one of the above
Mughal dynasty to an end in the 1840s. The name of the Mughal Answer: (b)
emperor was erased from the Company's coinage. Governor-General Op on (b) is correct: Auckland reached India in 1836. He advocated
Dalhousie declared in 1849 that upon the death of Bahadur Shah forward policy, which implies that the government of East India
Topic Name: Modern History
Company in India would take ini a ves so that fron ers and the (a) He was asked by the Board of Directors in London to do so.
boundary of Bri sh India could be protected from a probable Russian (b) He wanted to revive his interest in oriental learning in India.
a ack. The forward policy was to be implemented through the signing (c) He wanted to provide skill educa on to Indians.
of trea es with neighbouring countries or annexing them completely. (d) He wanted to train Bri sh civilians for administra ve purposes in
Dost Mohammad, The Amir of Afghanistan, asked for Bri sh India.
friendship on the condi on that the Bri sh would help him to recover (e) None of the above/More than one of the above
Peshawar from Sikhs, a condi on which the Bri sh rejected. Dost Answer: (d)
Mohammad asked for the same favour from Russia and Persia. This Op on (d) is correct: Wellesley (governor-general, 1798-1805)
triggered the Bri sh to move with a forwarding policy, and therefore a established the Fort William College in 1800 to train new recruits
tripar te treaty (1838) was signed between the Bri sh, Sikh and Shah (Bri sh civilians for administra on). The Court of Directors rejected
Shuja (who had been deposed from the Afghan throne in 1809 and Wellesley's ins tu on in 1806 and instead established the East India
had been living since then as a Bri sh pensioner at Ludhiana). College at Haileybury, England, to provide two years of training to the

129. िन िल खत म से िकस गवनर जनरल ने आगे की नीित की वकालत की recruits. On July 10, 1800, he established Fort Williams College. The
goal was to teach Bri sh ci zens about Oriental culture, custom, law,
थी, िजसम िनिहत था िक भारत म ई इं िडया कंपनी की सरकार को संभािवत
and administra on so that they might be er coordinate in the
सी हमले से ि िटश भारत की सीमा की र ा के िलए यं पहल करनी होगी? "governance."

(a) वारे न हे ं
130. गवनर-जनरल वेले ी ने कलक ा म फोट िविलयम कॉलेज की थापना
(b) लॉड ऑकलड
की ोंिक:
(c) लॉड कानवािलस
(a) उ लंदन म िनदे शक मंडल ने ऐसा करने के िलए कहा था।
(d) लॉड िविलयम बिटं क
(b) वह भारत म ा िश ा म अपनी िच को पुनज िवत करना चाहता था।
(e) उपरो म से कोई नहीं/उपरो म से एक से अिधक
(c) वे भारतीयों को कौशल िश ा दान करना चाहते थे।
उ र: (b)
(d) वह भारत म शासिनक उ े ों के िलए ि िटश नाग रकों को िशि त
िवक (b) सही है :
करना चाहता था।
ऑकलड 1836 म भारत प ं चा। उ ोंने आगे की नीित की वकालत की,
(e) उपरो म से कोई नहीं/उपरो म से एक से अिधक
िजसका अथ है िक भारत म ई इं िडया कंपनी की सरकार पहल करे गी तािक
उ र: (d)
संभािवत सी हमले से सीमाओं और ि िटश भारत की सीमा की र ा की जा
िवक (d) सही है :
सके। आगे की नीित को पड़ोसी दे शों के साथ संिधयों पर ह ा र करके या
वेले ी (गवनर-जनरल, 1798-1805) ने नए रं ग टों ( शासन के िलए ि िटश
उ पूरी तरह से समा करके लागू िकया जाना था।
नाग रक) को िशि त करने के िलए 1800 म फोट िविलयम कॉलेज की
दो मोह द, अफगािन ान के अमीर, ने इस शत पर ि िटश िम ता के िलए
थापना की। कोट ऑफ डायरे स ने 1806 म वेले ी की सं था को खा रज
कहा िक अं ेज उसे पेशावर को िसखों से वापस लेने म मदद करगे, एक शत
कर िदया और इसके बजाय रं ग टों को दो साल का िश ण दान करने के
िजसे अं ेजों ने अ ीकार कर िदया। दो मोह द ने स और फारस से भी
िलए है लेबरी, इं डमई इं िडया कॉलेज की थापना की। 10 जुलाई, 1800
यही एहसान माँ गा। इसने अं ेजों को अ ेषण नीित के साथ आगे बढ़ने के िलए
को उ ोंने फोट िविलय कॉलेज की थापना की। ल ि िटश नाग रकों को
े रत िकया, और इसिलए ि िटश, िसख और शाह शुजा के बीच एक ि प ीय
ओ रएं टल सं ृ ित, रीित- रवाजों, कानून और शासन के बारे म पढ़ाना था
संिध (1838) पर ह ा र िकए गए (िज 1809 म अफगान िसंहासन से हटा
तािक वे "शासन" म बेहतर सम य कर सक।
िदया गया था और तब से ि िटश पशनभोगी के प म रह रहे थे) लुिधयाना)।

Q131. The doctrine of Lapse was devised by:


Q130. Governor-general Wellesley established the Fort William
(a) Lord Canning
College at Calcu a because:
(b) Lord Dalhousie
Topic Name: Modern History
(c) Lord Clive (e) None of the above/More than one of the above
(d) Lord Curzon Answer: (c)
(e) None of the above/More than one of the above Op on (c) is correct: Significant events during the reign of Lord
Answer: (b) Has ngs (1813-1823):
Op on (b) is correct: Lord Dalhousie, the Governor-General from ● Anglo-Nepal War (1814-16).
1848 to 1856, was in charge of the annexa ons of Indian kings. He ● Treaty of Sagauli, 1816.
created the Doctrine of Lapse, a policy that became well-known. If an ● Third Maratha War (1817-19) and dissolu on of Maratha
Indian ruler died without a male successor, the concept said that his Confederacy; crea on of Bombay Presidency (1818).
kingdom would "lapse" or become part of the Company territory. ● Strife with Pindaris (1817-1818).
Satara (1848), Sambalpur (1850), Udaipur (1852), Nagpur (1853), and ● Treaty with Sindhia (1817).
Jhansi (1853) were only a few of the kingdoms that were annexed ● Establishment of the Ryotwari System by Thomas Munro, Governor
using this theory (1854). Finally, the Company seized Awadh in 1856. of Madras (1820).
The Bri sh had a new reason this me: they said they were "obliged 132. सगौली की संिध, 1816, िकसके शासनकाल म ह ा रत ई थी?
by duty" to take over Awadh in order to rescue the people from the
(a) लॉड िमंटो आई
Nawab's "misgovernment."

131. पगत का िस ां त िकसके ारा तैयार िकया गया था: (b) लॉड एमह

(a) लॉड कैिनंग (c) लॉड हे ं

(b) लॉड डलहौजी (d) लॉड मेटकाफ

(c) लॉड ाइव (e) उपरो म से कोई नहीं/उपरो म से एक से अिधक

(d) लॉड कजन उ र: (c)

(e) उपरो म से कोई नहीं/उपरो म से एक से अिधक िवक (c) सही है :

उ र: (b) लॉड हे ं (1813-1823) के शासनकाल के दौरान मह पूण घटनाएं :

िवक (b) सही है : ● आं -नेपाल यु (1814-16)।

1848 से 1856 तक गवनर-जनरल लॉड डलहौजी, भारतीय राजाओं के िवलय ● सगौली की संिध, 1816।

के भारी थे। उ ोंने डॉ न ऑफ लै नामक नीित बनाई, जो िस ई। ● तीसरा मराठा यु (1817-19) और मराठा संघ का िवघटन; बंबई ेसीडसी

यिद कोई भारतीय शासक िकसी पु ष उ रािधकारी के िबना मर गया, तो का िनमाण (1818)।

अवधारणा ने कहा िक उसका रा " पगत" हो जाएगा या कंपनी के े का ● िपंडा रयों के साथ संघष (1817-1818)।

िह ा बन जाएगा। सतारा (1848), संबलपुर (1850), उदयपुर (1852), नागपुर ● िसंिधया से स (1817)।

(1853), और झां सी (1853) केवल कुछ ही रा थे िज इस िस ां त (1854) ● म ास के गवनर थॉमस मुनरो ारा रै यतवारी व था की थापना (1820)।

Q133.He abolished Sa and other cruel rites, suppressed thugi, and


का उपयोग करके जोड़ा गया था। अंत म, कंपनी ने 1856 म अवध पर क ा
annexa on of Mysore (1831) took place in his tenure. He was:
कर िलया। अं ेजों के पास इस बार एक नया कारण था: उ ोंने कहा िक वे (a) Lord William Ben nck

नवाब के "कुशासन" से लोगों को बचाने के िलए अवध पर क ा करने के िलए (b) Lord Auckland
(c) Lord Ellenborough
"कत से बा " थे।
(d) Lord Dalhousie
(e) None of the above/More than one of the above
Q132. The Treaty of Sagauli, 1816, was signed during the reign of: Answer: (a)
(a) Lord Minto I Op on (a) is correct: Significant events during the reign of Lord
(b) Lord Amherst William Ben nck (1828-1835) are
(c) Lord Has ngs ● Aboli on of Sa and other cruel rites (1829).
(d) Lord Metcalf
Topic Name: Modern History
● Suppression of thuggee (1830). Op on (a) is correct: The conquest of Awadh happened in stages. The
● Charter Act of 1833. Subsidiary Alliance had been imposed on Awadh in 1801. By the terms
● Resolu on of 1835, and educa onal reforms and introduc on of of this alliance, the Nawab had to disband its military force allowing
English as the official language. the Bri sh to posi on their troops within the kingdom and act in
● Annexa on of Mysore (1831), Coorg (1834) and Central Cachar accordance with the advice of the Bri sh residents. Deprived of his
(1834). armed forces, the Nawab became increasingly dependent on the
● Treaty of ‘Perpetual Friendship’ with Ranjeet Singh. Bri sh to maintain law and order within the kingdom. The region was
● Aboli on of the Provincial Courts of appeal and circuit set up by ideally located to be developed into the principal market of Upper
Cornwallis, the appointment of Commissioners of revenue and India and was expected to complete a process of territorial
circuit. annexa on. In 1851 Governor-General Lord Dalhousie described the

133. उ ोंने सती और अ ू र सं ारों को समा कर िदया, ठगी को दबा kingdom of Awadh as “a cherry that will drop into our mouth one
day”. In 1856, Awadh was annexed on the charges of misgovernance
िदया, और मैसूर का िवलय (1831) उनके कायकाल म आ। वह था:
based on the Outram Report by Lord Dalhousie.
(a) लॉड िविलयम बिटं क 134. “एक चेरी जो एक िदन हमारे मुँह म िगरे गी” – इस कथन का संबंध िकससे

(b) लॉड ऑकलड है और िकस संदभ म है ?

(c) लॉड एलेनबरो (a) लॉड डलहौजी अवध के संदभ म

(d) लॉड डलहौजी (b) िसंध के संदभ म चा नेिपयर

(e) उपरो म से कोई नहीं/उपरो म से एक से अिधक (c) मैसूर के संदभ म लॉड वेले ी

उ र: (a) (d) बंगाल के िवभाजन के संदभ म कजन

िवक (a) सही है : (e) उपरो म से कोई नहीं/उपरो म से एक से अिधक

लॉड िविलयम बिटक (1828-1835) के शासनकाल के दौरान मह पूण उ र: (a)

घटनाएं ह िवक (a) सही है :

● सती और अ ू र सं ारों का उ ूलन (1829)। अवध की िवजय चरणों म ई। 1801 म अवध पर सहायक गठबंधन लगाया

● ठगी का दमन (1830)। गया था। इस गठबंधन की शत के अनुसार, नवाब को अपने सै बल को भंग

● 1833 का चाटर अिधिनयम। करना पड़ा, िजससे अं ेज अपने सैिनकों को रा के भीतर तैनात कर सक

● 1835 का संक , और शैि क सुधार और आिधका रक भाषा के प म और ि िटश िनवािसयों की सलाह के अनुसार काय कर सक। अपने सश

अं ेजी का प रचय। बलों से वंिचत, नवाब रा के भीतर कानून और व था बनाए रखने के िलए

● मैसूर (1831), कूग (1834) और सटल कछार (1834) का िवलय। तेजी से अं ेजों पर िनभर हो गया। यह े आदश प से ऊपरी भारत के

● रणजीत िसंह के साथ 'सदा मै ी' की संिध। मुख बाजार के प म िवकिसत होने के िलए थत था और उ ीद की जाती

● कॉनवािलस ारा थािपत अपील और सिकट के ां तीय ायालयों का थी िक यह े ीय िवलय की एक ि या को पूरा करे गा। 1851 म

उ ूलन, राज और सिकट के आयु ों की िनयु । गवनर-जनरल लॉड डलहौजी ने अवध के रा को "एक चेरी जो एक िदन
Q134. “A cherry that will drop into our mouth one day '' - Who is
हमारे मुंह म िगर जाएगी" के प म विणत िकया। 1856 म, लॉड डलहौजी की
associated with this statement, and in what context?
(a) Lord Dalhousie in the context of Awadh आउटाम रपोट के आधार पर अवध को कुशासन के आरोपों पर क ा कर

(b) Charles Napier in the context of Sindh िलया गया था।


(c) Lord Wellesley in the context of Mysore
(d) Curzon in the context of par on of Bengal
Q135. The Gandhi-Irwin Pact was endorsed in the following:
(e) None of the above/More than one of the above
(a) Delhi Session of Congress
Answer: (a)
(b) Amritsar Session of Congress
Topic Name: Modern History
(c) Lucknow Session of Congress (c) Lucknow Session of Congress (1936)
(d) Lahore Session of Congress (d) Haripura Session of Congress (1938)
(e) None of the above/More than one of the above (e) None of the above/More than one of the above
Answer: (e) Answer: (a)
Op on (e) is correct: On February 14, 1931, Viceroy Lord Irwin, Op on (a) is correct: A special session of the Congress was called in
represen ng the Bri sh Indian Government, and Gandhi, represen ng Karachi in March 1931 to approve the Gandhi-Irwin Pact. This was the
the Indian people, signed a contract in Delhi. The Gandhi-Irwin Pact, first me Congress defined swaraj for the masses: "In order to stop
commonly known as the Delhi Pact, put Congress on a level playing mass exploita on, poli cal freedom must entail economic freedom for
field with the government. A special session of the Congress was millions who are starving." In the following years, Congress's primary
called in Karachi in March 1931 to approve the Gandhi-Irwin Pact. poli cal and economic pla orm will be based on the Karachi
Bhagat Singh, Sukhdev, and Rajguru were killed six days before the Resolu on. The Karachi Session of Congress (1931) passed two
session (on March 29). The Punjab Naujawan Bharat Sabha met resolu ons, one on "Fundamental Rights" and the other on "Na onal
Gandhi on his way to Karachi with black flag marches in protest of his Economic Program."
failure to achieve the remission of Bhagat's and his comrade’s death 136. िन िल खत कां ेस अिधवेशनों म से िकस एक म, 'मौिलक अिधकार और
sentence.
रा ीय आिथक काय म' पर संक ीकृत िकए गए थे?
135. िन िल खत म गां धी-इरिवन समझौते का समथन िकया गया था:
(a) कां ेस के कराची स (1931)
(a) कां ेस के िद ी अिधवेशन
(b) कां ेस के बॉ े स (1934)
(b) कां ेस का अमृतसर अिधवेशन
(c) कां ेस का लखनऊ अिधवेशन (1936)
(c) कां ेस का लखनऊ अिधवेशन
(d) कां ेस का ह रपुरा अिधवेशन (1938)
(d) कां ेस का लाहौर अिधवेशन
(e) उपरो म से कोई नहीं/उपरो म से एक से अिधक
(e) उपरो म से कोई नहीं/उपरो म से एक से अिधक
उ र: (a)
उ र: (e)
िवक (a) सही है :
िवक (e) सही है :
गां धी-इरिवन समझौते को मंजूरी दे ने के िलए माच 1931 म कराची म कां ेस
14 फरवरी, 1931 को, ि िटश भारत सरकार का ितिनिध करने वाले
का एक िवशेष स बुलाया गया था। यह पहली बार था जब कां ेस ने जनता के
वायसराय लॉड इरिवन और भारतीय लोगों का ितिनिध करने वाले गां धी ने
िलए राज को प रभािषत िकया: "सामूिहक शोषण को रोकने के िलए,
िद ी म एक अनुबंध पर ह ा र िकए। गां धी-इरिवन पै , िजसे आमतौर पर
राजनीितक तं ता के िलए भूख से मर रहे लाखों लोगों के िलए आिथक
िद ी पै के प म जाना जाता है , ने कां ेस को सरकार के साथ बराबरी का
तं ता आव क है ।" आने वाले वष म, कां ेस का ाथिमक राजनीितक
मौका िदया। गां धी-इरिवन समझौते को मंजूरी दे ने के िलए माच 1931 म कराची
और आिथक मंच कराची ाव पर आधा रत होगा। कां ेस के कराची
म कां ेस का एक िवशेष स बुलाया गया था। स से छह िदन पहले (29 माच
अिधवेशन (1931) म दो ाव पा रत ए, एक "मौिलक अिधकार" पर और
को) भगत िसंह, सुखदे व और राजगु की ह ा कर दी गई थी। पंजाब नौजवान
दू सरा "रा ीय आिथक काय म" पर।
भारत सभा ने भगत और उनके साथी की मौत की सजा को कम करने म
Q137. The Ramgarh Session of the Congress was held under the
उनकी िवफलता के िवरोध म काले झंडे माच के साथ कराची जाने के रा े म leadership of:

गां धी से मुलाकात की। (a) Mo lal Nehru


(b) Chitranjan Das
(c) Maulana Abul Kalam Azad
Q136. In which one of the following Congress sessions, resolu ons
(d) Subhash Chandra Bose
on ‘Fundamental Rights and Na onal Economic Programme’ were
(e) None of the above/More than one of the above
adopted?
Answer: (c)
(a) Karachi Session of Congress (1931)
(b) Bombay Session of Congress (1934)
Topic Name: Modern History
Op on (c) is correct: The Congress's Ramgarh Session was held in service workers, she constantly assisted him and fought for the
March 1940, with Maulana Abul Kalam Azad as President. Everyone in recogni on of the rights of the workers. She also supported and
this Session believed that a war against the Bri sh was necessary, but advanced the cause of the tea cul va on labourers who were
there was a dispute over how to go about it. It was agreed to entrust stranded in Chandpur and were brutally tortured by the Bri sh police.
Gandhi with the ba le's structure and ming. However, Mahatma In 1933, as a recogni on of her contribu on to the party and the
Gandhi stated that he would provide moral support to the Bri sh country, Nellie was elected Congress President.
during WWII, but only on a non-violent basis. Meanwhile, Jawaharlal 138. िन िल खत म से िकसने 1933 म भारतीय रा ीय कां ेस के वािषक
Nehru underlined that Congress's support for the Bri sh war effort
अिधवेशन की अ ता की थी?
must be condi oned on India gaining total independence.

137. कां ेस का रामगढ़ अिधवेशन िकसके नेतृ म आ था? (a) अमृत रणछोड़दास सेठ

(a) मोतीलाल नेह (b) नेली सेनगु ा

(b) िचतरं जन दास (c) राज साद

(c) मौलाना अबुल कलाम आज़ाद (d) जवाहरलाल नेह

(d) सुभाष चं बोस (e) उपरो म से कोई नहीं/उपरो म से एक से अिधक

(e) उपरो म से कोई नहीं/उपरो म से एक से अिधक उ र: (b)

उ र: (c) िवक (b) सही है :

िवक (c) सही है : नेली सेनगु ा महा ा गां धी और सरोिजनी नायडू से े रत थीं। उ ोंने गां धी जी

कां ेस का रामगढ़ अिधवेशन माच 1940 म आ था, िजसके अ मौलाना के नेतृ म असहयोग आं दोलन म भाग िलया और ि िटश िवरोधी गितिविधयों

अबुल कलाम आज़ाद थे। इस अिधवेशन म सभी का मानना था िक अं ेजों के के िलए उ िगर ार कर िलया गया। जब उनके पित बंगाल-असम रे लवे

खलाफ यु ज री है , लेिकन इसे कैसे िकया जाए, इस पर िववाद था। गां धी कमचा रयों की हड़ताल और ीमर सिवस कमचा रयों के आं दोलन म शािमल

को यु की संरचना और समय के साथ सौंपने पर सहमित ई। हालाँ िक, थे, तो उ ोंने लगातार उनकी सहायता की और िमकों के अिधकारों की

महा ा गां धी ने कहा िक वे WWII के दौरान अं ेजों को नैितक समथन दान मा ता के िलए संघष िकया। उ ोंने उन चाय की खेती करने वाले मजदू रों का

करगे, लेिकन केवल अिहं सक आधार पर। इस बीच, जवाहरलाल नेह ने भी समथन िकया और उ आगे बढ़ाया, जो चां दपुर म फंसे ए थे और ि िटश

रे खां िकत िकया िक ि िटश यु के यासों के िलए कां ेस का समथन भारत को पुिलस ारा ू रता से तािड़त िकए गए थे। 1933 म, पाट और दे श म उनके

पूण तं ता ा करने पर िनभर होना चािहए। योगदान की मा ता के प म, नेली को कां ेस अ चुना गया।

Q139. Who of the following was the president of the Indian Na onal

Q138. Who among the following presided over the annual session of Congress when the Surat Split (1907) happened?
the Indian Na onal Congress in 1933? (a) Rash Behari Ghosh

(a) Amrit Ranchhorddas Seth (b) Mo lal Nehru

(b) Nellie Sengupta (c) Maulana Mohammad Ali

(c) Rajendra Prasad (d) Acharya J.B. Kripalani

(d) Jawaharlal Nehru (e) None of the above/More than one of the above

(e) None of the above/More than one of the above Answer: (a)

Answer: (b) Op on (a) is correct: During the Surat Split, Rash Behari Ghosh

Op on (b) is correct: Nellie Sengupta was inspired by Mahatma presided over a Congress session (1907). In 1907, at the Surat Session,
Gandhi and Sarojini Naidu. She par cipated in the Non-Coopera on a split in Congress arose on the issue of the Presidentship of Congress.
Movement led by Gandhi Ji and was arrested for her an -Bri sh The radicals wanted the 1907 Congress to be held at Nagpur (Central
ac vity. When her husband was involved in the strike of the Provinces), with Tilak or Lajpat Rai as President and a reaffirma on of
Bengal-Assam Railway workers and in the movement of steamer the Swadeshi boyco and na onal educa on resolu ons. Because a
poli cian from the host Province could not be Session President (Surat
Topic Name: Modern History
being in Tilak's home province of Bombay), the Moderates wanted the 25, 1931. The CWC permi ed Gandhi to begin talks with the viceroy.
Session to be held in Surat in order to keep Tilak out of the presidency. On February 14, 1931, the Viceroy, represen ng the Bri sh Indian
Instead, moderates wanted Rash Behari Ghosh as the President and Government, and Gandhi, represen ng the Indian people, signed a
sought to drop the resolu ons on Swadeshi, boyco and na onal contract in Delhi as a result of their conversa ons. The Gandhi-Irwin
educa on. Both moderates and radicals adopted rigid posi ons, Pact, also known as the Delhi Pact. The Karachi Congress Session
leaving no room for compromise, which led to the split. (March 1931) endorsed the Delhi Pact or Gandhi-Irwin Pact.

139. सूरत िवभाजन (1907) के समय भारतीय रा ीय कां ेस के अ 140. भारतीय रा ीय कां ेस के िन िल खत म से िकस अिधवेशन ने

िन िल खत म से कौन थे? गां धी-इरिवन समझौते का समथन िकया?

(a) रास िबहारी घोष (a) लाहौर (1929)

(b) मोतीलाल नेह (b) कराची (1931)

(c) मौलाना मोह द अली (c) िद ी (1932)

(d) आचाय जे.बी. कृपलानी (d) कलक ा (1933)

(e) उपरो म से कोई नहीं/उपरो म से एक से अिधक (e) उपरो म से कोई नहीं/उपरो म से एक से अिधक

उ र: (a) उ र: (b)

िवक (a) सही है : िवक (b) सही है :

सूरत िवभाजन के दौरान, रास िबहारी घोष ने कां ेस अिधवेशन (1907) की गां धी और कां ेस काय सिमित (सीड ूसी) के अ सद ों को 25 जनवरी,

अ ता की थी। 1907 म, सूरत अिधवेशन म, कां ेस के अ पद के मु े 1931 को िबना शत रहा कर िदया गया। सीड ूसी ने गां धी को वायसराय के

पर कां ेस म िवभाजन हो गया। क रपंथी चाहते थे िक 1907 की कां ेस साथ बातचीत शु करने की अनुमित दी। 14 फरवरी, 1931 को, वायसराय,

नागपुर (म ां त) म आयोिजत की जाए, िजसम ितलक या लाजपत राय ि िटश भारत सरकार का ितिनिध करते ए, और गां धी, भारतीय लोगों का

अ हों और दे शी बिह ार और रा ीय िश ा ावों की पुि हो। ितिनिध करते ए, उनकी बातचीत के प रणाम प िद ी म एक

ोंिक मेजबान ां त का एक राजनेता स अ नहीं हो सकता था (सूरत अनुबंध पर ह ा र िकए। गां धी-इरिवन पै , िजसे िद ी पै के नाम से भी

ितलक के गृह ां त बंबई म होने के कारण), उदारवादी चाहते थे िक ितलक को जाना जाता है । कराची कां ेस अिधवेशन (माच 1931) ने िद ी पै या

रा पित पद से बाहर रखने के िलए स सूरत म आयोिजत िकया जाए। इसके गां धी-इरिवन पै का समथन िकया।

बजाय, नरमपंथी रास िबहारी घोष को अ बनाना चाहते थे और उ ोंने


Q141. Who among the following was elected as the president of the
दे शी, बिह ार और रा ीय िश ा पर ावों को छोड़ने की मां ग की।
Haripura Session of Congress?
नरमपंथी और क रपंथी दोनों ने कठोर थित अपनाई, समझौते के िलए कोई (a) Jawaharlal Nehru

(b) Dadabhai Naoroji


जगह नहीं छोड़ी, िजससे िवभाजन आ।
(c) Gopalkrishna Gokhale
(d) Mahatma Gandhi
Q140. Which one of the following sessions of the Indian Na onal
(e) None of the above/More than one of the above
Congress endorsed the Gandhi-Irwin pact?
Answer: (e)
(a) Lahore (1929)
Op on (e) is correct: At the Congress mee ng in Haripura, Gujarat, in
(b) Karachi (1931)
February 1938, Bose was unanimously elected president of the
(c) Delhi (1932)
session. He was firm in his belief that the Congress ministries in the
(d) Calcu a (1933)
provinces had immense revolu onary poten al, as he said in his
(e) None of the above/More than one of the above
presiden al address. The session adopted a resolu on that Congress
Answer: (b)
would give moral support to those who were agita ng against the
Op on (b) is correct: Gandhi and the other members of the Congress
governance in the princely states.
Working Commi ee (CWC) were released uncondi onally on January
Topic Name: Modern History
141. िन िल खत म से िकसे कां ेस के ह रपुरा अिधवेशन का अ चुना गया (e) उपरो म से कोई नहीं/उपरो म से एक से अिधक

था? उ र: (a)

(a) जवाहरलाल नेह िवक (a) सही है :

(b) दादाभाई नौरोजी बंगाल गजट या कलक ा जनरल एडवटाइजर की थापना 1780 म जे

(c) गोपालकृ गोखले ऑग स िह ी ने की थी और यह भारत का पहला अखबार था। सरकार की

(d) महा ा गां धी मुखर आलोचना के कारण इसे 1872 म ज कर िलया गया था। द बंगाल

(e) उपरो म से कोई नहीं/उपरो म से एक से अिधक जनल, द कलक ा ॉिनकल, द म ास कू रयर और द बॉ े हे रा सबसे

उ र: (e) पहले छपने वाले अखबारों/जन म से थे। कंपनी के अिधका रयों को िचंता थी

िवक (e) सही है : िक ये अखबार लंदन प ं चकर अपनी करतूतों का खुलासा कर दगे।

फरवरी 1938 म गुजरात के ह रपुरा म कां ेस की बैठक म बोस को सवस ित प रणाम प, उ ेस ितबंधों की आव कता महसूस ई।

से अिधवेशन का अ चुना गया। जैसा िक उ ोंने अपने अ ीय भाषण म


Q143. The famous book ‘The Philosophy of the Bomb’ was wri en
कहा, वे अपने इस िव ास पर अिडग थे िक ां तों म कां ेस मंि मंडलों म अपार
by:
ां ितकारी मता है । अिधवेशन ने एक ाव पा रत िकया िक कां ेस उन (a) Bhagwa Charan Vohra

(b) Durga Devi


लोगों को नैितक समथन दे गी जो रयासतों म शासन के खलाफ आं दोलन कर
(c) Bhagat Singh
रहे ह।
(d) Chandrashekhar Azad
(e) None of the above/More than one of the above
Q142. Who among the following started the newspaper ‘The Bengal Answer: (a)
Gaze e’? Op on (a) is correct: Bhagwa charan Vohra wrote 'The Philosophy of
(a) James Augustus Hickey the Bomb.' He made contact with the newly formed Communist Party
(b) John Adams of India and began smuggling M. N. Roy's monthly, Vanguard and
(c) Rammohan Roy other Marxist material into the country. However, he became
(d) G. Subramaniya Aiyar disillusioned with the young communists' lack of ac on and was
(e) None of the above/More than one of the above drawn instead to the underground armed resistance organised by the
Answer: (a) Hindustan Republican Associa on (HRA).
Op on (a) is correct: The Bengal Gaze e or Calcu a General 143. िस पु क 'द िफलॉसफी ऑफ द बॉ ' िकसके ारा िलखी गई थी:
Adver ser was founded in 1780 by James Augustus Hickey and was
(a) भगवती चरण वोहरा
the first newspaper in India. It was seized in 1872 because of its
outspoken cri cism of the Government. The Bengal Journal, The (b) दु गा दे वी
Calcu a Chronicle, The Madras Courier, and The Bombay Herald were
(c) भगत िसंह
among the first newspapers/journals to appear. The officers of the
Company were concerned that these newspapers would reach London (d) चं शेखर आजाद
and reveal their wrongdoings. As a result, they felt the need for press (e) उपरो म से कोई नहीं/उपरो म से एक से अिधक
restric ons.
उ र: (a)
142. िन िल खत म से िकसने 'द बंगाल गजट' अखबार शु िकया था?
िवक (a) सही है :
(a) जे ऑग स िह ी
भगवतीचरण वोहरा ने 'द िफलॉसफी ऑफ द बम' िलखा था। उ ोंने नवगिठत
(b) जॉन एड
भारतीय क ुिन पाट के साथ संपक बनाया और दे श म एमएन रॉय की
(c) राममोहन राय
मािसक, मोहरा और अ मा वादी साम ी की त री शु कर दी।
(d) जी. सु म मअ र
Topic Name: Modern History
हालाँ िक, युवा क ुिन ों की कारवाई म कमी से उनका मोहभंग हो गया और काय का िव ार करने म मदद की। अबुल कलाम आजाद उनके सहायक थे।

इसके बजाय िहं दु ान रप कन एसोिसएशन (HRA) ारा आयोिजत उनकी सहायता अबुल कलाम आज़ाद ने की थी।

भूिमगत सश ितरोध की ओर आकिषत ए।


Q145. Who among the following started a Workingmen’s club and
Q144. Who among the following was associated with Amrit Bazar
Bharat Shramjeevi newspaper?
Patrika?
(a) Sasipada Banerjea
(a) Mo lal Ghosh
(b) Sorabjee Shapoorji Bengalee
(b) Acharya P.C. Roy
(c) Narain Meghajee Lokhanday
(c) Brahmabandhab Upadhyay
(d) G. Subramanya Aiyar
(d) Abdul Halim Guznavi
(e) None of the above/More than one of the above
(e) None of the above/More than one of the above
Answer: (a)
Answer: (a)
Op on (a) is correct: In Bengal, Sasipada Banerjea, a Brahmo social
Op on (a) is correct: Mo lal Ghosh of the Amrita Bazar Patrika group
reformer, founded a Workingmen's Club in 1870 and published Bharat
wrote several fiery pieces in the newspaper to raise patrio c feelings
Shramjeevi, a monthly periodical (Indian Labour).
and was a supporter of Extremism.
In 1878, Sorabjee Shapoorji Bengalee a empted to enact a law in the
Brahmabandhab Upadhyay popularised swaraj and the Swadeshi
Bombay Legisla ve Council that would improve labour condi ons.
Movement with his Sandhya and Yugantar (published by a group
Narain Meghajee Lokhanday founded the Bombay Mill and Millhands
linked with Barindrakumar Ghosh). To promote swadeshi, Acharya P.C.
Associa on and the periodical Deenbandhu in 1880.
Roy set up the Bengal Chemicals Factory. Abdul Halim Guznavi, a
zamindar and lawyer, helped Aurobindo Ghosh expand revolu onary 145. िन िल खत म से िकसने विकगमे ब और भारत मजीवी अखबार

opera ons beyond Bengal by establishing swadeshi companies. Abul शु िकया?


Kalam Azad was his assistant. He was assisted by Abul Kalam Azad.
(a) शिसपदा बनज
144. िन िल खत म से कौन अमृत बाजार पि का से जुड़ा था?
(b) सोराबजी शापुरजी बंगाली
(a) मोतीलाल घोष
(c) नारायण मेघाजी लोखंडे
(b) आचाय पी.सी. रॉय
(d) जी. सु म अ र
(c) बंधब उपा ाय
(e) उपरो म से कोई नहीं/उपरो म से एक से अिधक
(d) अ ु ल हलीम गुज़नवी
उ र: (a)
(e) उपरो म से कोई नहीं/उपरो म से एक से अिधक
िवक (a) सही है :
उ र: (a)
बंगाल म, ो समाज सुधारक शिसपदा बनज ने 1870 म एक विकगमेन
िवक (a) सही है :
ब की थापना की और एक मािसक आविधक (भारतीय म) भारत
अमृत बाजार पि का समूह के मोतीलाल घोष ने दे शभ की भावनाओं को
मजीवी कािशत िकया।
जगाने के िलए अखबार म कई लंत अंश िलखे और वे अितवाद के समथक
1878 म, सोराबजी शापुरजी बगाली ने बॉ े िवधान प रषद म एक कानून
थे।
बनाने का यास िकया जो म की थित म सुधार करे गा।
बंधब उपा ाय ने अपनी सं ा और युगां तर (ब रं कुमार घोष से जुड़े एक
नारायण मेघाजी लोखंडे ने 1880 म बॉ े िमल एं ड िमलहड् स एसोिसएशन और
समूह ारा कािशत) के साथ राज और दे शी आं दोलन को लोकि य
आविधक दीनबंधु की थापना की।
बनाया। दे शी को बढ़ावा दे ने के िलए आचाय पी.सी. रॉय ने बंगाल केिमक
Q146. The book named 'Gulamgiri', seminal on women, caste and
फै ी की थापना की। एक ज़मींदार और वकील अ ु ल हलीम गुज़नवी ने reform, was published by?

दे शी कंपिनयों की थापना करके अरिबंदो घोष को बंगाल से परे ां ितकारी (a) Gopal Baba Walangkar
(b) Jyo ba Phule
(c) Mohandas Karamchand Gandhi
Topic Name: Modern History
(d) Bhimrao Ramji Ambedkar 147. सामा सेवा िवि अिधिनयम 1856 के िलए दान िकया गया:
(e) None of the above/More than one of the above
(a) उ ोगों के बंधन म िमकों की भागीदारी।
Answer: (b)
Op on (b) is correct: In 1873, Phule wrote a book named Gulamgiri, (b) औ ोिगक िववादों को दबाने के िलए बंधन को मनमानी श यां ।

meaning slavery. Phule dedicated his book to all those Americans who
(c) आव कता पड़ने पर भारतीय सैिनकों को कहीं भी सेवा करने के िलए
had fought to free slaves, thus establishing a link between the
अिनवाय करना।
condi ons of the “lower” castes in India and the black slaves in
America. (d) ायािधकरणों की एक णाली और हड़तालों पर ितबंध।

146. मिहलाओं, जाित और सुधार पर मौिलक पु क 'गुलामिगरी' िकसके ारा (e) उपरो म से कोई नहीं/उपरो म से एक से अिधक

कािशत की गई थी? उ र: (c)

(a) गोपाल बाबा वालंगकर िवक (c) सही है :

(b) ोितबा फुले समु पार करने का मतलब उस समय के धािमक िहं दुओं के िलए अपनी जाित

(c) मोहनदास करमचंद गां धी को खोना था। लॉड कैिनंग की सरकार ने 1856 म जनरल सिवस एनिल मट

(d) भीमराव रामजी अ ेडकर ए पा रत िकया, िजसम बंगाल सेना म भिव की सभी भितयों की

(e) उपरो म से कोई नहीं/उपरो म से एक से अिधक आव कता थी, जहाँ सरकार उनकी सेवाएँ चाहती थी, वहाँ सेवा करने का

उ र: (b) आ ासन दान करना। इसको लेकर नाराजगी थी। यह 1857 की ां ित के

िवक (b) सही है : कारणों म से एक था।

1873 म, फुले ने गुलामिगरी नाम की एक िकताब िलखी, िजसका अथ गुलामी अपने ि िटश सहयोगी की तुलना म भारतीय िसपाही अपने वेतन से समान प

है । फुले ने अपनी पु क उन सभी अमरीिकयों को समिपत की िज ोंने दासों से असंतु था। यह फरमान िक िसंध या पंजाब म सेवा करते समय उ िवदे श

को मु कराने के िलए लड़ाई लड़ी थी, इस कार भारत म "िन " जाितयों की सेवा भ ा (भ ा) नहीं िदया जाएगा, िसपािहयों के िलए नाराजगी का एक और

थितयों और अमे रका म काले दासों के बीच एक कड़ी थािपत की| त ाल ोत था। कई िसपािहयों के घर अवध के अिध हण ने उनकी भावनाओं

को और भी अिधक उ ेिजत कर िदया।


Q147. General Service Enlistment Act 1856 provided for:
(a) The par cipa on of workers in the management of industries.
Q148. The objec ve of the Queen’s Proclama on of 1858 was to:
(b) Arbitrary powers to the management to quell industrial disputes.
(a) making Governor-General of India as director of East India
(c) Manda ng Indian soldiers to serve anywhere if required.
Company
(d) A system of tribunals and a ban on strikes.
(b) Transfer the powers of the East India Company to the Bri sh
(e) None of the above/More than one of the above
Crown
Answer: (c)
(c) Reform the army of Bri sh India
Op on (c) is correct: Crossing the seas meant losing one's caste to the
(d) establishing the rule of law in India
religious Hindus of the me. Lord Canning's government passed the
(e) None of the above/More than one of the above
General Service Enlistment Act in 1856, requiring all future recruits to
Answer: (b)
the Bengal Army to provide an assurance to serve wherever the
Op on (b) was correct: On August 2, 1858, the Bri sh Parliament
government wanted their services. There was resentment as a result
approved an Act for the Be er Government of India. The Act
of this. It was one of the causes of the 1857 revolu on.
established Queen Victoria as the sovereign of Bri sh India and
In comparison to his Bri sh colleague, the Indian sepoy was equally
established a Secretary of State for India (a member of the Bri sh
dissa sfied with his pay. The edict that they would not be awarded
cabinet). The Bri sh Crown seized direct responsibility for the
the foreign service allowance (Bha a) when serving in Sindh or Punjab
country's governance, and Company control was dissolved. Lord
was a more immediate source of displeasure for the sepoys. The
Canning declared the acceptance of the Government of India by the
acquisi on of Awadh, the home of numerous sepoys, aggravated their
feelings even more.
Topic Name: Modern History
sovereign of Great Britain at a durbar in Allahabad on November 1, Q149. Which of the following best explains the " nkathia" system in
1858, in the 'Queen's Proclama on.' Bri sh India?
In 1858, the Bri sh Parliament approved a new Act transferring the (a) Champaran tenants were bound by law to plant three out of
East India Company's responsibili es to the Bri sh Crown, ensuring every ten parts of their land.
more responsible management of Indian affairs. A member of the (b) Champaran tenants were bound by law to plant three out of
Bri sh Cabinet was named Secretary of State for India, with every nine parts of their land.
responsibility for all aspects of India's governance. He was granted the (c) Champaran tenants were bound by law to plant three out of
India Council as a council to advise him. The Governor-General of every fi een parts of their land.
India was granted the tle of Viceroy, implying that he was the (d) Champaran tenants were bound by law to plant three out of
Crown's personal representa ve. The Bri sh government accepted every twenty one parts of their land.
direct responsibility for the rule of India as a result of these policies. (e) None of the above/More than one of the above

148. 1858 की रानी की उद् घोषणा का उ े था: Answer: (e)


Op on (e) is correct: Champaran was a district in Northern Bihar.
(a) ई इं िडया कंपनी के िनदे शक के प म भारत के गवनर-जनरल बनाना
When Gandhi was called there, it was virtually under the rule of
(b) ई इं िडया कंपनी की श यों को ि िटश ताज म थानां त रत कर European indigo planters. They cruelly exploited and terrorised the
tenants. Under the ' nkathia' system, the tenants had to cul vate
(c) ि िटश भारत की सेना म सुधार
indigo in the 3/20th part of the land. The tenants were oppressed and
(d) भारत म कानून के शासन की थापना करना
fear-stricken. The Bri sh administra on supported the planters.
(e) उपरो म से कोई नहीं/उपरो म से एक से अिधक 149. िन िल खत म से कौन ि िटश भारत म "तीनकिठया" णाली की सबसे

उ र: (b) अ ी ा ा करता है ?

िवक (b) सही था: (a) चंपारण के िकरायेदार कानून ारा बा थे िक वे अपनी भूिम के ेक दस

2 अग , 1858 को, ि िटश संसद ने भारत की बेहतर सरकार के िलए एक भागों म से तीन पौधे लगाएँ ।

अिधिनयम को मंजूरी दी। इस अिधिनयम ने रानी िव ो रया को ि िटश भारत (b) चंपारण के िकरायेदार कानून ारा बा थे िक वे अपनी जमीन के नौ िह ों

के सं भु के प म थािपत िकया और भारत के िलए एक रा सिचव (ि िटश म से तीन पौधे लगाएं गे।

कैिबनेट का सद ) की थापना की। ि िटश ताज ने दे श के शासन की सीधी (c) चंपारण के िकरायेदार कानून ारा बा थे िक वे अपनी भूिम के ेक

िज ेदारी अपने हाथ म ले ली और कंपनी का िनयं ण भंग कर िदया गया। पं ह भागों म से तीन पौधे लगाएं गे।

लॉड कैिनंग ने 1 नवंबर, 1858 को 'रानी की उद् घोषणा' म इलाहाबाद के एक (d) चंपारण के का कार अपनी भूिम के ेक इ ीस भागों म से तीन पौधे

दरबार म ेट ि टे न के सं भु ारा भारत सरकार की ीकृित की घोषणा की। लगाने के िलए कानूनन बा थे।

1858 म, ि िटश संसद ने भारतीय मामलों के अिधक िज ेदार बंधन को (e) उपरो म से कोई नहीं/उपरो म से एक से अिधक

सुिनि त करने के िलए ई इं िडया कंपनी की िज ेदा रयों को ि िटश ाउन उ र: (e)

को थानां त रत करने के िलए एक नए अिधिनयम को मंजूरी दी। ि िटश िवक (e) सही है :

मंि मंडल के एक सद को भारत के रा सिचव के प म नािमत िकया चंपारण उ री िबहार का एक िजला था। जब गां धी को वहां बुलाया गया था, तो

गया था, िजसके पास भारत के शासन के सभी पहलुओं की िज ेदारी थी। उ वह वा व म यूरोपीय नील बागान मािलकों के शासन के अधीन था। उ ोंने

सलाह दे ने के िलए एक प रषद के पमउ भारत प रषद दान की गई थी। ू रता से िकरायेदारों का शोषण िकया और उ आतंिकत िकया। तीनकिठया

भारत के गवनर-जनरल को वायसराय की उपािध दान की गई, िजसका अथ व था के तहत का कारों को जमीन के 3/20व िह े म नील की खेती

था िक वह ाउन का िनजी ितिनिध था। इन नीितयों के प रणाम प करनी पड़ती थी। िकरायेदारों पर अ ाचार िकया गया और वे भयभीत थे।

ि िटश सरकार ने भारत के शासन के िलए उ रदािय ीकार िकया। ि िटश शासन ने बागान मािलकों का समथन िकया।
Topic Name: Modern History
Q150. In the context of Indian history, ‘India Council’ refers to: (a) To solve the poli cal deadlock between the All-India Muslim
(a) Advisory council for Governor-General of India League and the Indian Na onal Congress
(b) Council that managed the commercial affairs of East India (b) For the forma on of a joint interim government
Company (c) For the immediate transfer of power to the Indian Government
(c) Governor-General in Council along with four Councillors (d) Expansion of Viceroy’s council and Indian majority
(d) Advisory council for Secretary of State for India (e) None of the above/More than one of the above
(e) None of the above/More than one of the above Answer: (a)
Answer: (d) Op on (a) is correct: Rajagopalachari’s formula (The way out) was
Op on (d) is correct: On August 2, 1858, the Bri sh Parliament formulated to resolve the deadlock between the All India Muslim
approved an Act for the Be er Government of India. The Act League and the INC. According to it, Congress and Muslim League
established Queen Victoria as Bri sh India's sovereign. The Bri sh must unite and fight against the Bri sh and called upon Congress to
Crown seized direct responsibility for the country's governance, and offer the league the Muslim na on Pakistan based on a plebiscite, but
Company control was dissolved. A member of the Bri sh Cabinet was it failed to make any progress.
named Secretary of State for India, with responsibility for all aspects
of India's governance. He was granted the India Council as a council to
advise him.
The Government of India Act of 1935 eliminated the Secretary of
State's India Council.

150. भारतीय इितहास के संदभ म, 'इं िडया काउं िसल' संदिभत करता है :

(a) भारत के गवनर-जनरल के िलए सलाहकार प रषद

(b) प रषद जो ई इं िडया कंपनी के वािण क मामलों का बंधन करती थी

(c) चार पाषदों के साथ काउं िसल म गवनर-जनरल

(d) भारत के रा सिचव के िलए सलाहकार प रषद

(e) उपरो म से कोई नहीं/उपरो म से एक से अिधक

उ र: (d)

िवक (d) सही है :

2 अग , 1858 को, ि िटश संसद ने भारत की बेहतर सरकार के िलए एक

अिधिनयम को मंजूरी दी। इस अिधिनयम ने रानी िव ो रया को ि िटश भारत

की सं भुता के प म थािपत िकया। ि िटश ताज ने दे श के शासन की सीधी

िज ेदारी अपने हाथ म ले ली और कंपनी का िनयं ण भंग कर िदया गया।

ि िटश मंि मंडल के एक सद को भारत के रा सिचव के प म नािमत

िकया गया था, िजसके पास भारत के शासन के सभी पहलुओं की िज ेदारी

थी। उ सलाह दे ने के िलए एक प रषद के पमउ भारत प रषद दान

की गई थी।

1935 के भारत सरकार अिधिनयम ने रा के भारत प रषद के सिचव को हटा

िदया।

BACKUP QUESTIONS:
What was the basic aim of the C.R formula of 1944?

You might also like